当前位置:文档之家› 内科学题库

内科学题库

内科学题库
内科学题库

内科学国家题库 2010-6-3 目录 一.呼吸内科试题 ........................................................................................................................................................................................................................................................ 0 A 题型 ........................................................................................................................................................................................................................................................................... 0 B 型题: ...................................................................................................................................................................................................................................................................... 3 二.循环内科试题 ........................................................................................................................................................................................................................................................ 4 A 题型 ........................................................................................................................................................................................................................................................................... 4 B 型题: ...................................................................................................................................................................................................................................................................... 7 三.消化内科试题 ........................................................................................................................................................................................................................................................ 7 A 题型 ........................................................................................................................................................................................................................................................................... 7 B 型题 ........................................................................................................................................................................................................................................................................ 10 四.泌尿内科试题 ...................................................................................................................................................................................................................................................... 11 A 题型 ......................................................................................................................................................................................................................................................................... 11 B 型题: .................................................................................................................................................................................................................................................................... 14 五.血液内科试题 ...................................................................................................................................................................................................................................................... 15 A 题型 ......................................................................................................................................................................................................................................................................... 15 B 型题 ........................................................................................................................................................................................................................................................................ 18 六.内分泌内科试题 ................................................................................................................................................................................................................................................... 19 A 题型 ......................................................................................................................................................................................................................................................................... 19 B 型题: .. (21)

一.呼吸内科试题

A 题型

1、下述克雷白杆菌肺炎的临床表现中,哪一项诊断价值最大?

A 、呼吸困难、紫绀

B 、寒战、高热

C 、早期可出现休克

D 、红砖色胶冻样痰

E 、胸痛

2、支气管哮喘诊断的最主要依据是: A 、动脉血氧分压降低,二氧化碳分压升高 B 、血液中嗜酸性粒细胞增加 C 、胸部透视,肺透光度增加,横膈下降 D 、有反复发作,呼气性呼吸困难和肺部有哮鸣音 E 、肺功能测验有阻塞通气障碍

3、男性,32 岁,自幼患发作性呼吸困难,发作时经用药数小时至数日缓解,查 BP :(40/90mmHg ),心界 不大,心尖部 I 级收缩期杂音,律齐,双肺布满哮鸣音,肝脏不大,下肢不肿,诊断首先应考虑: A 、慢性支气管炎 B 、支气管扩张症 C 、风湿性心脏病左心衰竭 D 、支气管哮喘 E 、高血压性心脏病

4、慢性支气管炎的临床最重要的诊断依据是: A 、长期反复发作的呼吸困难,肺部有普遍性的干罗音 B 、咳嗽、咯痰或伴喘息反复发作,每年有三个月,连

续两年以上 C 、长期咳嗽、咯血、胸痛、呼吸困难,伴低热乏力,食欲差和体

重下降 D 、反复呼吸道感染,进行性呼吸困难,伴缺氧和二氧化碳潴留的症状 E 、自幼发病,咳嗽、咯脓痰、咯血、反复呼吸道感染 5、下列哪项不符合克雷白杆菌肺炎(肺炎杆菌肺炎)的临床特点? A 、多见于老年,营养不良者 B 、咯棕色胶冻状痰 C 、青壮年多见,预后好 D 、中毒症状重,预后差 E 、多见于院内感染 6、浸润性肺结核最好发的部位是: A 、上叶前段 B 、上叶尖后段 C 、下叶内基底段 D 、中叶内侧段 E 、中叶外侧段 7、对慢性阻塞性肺气肿诊断最有意义的检查是: A 、血气分析 B 、胸部 X 线检查 C 、心电图和心电向量检查 D 、肺功能检查 E 、肺动脉压测定 8、下列各项中哪一项不属于慢性支气管炎的诊断标准? A 、反复发作的咳嗽、咯痰或喘息 B 、每年患病持续三个月以上 C 、连续两年以上 D 、有肺气肿的体征 E 、除外其他心肺疾患 9、慢性支气管炎最主要的诊断标准是: A 、凡有咳嗽、咯痰或伴喘息反复发作,每年患病至少一个月以上,持续二个月以上,并排除其它心肺疾 病者 B 、凡有咳嗽、咯痰或伴喘息反复发作,每年患病至少 2 个月以上,持续三年以上,并排除 其它心肺疾病者 C 、凡有咳嗽、咯痰或伴喘息反复发作,每年患病至少三个月以上,持续一年以上, 并排除其它心肺疾病者 D 、凡有咳嗽、咯痰或伴喘息反复发作,每年患病至少 3 个月以上,持续三年以上,并排除其它心肺疾病 者 E 、凡有咳嗽、咯痰或伴喘息反复发作,每年患病至少 3 个月以上,持续 2 年以上,并排除其它心肺疾病 者 10、男、26 岁,农民,哮喘严重发作 24 小时以上,下列哪项不符合发作时的

临床表现: A 、张口呼吸 B 、大量出汗 C 、紫绀明显 D 、端坐呼吸 E 、

四肢厥冷 11、慢性支气管炎的临床分型是: A 、单纯型、喘息型、混合型 B 、急性型、慢性型 C 、急性型、迁延型 D 、急性型、慢性型、迁延型 E 、单纯型、喘息型 12、慢性支气管炎的最重要的临床诊断标准是: (略) 13、肺炎球菌肺炎的病理改变中下列哪项是错误的? A 、肺组织充血水肿、肺泡内浆液渗出 B 、肺泡内大量红细胞、白细胞浸润 C 、肺泡内大量白细胞、纤维蛋白渗出 D 、肺泡内纤维蛋白渗出物溶解吸收、肺泡重新充气 E 、肺组织坏死形成空洞 14、诊断呼吸衰竭的血气标准是: A 、PaO2<65mmHg 和(或)伴有 PaCO2>55mmHg B 、PaO2<50mmHg 和(或)伴有 PaCO2>60mmHg C 、PaO2<60mmHg 和(或)伴有 PaCO2>55mmHg D 、PaO2<60mmHg 和(或)伴有 PaCO2>50mmHg E 、PaO2<65mmHg 和

(或)伴有 PaCO2>50mmHg

15、哪项肺能测定对诊断阻塞性肺气肿最有价值? A 、潮气量 B 、肺活量 C 、残气容积占肺总量百分比>40% D 、动脉血氧分压 E 、每分钟静息通气量 16、慢性支气管炎患者剧烈咳嗽后呼吸困难,左肺呼吸音低,叩诊呈现鼓音,应首选哪项检查? A 、血气分析 B 、胸部 X 线透视 C 、痰细菌培养+药敏 D 心电图 E 、胸部 CT 扫描 17、支气管哮喘患者突然出现胸痛、气急、呼吸困难、大汗不安,首先应考虑? A 、自发性气胸 B 、支气管哮喘急性发作 C 、左心衰竭 D 、肺炎 E 、胸膜炎 18、以下叙述的金黄色葡萄球菌肺炎的临床表现中,哪一项是不正确的? A 、寒战、高热 B 、巧克力样痰 C 、呼吸困难 D 、早期可出现休克 E 、肺部有湿性罗音

19、男性,58 岁,有慢性咳嗽、咯痰史 15 年,1 周来高热、咯红砖色胶冻样痰,伴气急紫绀,谵妄,本 例可能性最大的诊断是: A 、肺炎球菌肺炎 B 、克雷白杆菌肺炎 C 、浸润型肺结核 D 、病毒性肺炎 E 、支原体肺炎 20、下面所述张力性自发性气胸患者的临床表现中,哪项最具有诊断意义? A 、突发性呼吸困难 B 、突发性胸痛 C 、气管移向健侧 D 、患侧叩诊呈鼓音 E 、患侧呼吸音减低 21、控制肺结核流行的最具有决定性的措施是: A 、隔离,消毒 B 、加强卫生宣教 C 、合理化疗 D 、卡介苗接种 E 、作好痰的处理 22、肺结核病人发热的最典型热型是: A 、稽留热 B 、驰张热 C 、波浪热 D 、间歇热 E 、午后抵热 23、结核性渗出性胸膜炎伴有大量胸腔积液时,以下体征中哪一项不正确: A 、患侧胸廓饱满 B 、气管向健侧移位 C 、病变部位语颤增强 D 、患者呼吸音减低 E 、患侧叩诊呈浊音 24、慢性阻塞性肺气肿常见的并发症,下列哪一项是不符合的? A 、自发性气胸 B 、肺部急性感染 C 、慢性肺源性心脏病 D 、肺性脑病 E 、肺结核 25、下列是慢性呼吸衰竭的发病机理,除了? A 、通气不足 B 、大量胸腔积液 C 、通气/血流比例失调 D 、气体弥散障碍 E 、肺结核 26、慢性咳嗽、咯痰 30 年,加重伴尿少 2 天,血气分析 PH7.15,PaO2:36mmHg ,PaCO2:75mmHg ,BE :—16mmol/l , 最可能的诊断是: A 、呼吸性酸中毒 B 、代谢性酸中毒 C 、呼吸性酸中毒+代谢性碱中毒 D 、

的是: A、耐药菌 B、休眠菌 C、细胞内酸性环境中的菌 D、不断生长繁殖菌 E、偶然繁殖菌

29、男性,35 岁,发热、寒颤 3 天,体温 39 度,胸片示右上肺大片阴影,痰涂片见较多革兰氏阳性成对或短链状球菌,这时治疗首选?

A、头孢唑啉

B、丁胺卡那霉素

C、青霉素

D、氟哌酸

E、红霉素

30、肺气肿患者最通常的主诉是:

A、胸及胸骨后痛

B、发热和咯痰

C、咳嗽和咯痰

D、气短

E、体重减轻

31、下列疾病中哪一种肿瘤最能引起异源性 ACTH 综合症?

A、胰头癌

B、甲状腺癌

C、肺癌

D、胸腺癌

E、嗜铬细胞瘤

32、以下哪种情况有助于鉴别心源性哮喘与支气管哮喘 A、哮鸣音与双肺底小水泡音 B、端坐呼吸 C、咯粉红色泡沫痰 D、心脏扩大 E、高血压

33、女性,67 岁,反复咳嗽、咯痰 15 年,发热伴神志不清 1 天,体检:T38 度,神志昏迷,口唇发绀,双肺叩诊过清音,散在干湿罗音,心率 120 次每分,无杂音,神经系统检查无阳性体征,为明确昏迷的原因,首先应立即作以下哪项检查?

A、血白细胞计数

B、动脉血气分析

C、肝肾功能测定

D、脑脊液检查

E、X 线胸部照片

34、44 岁,男性,20 年前患过肺结核,吸烟 15 年,慢性咳嗽 5 年,半小时前剧咳后右侧胸痛,呼吸困难,检查患者呼吸窘迫,大汗,紫绀,气管左偏,右胸饱满,叩诊鼓音,呼吸音明显减弱,BP:50/20mmHg,脉搏微弱,紧急处理,首先要:

A、注射可拉明、洛贝林

B、吸氧

C、紧急胸透可拍胸片

D、静脉补液+多巴胺

E、胸穿排气或闭式引流

35、关于典型支气管哮喘的临床表现,下列哪项描述是正确的? A、发作性吸气性呼吸困难伴哮鸣音 B、发作性呼气性呼吸困难伴哮鸣音 C、哮喘发作越重,哮鸣音越强 D、肺内同时闻及哮鸣音及水泡音 E、进行性呼吸困难36、男性、50 岁,吸烟 30 年,近 3 年来咳嗽、咯痰,气喘,受凉后加重,每次持续 3 个月,有时痰中带血丝,查体:双肺呼吸音减弱,可闻及哮鸣音,最可能的诊断是:

A、支气管哮喘

B、心源性哮喘

C、支气管扩张症

D、慢性支气管炎喘息型

E、支气管肺癌

37、关于肺心病的病理病因下列哪项不正确? A、肺心病发生的先决条件是肺气肿 B、患病年龄多在 40 岁以上

C、肺心病病因中肺血管疾病少见

D、引起肺动脉高压的原因主要是肺血管阻力增加

E、肺心病少数可有左室受累

38、男性,59 岁,咳嗽、咯痰 20 年,加重 10 天,昏睡一天,明显紫绀,呼吸 10 次每分,表浅,双肺闻干湿罗音,心率 110 次每分,BP:16/11Kpa,血气分析 PaO2:32mmHg,PaCO2:80mmHg,PH:7.293,最重要的抢救措施是:A、可拉明静脉点滴 B、多巴胺静脉点滴 C、5%碳酸氢钠静滴 D、呼吸机辅助呼吸 E、β2 受体兴奋剂喷雾吸入

39、男,30 岁,自幼出现发作性呼吸困难,有时咳嗽,三天来喘息发作不止,伴咳嗽,咯小量粘液痰,查体:端坐呼吸,大汗,口唇紫绀,两肺呼吸音弱,广泛哮鸣音,诊断最可能的是:

A、支气管哮喘

B、喘息型支气管炎

C、哮喘持续状态

D、慢性支气管炎合并肺内感染

E、自发性气胸

40、患阻塞性肺气肿 12 余年,近日着凉后咳嗽、咯痰,气喘加剧,伴发热,上腹胀痛,纳差,肝大伴压痛、尿少,上肢浮肿明显,心电图偶见房性早搏,下列各项治疗中最重要的是:

A、抗心律失常

B、强心剂

C、保肝治疗

D、抗生素治疗

E、平喘、镇咳祛痰

41、诊断呼吸衰竭的确切依据是:

A、发绀、呼吸困难

B、肺通气功能明显减退

C、呼吸的节律,频率和幅度发生改变

D、粘膜充血、水肿和精神、神经症状

E、PaCO2>50mmHg, PaO2<60mmHg

42、阻塞性肺气肿的肺功能检查最有价值的是:

A、潮气量

B、肺活量

C、流量—容积曲线测定

D、残气量及残气量/肺总量百分比测定

E、最大通气量

43、肺结核空洞和肺脓肿空洞的最主要的鉴别诊断方法是:

A、病史

B、体征

C、胸部 X 线检查

D、痰细菌学检查

E、血白细胞检查

44、诊断早期肺心病的最主要依据是:

A、长期呼吸道病史

B、右心衰竭体征

C、肺气肿及肺部罗音

D、肺动脉高压及右心肥大征象

E、高碳酸血症

45、按累及肺小叶部位进行病理分型,下列哪项最符合阻塞性肺气肿? A、老年性肺气肿 B、代偿性肺气肿 C、局限性肺气肿 D、灶性肺气肿 E、小叶中央型肺气肿

46、对诊断阻塞性肺气肿最有意义的实验室检查是:

A、FEV1%<60%

B、最大通气量低于预计值 80%

C、残气容积占肺总量的百分比大于 40%

D、PaCO2<60mmHg)

E、PaCO2<50mmHg

47、在以下临床表现中除哪项外均为支气管哮喘的危重表现? A、呼吸困难加重,哮鸣音反而减少 B、意识模糊或昏迷 C、血压低,有奇脉 D、心电图示电轴左偏 E、呼酸及(或)代酸

48、肺心病呼吸衰竭患者,入院后急查血气:PaO2:40mmHg,PaCO2:75mmHg,立即鼻导管给氧 4L/分,2 小时后病人发绀减轻,但神志不清呈现昏睡状态,复查血气为 PaO2:80mmHg,PaCO2:95mmHg,此时首先应立即给予的处理是:

A、加大吸氧流量

B、降低吸氧流量

C、静脉注射地塞米松

D、静脉滴注头孢菌素

E、继续观察

49、慢性支气管炎发展演变至肺气肿时,最早期症状是:

A、咯血

B、心悸

C、心前区疼痛

D、咳嗽、咳痰加重

E、气短

50、阻塞性肺气肿出现下列哪种情况即可诊断为呼吸衰竭? A、发绀、呼吸困难 B、出现神经、精神症状 C、二氧化碳结合力升高 D、心慌、多汗、血压升高 E、动脉血氧分压低于 60mmHg

51、肺癌有不同的组织类型,其中恶性程度最高预后最差的是:

A、鳞癌

B、腺癌

C、小细胞未分化癌

D、大细胞未分化癌

E、细支气管肺泡癌

52、在动脉血气检查中,下列哪项指标最能反映肺泡通气的情况? A、SaO2

B、PaCO2

C、PaO2

D、HCO3-

E、T—CO2

53、下列哪项不符合肺心病的 X 线诊断依据

A、肺胸基础疾病表现

B、右下肺动脉干横径大于 15mm

C、右心室增大

D、肺动脉段明显突出或其高度大于 3mm

E、心衰发生后,增大的心脏不再缩小

54、肺炎球菌肺炎在病变消散后肺组织结构:

A、纤维组织增生

B、有小空洞残留

C、肺泡壁水肿

D、局部支气管扩张

E、肺泡壁无损坏

55、下列血气分析指标哪项不符合呼衰的诊断? A、PaO2<60mmHg,PaCO2>50mmHg

D、PaO2>60mmHg,PaCO2<35mmHg

E、PaO2<56.25mmHg,PaCO2>55mmHg

56、肺结核患者,咯血痰 2 天,突然大咯血,从鼻中涌出,因害怕,患者极力屏气随即出现烦燥不安,挣扎坐起,极度呼吸困难,颤面青紫,大汗淋漓,双眼上翻,立即应采取的抢救措施是: A、鼻导管给氧、注射呼吸兴奋剂 B、进行人工呼吸 C、胸穿抽气

D、清除血块或气管切开

E、输液、输血、注射止血剂

57、男、45 岁,咳嗽、咯痰 5 个月,活动后气促 2 年,为了解患者肺气肿程度应作下列哪项检查最有意义: A、心电图 B、血气分析 C、胸部 X 线片

D、血常规

E、肺功能测定

58、对诊断阻塞性肺气肿最有意义的检查是:

A、X 线检查两肺野的透亮度增加

B、心电图检查呈现低电压

C、肺放射核素扫描

D、残气容积超肺总量 40%

E、痰细菌学及细胞学检查

59、男,50 岁,10 天前发冷发热,T39.5 度,右胸痛,咳嗽、咯脓痰 250ml/日,臭味,有结核病密切接触史,为进一步明确诊断,应首先做哪项检查?A、肺 CT 检查 B、肺放射核素扫描 C、胸部 X 线正侧位片 D、纤维支气管镜检查 E、痰细菌学及细胞学检查

60、成人结核菌素试验有:1:2000 阳性,此结果最可能提示: A、现在正患活动性肺结核 B、可排除结核病 C、非典型分枝杆菌感染 D、结核病已经治愈 E、曾有结核菌感染

61、男性,60 岁,慢支肺气肿 20 年,近日感冒,咳嗽、气促加剧,发热,失眠,昨晚服冬眠灵 2 片,今晨神志朦胧,皮肤潮红,眼球结膜充血水肿,巴彬斯基征阳性,最应立即做何处理? A、抽胃内容物送检查 B、血液气体分析 C、血常规检查 D、脑电图检查

E、胸部摄片

62、慢性肺源性心脏病最多见的病因是: A、慢性支气管炎合并阻塞性肺气肿

B、多发性肺小动脉栓塞

C、严重胸廓及脊柱畸形

D、重症肺结核

E、支气管哮喘

63、女性,63 岁,患肺结核 10 年,近一周来痰中带血,10 分钟前大咯血约 600 毫升,突然血中断,颜面青紫,牙关紧闭,此时应立即采取的抢救措施是:A、进行人工呼吸 B、使用呼吸兴奋剂 C、行导管吸氧 D、输血补液 E、解除呼吸道梗阻

64、在肺脓肿时所有常见的厌氧菌都对青霉素敏感除去?

A、脆弱类杆菌

B、梭形菌

C、胨球菌

D、胨链球菌

E、微需氧链球菌

65、弥漫性肺间质纤维化的最主要症状是:

A、逐渐加重的呼吸困难

B、发作性呼吸困难

C、呼吸困难伴哮鸣音

D、长期咳嗽、咯痰

E、反复痰中带血

66、某患,30 岁,患发作性气喘病已十年,近一月来犯病,听诊肺有弥漫性哮鸣音,X 线胸片示肺透亮度增高,膈肌低平,为判断患者是否并发肺气肿,下列检查中最有意义的是:

A、动脉血气分析

B、FEV1%及其改善率测定

C、残气容积与肺总量

D、肺 CT 扫描

E、肺下界移动度

67、早期二氧化碳蓄积,血压、脉搏的变化应该是:

A、血压下降,脉搏减慢

B、血压上升,脉搏增快

C、血压不变,脉搏减慢

D、血压上升,脉搏减慢

E、血压下降、脉搏增快

68、干酪性肺炎属于下列疾病?

A、杆菌性肺炎

B、肌炎球菌肺炎

C、浸润型肺结核

D、军团菌肺炎

E、支原体肺炎

69、72 岁患者,8 年前开始经常咳嗽、咯痰,近一年来症状加重,今晨排便时突然出现气急,呼吸困难,查体:呼吸 30 次每分,口唇紫绀,气管轻度右偏,桶状胸,左肺叩呈鼓音,呼吸音消失,首先应急诊进行下列哪项检查?

A、纤维支气管镜检查

B、肺功能测定

C、痰培养+药敏

D、X 线胸部检查

E、心电图

70、下列哪组肺功能测定结果最有助于肺气肿的诊断? A、残气量>肺总量40%,最大通气量<预计值 80% FEV1/FVC>60% B、残气量>肺总量 40%,最大通气量>预计值 80% FEV1/FVC<60% C、残气量>肺总量 40%,最大通气量<预计值 80% FEV1/FVC<60% D、残气量>肺总量 40%,最大通气量<预计值80% FEV1/FVC>65% E、残气量>肺总量 40%,最大通气量<预计值 80%

FEV1/FVC<60%

71、急性起病,胸痛伴高热,咳嗽咯血和大量脓臭痰的患者,最可能的诊断的是: A、金黄色葡萄球菌肺炎 B、支气管扩张 C、肺癌 D、肺栓塞 E、肺脓肿

72、下列哪项改变可避免哮喘发作?

A、环磷酸腺苷水平升高

B、环磷酸鸟苷水平升高

C、白三烯水平升高

D、M—胆碱能受体兴奋性升高

E、鸟苷环化酶活性升高

73、男性,60 岁,慢支肺气肿多年,血脂高,剧烈咳嗽后突感左胸剧痛,气急憋气,不能平卧,大汗淋漓,左侧呼吸音比右侧更弱,心音遥远,哪项检查应首选?

A、胸部 X 线检查

B、血液气体分析

C、心电图检查

D、血常规检查

E、心脏超声检查

74、抗原脱敏治疗支气管哮喘的机理是:

A、扩张支气管(缓解支气管平滑肌痉挛)

B、抑制迷走神经

C、抑制 M 胆碱能受体

D、抑制肥大细胞释放生物活性物质

E、增加封闭抗体

75、X 线胸片显示肺透过度增高,肺纹理增粗,横膈降低,肋间隙增宽,肺功能:FEV1 和最大通气量均降低,RV/TLC%增加,以上检查结果最常见于下列哪种疾病?

A、肺间质纤维化

B、双侧自发性气胸

C、慢性支气管炎合并阻塞性肺气肿

D、支气管哮喘缓解期

E、先天性肺囊肿

76、男性,40 岁,支气管哮喘 20 多年,6 天来加重,经治疗后症状明显好转,2 小时前突然右胸剧痛,呼吸困难进行性加重,体检:紫绀,大汗,烦躁不安,气管左移,右肺叩为鼓音,BP12/8Kpa(90/60mmHg),心率 120 次每分,最应立即进行的检查是:

A、心电图

B、胸部 X 线

C、B 超

D、痰培养+药敏试验

E、化验白细胞+分类

77、慢性阻塞性肺气肿病人发生缺氧的主要机理是:

A、肺组织弹力减退

B、通气与血流比例失调

C、肺动脉分流异常

D、弥散功能

78、结核病传染的主要途径是:

A、饮用未经消毒的病牛奶

B、吸入病人咳嗽、喷嚏时排出的带菌飞沫

C、皮肤外伤

D、泌尿生殖道外伤

E、吸入带菌的干燥痰液碎末

79、慢性肺心病形成肺动脉高压的最主要的因素是:

A、血容量增多

B、血液粘滞度增高,增加肺血管阻力

C、血管床横面积减

少 D、肺小动脉痉挛 E、肺小血管形成

80、慢性阻塞性肺气肿的遗传因素与下列哪项关系最密切 A、弹性蛋白酶分泌增加 B、cAMP/cGMP 降低 C、丙种球蛋白缺乏

D、α抗胰蛋白酶减少

E、前列腺素 E 增加(PGE 增加)

81、一慢性支气管炎,肺气肿患者,病情急剧恶化,呼吸困难加重,给予高浓度 60%吸氧治疗后,PaCO2 由 50mmHg 上升至 80mmHg,产生此种变化的最可能的原因为:

A、补液过多,气道阻力增高

B、心功能不全,心排出量降低

C、支气管痉挛未缓解

D、血中 2.3DPG 水平降低

E、呼吸中枢兴奋性降低

82、男,20 岁,咳嗽半年,有血丝痰,易汗,乏力,发热 1 个月,体温 37.5—38 度,胸片示右上肺片状阴影,密度不均可疑小透亮区,血 WBC:11*

10^9/L,诊断首先考虑:

A、肺炎

B、肺癌

C、肺脓肿

D、支气管扩张症

E、肺结核

83、以下体液因素中除哪项外均可导致肺血管阻力增加而促使肺动脉高压的形成? A、前列腺素 B、前列腺素 E C、白三烯 D、血栓素 A2 E、花生四烯酸

84、45 岁男性,1 年前不明原因活动后气促,逐渐加重,体检:轻度发绀,肺底部少量细湿罗音,有栓状指,胸片示双肺中下野肺纹理增多呈网状,并有结节状阴影,血气分析:PaO2:60mmHg,PaCO2:35mmHg,以下哪项诊断可能性最大?

A、慢性支气管炎

B、支气管哮喘

C、支气管扩张

D、弥漫性肺间质纤维化

E、支气管肺癌

85、男,20 岁,低热咽痛,咳嗽半月入院,咳嗽甚剧,为刺激性干咳,体检:T37.8 度,咽充血,心肺无阳性体征,化验:WBC:8*10^9/L,中性 70%,X 线胸片示右下肺间质性炎变,间有小片状阴影,以下哪项检查对明确诊断意义较大?

A、痰细菌培养

B、咽拭子细菌培养

C、痰查抗酸杆菌

D、结核菌素试验

E、冷凝集试验

86、男,56 岁,咯痰带血,继之左侧胸伴呼吸困难两个月,体温正常,气管居中,左侧大量胸腔积液体征,血沉增快,最可能的诊断是:

A、肺梗塞

B、癌性胸膜炎

C、结核性渗出性胸膜炎

D、漏出性胸腔积液

E、肺脓肿合并脓胸

87、女,24 岁,右侧胸腔大量积液,因呼吸困难不能平卧而行胸穿排液,操作中患者感头晕,心慌,面色苍白,冷汗,脉搏细数,血压降低,应立即采取的应急措施是:

A、静滴多巴胺等血管活性药

B、快速输注低分子右旋糖酐

C、平卧、皮下注射肾上腺素

D、缓慢静注西地兰

E、吸氧并静注氨茶碱

88、男,30 岁,有机磷农药中毒后,持续高热,WBC:15*10^9/L,10 余天后咯脓血痰,有臭味,经大量青霉素+灭滴灵治疗,明显好转,但随后体温再次升高,痰量反见减少,分析出现这种情况的最可能的原因是:

A、细菌产生耐药,病情出现反复

B、感染加重,可能出现并发症

C、支气管阻塞痰液引流不畅

D、营养支持疗法不力

E、出现了菌群失调

89、女,19 岁,乏力,低热三个月,咳嗽、咯痰带血一周,X 线胸片示右肺上叶片状阴影,大咯血一小时, BP:90/60mmHg,P:96 次每分,采取下列哪种治疗措施最恰当?

A、输血、补液

B、左侧卧位+青霉素静滴

C、吸氧+脑垂体后叶素静滴

D、止血芳酸静滴

E、右侧卧位+脑垂体后叶素静滴

90、肺炎球菌肺炎并发左心衰竭下列哪种药物不宜使用? A、氨茶碱 B、糖皮质激素 C、吗啡 D、心得安 E、西地兰

91、关于肺心病的体征,下列哪项是不正确的? A、剑突下收缩期心脏搏动

B、肺动脉瓣听诊区第二心音亢进

C、三尖瓣听诊区收缩期杂音

D、心尖部抬举肃心脏搏动

E、两肺呼气音延长

92、女,24 岁,呼吸困难,X 线胸片示胸腔积液,穿刺抽液时,突然面色苍白,出冷汗,血压下降,最好应给哪项处置?

A、氨茶碱+葡萄糖静脉注射

B、西地兰 0.4mg 缓慢静脉注射

C、平卧 0.1%肾上腺素 0.5mg

D、快速输注低分子右旋糖酐

E、静脉快速滴注升压药物93、患者,25 岁,女性,咽痛,咳嗽,乏力,四肢肌肉疼痛,中等发热,双肺呼吸音稍粗,未闻罗音,白细胞 9.6*10^9/L,中性 86%,胸片示:左下肺部斑片状浸润阴影,血清冷凝集试验:1:64 阳性,最好应选择的治疗药物是:

A、抗结核药

B、青霉素

C、头孢菌素

D、氨基甙类抗生素

E、红霉素

94、老年患者,咳嗽,右胸痛,气促,咯两口铁锈痰,检查:BP64/40mmHg,脉搏 120 次每分,紫绀,神志恍惚,肢冷,右下肺少量湿性罗音,WBC7.8*10^9/L,中性 85%,首先应采取的紧急措施是: A、高流量吸氧 B、补充血容量 C、静滴广谱抗生素 D、静滴血管活性药

E、静滴糖皮质激素

95、患者,32 岁,一周前面部疖肿挤压排脓,两天前高热寒战,头痛,伴咳嗽,少量脓痰,两肺少量湿罗音,WBC2.5*10^9/L,中性 90%,胸片示:两肺有散在类圆形密度增高阴影,部分病灶可见透光区,治疗应首先的药物的是:A、青霉素 B、氨基糖甙类抗生素 C、新青霉素 II 或万古霉素 D、红霉素 E、头孢菌素

96、关于结核感染,免疫与变态反应,下述哪项是错误的? A、免疫力与变态反应非同时产生,不同时存在 B、结核菌素试验只能测出变态反应,不能完全代表免疫力 C、结核菌类脂质结核蛋白与产生变态反应有关 D、菌体多肽及多糖复合物与产生免疫力有关

E、结核感染后 4—8 周,产生变态反应

97、男性,25 岁,哮喘反复发作 13 年,严重发作已持续 48 小时,体检:呼吸困难,紫绀,烦燥,心率

124 次每分,双肺呼吸音低有哮鸣音,下列哪项措施是错误的?

A、静滴 5%葡萄糖盐水

B、氨茶碱 0.25 加入输液中

C、氢化可的松 200mg 静滴

D、头孢菌素静脉滴入

E、肌注苯巴比妥

98、男性,76 岁,体重 45 公斤,身长 170cm ,突然出现右胸痛,咳嗽,咯红棕色胶冻样痰,伴发热,体温 38.7 度,查体右中下肺叩诊浊音,可闻管性呼吸音及少许湿罗音,X 线胸片示右中下肺野大片密度增高影,其中可见小透亮区,应首选以下哪种疾病?

A、肺炎球菌肺炎

B、金黄色葡萄球菌肺炎

C、复发性肺脓肿

D、肺炎杆菌肺炎

E、肺结核合并胸膜炎

99、呼吸衰竭时缺氧与二氧化碳潴留的最主要发病原理是:

A、通气/血流比例失调

B、弥散性功能障碍

C、肺泡通气量不足

D、氧耗量增加

E、肺内动静脉分流增加

100、下列哪项不符合弥漫性间质性肺疾病的表现?

A、干咳

B、气急

C、杵状指

D、X 线胸片呈弥漫性炎变

E、肺功能呈阻塞性通气功能障碍

101、支气管哮喘发病的最主要病理基础是:A、气道的非特异性炎症 B、副交感神经兴奋 C、细菌感染 D、支气管痉挛E、支气管分泌物过多

102、克雷白杆菌肺炎有别于其它肺炎的最主要的病理学特征是: A、有明显的发病阶段 B、小叶性分布 C、常引起组织坏死形成空洞 D、切面可挑起粘稠的丝状渗出物 E、纤维组织增生

103、男,38 岁,拔牙术后 10 天,寒战高热一周,咳嗽,咯脓性痰,300ml/日,有考虑引起本病最可能的致病菌是:

A、金黄色葡萄球菌

B、大肠杆菌

C、肺炎球菌

D、厌氧菌

E、化脓性链球菌

104、男,患者,58 岁,患慢性支气管炎,阻塞性肺气肿,近日犯病,2 小时前突然左胸部刺痛,呼吸困难加重,为明确诊断最适当的检查应选:

A、心电图

B、肺部 B 超

C、胸部 X 线检查

D、肺 CT

E、血气分析105、男性患者 55 岁,患慢性支气管炎,阻塞性肺气肿,近日犯病,2 小时前突然左胸部刺痛,呼吸困难加重,为明确诊断,最适当的检查应选?

A、心电图

B、肺部 B 超

C、胸部 X 线检查

D、肺部 CT

E、血气分析

106、某患者 55 岁,慢性咳嗽 20 多年,近五年来气促逐渐加重,体检:桶状胸,肺泡呼吸(缺)

107 对肺组织纤维化程度的监测最简单最实用的方法是:

108、男,48 岁,支气管哮喘病史 20 多年,近 10 年来加重,无明显缓解期并出现喘憋,活动后症状更为明显,为评定肺功能,应选择的最主要的检查项目是:

A、弥散功能测定

B、肺活量测定

C、动脉血气分析

D、FEV1

E、残气容积测定

109、女,60 岁,咳喘 25 年,近三年稍动即气促,三天前发热,咯黄痰,入院时动脉血气分析 PH7.30, PaO2:55mmHg,PaCO2:75mmHg,BE:—10mmol/L,此例酸碱平衡紊乱最佳诊断为:

A、呼吸性酸中毒代偿

B、呼吸性酸中毒失代偿

C、代谢性酸中毒

D、呼吸性酸中毒合并代谢性碱中毒

E、呼吸性酸中毒合并代谢性酸中毒

110、半年前劳动时损伤左小腿皮肤,未予处理,入院前 3 天,畏寒,发热(38.5),气急咳嗽,痰不多,黄色混血丝,体检:巩膜黄染,两肺散在干罗音,心率 120 次每分,第一心音低钝,血 WBC:22.8*10^9/L,中性 88%,有中毒颗粒,肝功:GPT:45u,X 线胸片示胸肺散在片状,结节状阴影,内有小液平,细胞壁酸抗体阳性,本例诊断下列哪项是不符合的?

A、败血症

B、传染性肝炎

C、金黄色葡萄球菌肺炎

D、中毒性心肌炎

E、中毒性肝炎

111、患者,16 岁,今日突发呼吸困难,发作过敏,干咳,查体:血压正常,端坐呼吸,肺有哮鸣音,心率 110 次每分,律齐无杂音,首先应考虑的是:A、上呼吸道感染 B、慢性支气管炎喘息型 C、支气管哮喘 D、心源性哮喘 E、过敏性肺炎

112、24 岁,一周来低热,乏力,胸痛,气促,体检:右胸叩诊实音,呼吸音消失,胸水常规草黄色,比重 1.030,WBC:0.8*10^9/L,淋巴占 80%,蛋白 34g/L,LDH:300u/dl,胸水 LDH/血清 LDH 比值为:0.75,最可能的诊断是:

A、右侧肺炎伴胸水

B、肺癌伴胸膜转移

C、肝硬化伴胸水

D、慢性肾功能不全伴胸水

E、结核性胸膜炎

113、女,25 岁,发热干咳,左胸痛 10 余日,逐渐出现呼吸困难,X 线胸片见中下肺野均匀致密阴影,上缘呈外高内低反抛物线形,为确定诊断,下列哪项检查是合适的?

A、胸腔试穿

B、胸腔 B 超

C、痰培养查致病菌

D、病灶断层拍片

E、纤维支气管镜检

114、肺脓肿发病最多见的病因是:

A、病原菌经口、鼻、咽腔吸入下呼吸道

B、骨髓炎病原体侵入血流发生脓毒血症

C、肾周围脓肿感染病变扩散蔓延而成

D、空洞性肺结核继发感染而发病

E、肺癌发生中心性坏死

115、女,60 岁,二日来高热寒战,咳嗽,咯痰带血丝,胸痛,查体:体温:37 度,面色灰白,皮肤湿冷,口唇紫绀,左上肺叩诊浊音,呼吸音弱,少许湿罗音,心率 128 次每分,BP(70/50mmHg),应立即采取的措施是:

A、拍胸部 X 线片

B、查白细胞+分类

C、痰培养+药敏试验

D、输液+血管活性药物

E、心电图检查

116、男,20 岁,3 天前受凉后突发畏寒发热,次日感右季肋部疼痛,深呼吸时加剧,轻咳,痰少。体检: T39 度,P120 次每分,R30 次每分,右下肺呼吸音减低,语颤增强,无罗音,右上腹肌张力增高,有压痛,无反跳痛,肝脾未扪及,化验:WBC:20*10^9/L,中性 90%,为明确诊断应首先作以下哪项检查? A、血培养 B、胸部 X 线检查 C、B 超探查有无胸液 D、B 超探查肝细胞

E、腹部 X 线照片

117、下列哪项属于呼吸衰竭二氧化碳潴留引起的临床表现? A、心率增快,血压升高 B、头痛、意识障碍 C、结膜充血,水肿 D、面色苍白,皮肤血管收缩 E、多汗

118、男,近十年来活动性气短逐渐加重,现不能和正常人一样走路,X 线检查:肺野清晰,肺功:FEV1/FVC 为 65%,残气率为 45%,观察病人呼气时鼓腮吹气,病人采取这种呼吸方式的最主要的道理是: A、自然习惯 B、增加肺容量以改善气体交换 C、增加呼气阻力,防止气道闭陷 D、因鼻腔有病,经鼻呼气不畅所致 E、用力加强呼吸的结果

119、下列各项中哪一项不符合红喘型慢性阻塞性肺气肿? A、多发于老年瘦弱患者 B、严重慢性支气管炎伴小中央型肺气肿 C、呼吸困难明显而无紫绀

B、动脉血氧分压轻度降低

E、X 线胸片显示主影不扩大

120、男,60 岁,因发热,咯脓痰 3 个月,抗菌治疗稍有缓解,胸片示左下肺空洞病变,近 2 天突然大咯血 1500ml 左右,以下哪项处理最合适

A、即作纤支镜检查

B、肺 CT 扫描

C、请胸外会诊

D、加照侧位胸片

E、痰中反复找瘤细胞

121、原发性肺结核,其自然演变过程中最常见的是:

A、自然吸收或钙化

B、发展为肺门淋巴结结核

C、病灶溶解,坏死形成空洞

D、血行播散

E、发展为胸膜炎

122、过敏性支气管哮喘属于哪一种?

A、单纯的 I 型变态反应

B、I 型和 III 型反应同时存在

C、II 和 IV 反应同时存在

D、单纯 III 型反应

E、单纯 IV 型反应

123、对诊断弥漫性间质性肺纤维化最有意义的临床表现是: A、进行性呼吸困难 B、发热、无力、消瘦 C、发绀,杵状指是 D、两肺听到 Veiero 罗音E、咳嗽,咯痰带血

124、患者,16 岁,发热 3 周,体温 38 至 39 度,伴咳嗽,表情淡漠,全身无疹,出血点,但肝脾肿大,血 WBC:23*10^9/L,中性 78%,胸片:双肺

有细小点状阴影,最可能的诊断考虑为:

A、伤寒

B、败血症

C、急性粟粒型肺结核

D、急性白血病

E、疟疾125、弥漫性肺间质疾病的最主要的临床表现为: A、进行性呼吸困难 B、反复咳嗽、咯痰,有时咯血 C、干咳 D、胸痛,肺底湿罗音 E、不同程度发热乏力

126、慢性阻塞性肺病(COPD)最确切的定义是指: A、单纯性慢性支气管炎合并阻塞性肺气肿 B、喘息性慢支合并阻塞性肺气肿 C、慢性阻塞性支气管炎合并阻塞性肺气肿 D、慢支合并阻塞性肺气肿及肺心病 E、支气管哮喘合并阻塞性肺气肿

127、下列关于肺心病心电图的诊断标准中,哪一项是错误的? A、心电图,额面平均电轴大于+90 度 B、窦性心动过快 C、高而宽大的 P 波 D、低电压 RV1+SV5 大于 1.05mb E、重度顺钟向转位

128、男,20 岁,近日来咳嗽,少量痰液,低热,前夜突起气促,张口呼吸,大汗淋漓,面色苍白,脉速

120 次每分,BP:(90/60mmHg),双肺布满哮鸣音,心无杂音,多次静注氨茶碱无改善,至今已一天多,最可能的诊断是:

A、急性左心衰竭

B、喘息型慢性支气管炎

C、支气管肺癌

D、支气管哮喘持续状态

E、过敏性肺炎

129、慢性肺心病人,血 PH7.51,PaCO2:60mmHg,HCO3:38mmol/L,Na:134mmol/L,Cl:76mmol/L,K:2.4mmol/L,下列哪项诊断最合适?

A、呼吸性酸中毒代偿

B、呼吸性酸中毒失代偿

C、呼吸性酸中毒+代谢性碱中毒

D、呼吸性酸中毒+代谢性酸中毒

E、代谢性碱中毒

130、临床上为鉴别渗出性或漏出性胸腔积液,下述哪项是主要的? A、胸腔积液的颜色和比重 B、胸腔积液静置后是否凝固 C、胸腔积液 LDH 测定与血清 LDH 测定 D、胸腔积液粘蛋白定性 E、胸腔积液细胞总分数

131、男,60 岁,咳嗽 30 年,近三年行走时气促,半月前因受凉、咳嗽、呼吸困难加剧入院治疗,自觉明显好转,但仍有活动后气促,血气分析:PH:7.41,PaO2:62mmHg,SaO291%,此例的给氧原则最好应为: A、高浓度,短期给氧 B、高浓度,高流量持续给氧 C、低浓度,间断给氧 D、低浓度,低流量,持续给氧 E、不需给氧

132、男,40 岁,发热咳嗽,咯脓痰 2 周,痰量每日约为 200ml,有时痰中带血,吸烟 20 年,WBC:32*

10^9/L,中性 90%,为进一步确诊,下列哪项检查最有价值? A、痰培养+药敏 B、X 线胸片 C、痰找结核菌 D、纤维支气管镜检 E、放射性核素肺扫描

133、男,20 岁,咳嗽、咽痛,肌痛,X 线胸片显示间质性肺炎改变,肌注青、链霉素 2 周无效,为明确诊断,最好选用以下哪项检查?

A、血常规

B、血沉

C、OT 试验

D、冷凝集试验

E、痰细菌培养134、特发性肺纤维化,有杵状指者占:

A、60%

B、30%

C、50%

D、10%

E、90%

135、女,30 岁,反复咳嗽 3 年,每年秋季发作,干咳,无痰,夜间咳嗽加重,伴有胸闷,气短,抗生素

和止咳药治疗无效,冬季过后咳嗽自行缓解,体检:双肺干湿罗音,X 线胸片检查:心肺正常,为明确诊断应首先作以下哪项检查?

A、胸部 CT

B、肺通气功能与弥散功能测定

C、用支气管扩张剂前后测定通气功能

D、纤维支气管镜检并取活检

E、超声心动图检查

136、关于特发性肺间质纤维化的临床表现,下列哪项是错误的? A、微热、咳嗽、呼吸困难

137、对于结核性胸膜炎,下列检查中哪项最有助于诊断?

A、胸水细胞数

B、病原体检查

C、胸膜活检

D、胸水查找胆固醇结晶

E、胸水 ADA 测定

138、对预防肺结核具有最根本性的影响措施是: A、卡介苗预防接种 B、口服异烟肼预防 C、隔离排菌病人 D、卫生宣传、禁止随地吐痰 E、彻底合理化疗

139、患者,68 岁,气促,神志恍惚,面色潮红,球结膜充血水肿,心率 120 次每分,律不齐,肝肋下 3cm,双下肢浮肿,尿蛋白+,为判断病情首要检查应选择:

A、肝功能

B、心电图

C、血尿素氮

D、动脉血气分析

E、电解质140、在下述说明呼吸道感染促使慢性肺心病心肺功能障碍加重和病情恶化的机理中最重要的是: A、感染使机体氧耗量增加 B、感染中毒对心肌的损害 C、肺通气血流比例失调加重 D、弥散障碍和静动脉分流增加 E、肺泡通气不足更加减少

B 型题:

A、患者需住院隔离、化学药物治疗到查痰涂片阴转

B、化学药物治疗完毕后,需每年拍照胸片一次,至少十年

C、应用至少两种以往未用或少用的抗结核药物

D、应用三种抗结核药物短程化疗 9 个月

E、二种抗结核药物治疗 6 个月141、治疗痰中结核菌阳性的活动性肺结核:

142、治疗痰中结核菌阳性的复发的慢性纤维空洞型肺结核: A、PaO2:90mmHg,PaCO2:32mmHg,PH7.25

B、PaO2:50mmHg,PaCO2:30mmHg,PH7.45

C、PaO2:85mmHg,PaCO2:38mmHg,PH7.40

D、PaO2:100mmHg,PaCO2:28mmHg,PH7.46

E、PaO2:50mmHg,PaCO2:68mmHg,PH7.30

143、女学生,10 岁,诉在期末考试前呼吸困难,口周麻木,感觉异常:

144、男,68 岁,肺心病,呼衰急性发作:

A、夜间发作性呼吸困难

B、两肺散在湿性罗音和哮鸣音

C、受体阻滞剂治疗有效

D、异丙肾上腺素喷雾吸入症状迅速改善

E、咳喘呼吸困难进行加重,痰中带血,支气管扩张剂治疗无效。

145、慢性喘息型支气管炎:

146、支气管哮喘:

A、链霉素

B、吡嗪酰胺

C、乙胺丁醇

D、利福平

E、异烟肼

147、可引起听力障碍的是:

148、可引起视力障碍的是:

149、大剂量可引起末梢神经炎的是: A、大叶肺炎 PaO2:

60mmHg ,PaCO2:30mmHg B、间质性肺炎 PaO2: 52mmHg ,PaCO2:30mmHg C、COPD PaO2: 56mmHg ,PaCO2:56mmHg D、肺结节病 PaO2:

66.6mmHg ,PaCO2:34mmHg E、胸廓畸形 PaO2: 64.5mmHg ,PaCO2:47.8mmHg 150、上述哪项符合 II 型呼衰的血气标准?

151、上述哪项符合 I 型呼衰的血气标准? A、PaO2:45mmHg,PaCO2:70mmHg,PH7.26

B、PaO2:85mmHg,PaCO2:27mmHg,PH7.51

C、PaO2:90mmHg,PaCO2:40mmHg,PH7.38

D、PaO2:70mmHg,PaCO2:60mmHg,PH7.44

E、PaO2:80mmHg,PaCO2:30mmHg,PH7.426

152、呼吸衰竭患者,应用机械通气 2 小时后,测定上述动脉血气结果,哪项要增加通气量?

153、上述结果,哪项吸氧浓度和通气量都不需要改变而继续原治疗方案? A、碳酸氢钠 B、盐酸 C、氯化钾 D、改善通气功能

154、肺心病呼吸衰竭患者,PH7.25,PaCO2:70mmHg,HCO3:27mmol/L,为纠正其酸碱失衡应首选以上哪项治疗措施?

155、肺心病呼吸衰竭患者,PH7.10,PaCO2:60mmHg,HCO3:18mmol/L,为纠正其酸碱失衡,应首选以上哪项治疗措施?

A、2HSE/10HE

B、2HRE/7HR

C、2HRE/7HE

D、2HREZ/7HR

E、2HSEZ/7HZE 注:H:异烟肼 S:链霉素 R:利福平 E:乙胺丁醇 Z:吡嗪酰胺

156、肺结核痰菌阳性的初治患者,最适当的化疗方案是:

157、复发的肺结核患者(已用过 INH、SM、PAS)最适当的化疗方案是? A、慢性阻塞性肺病(气肿型) B、慢性阻塞性肺病(支气管炎型) C、慢性支气管炎(单纯型) D、慢性支气管炎(喘息型) E、内源性支气管炎并哮喘

158、男、62 岁,反复咳嗽 15 年,咯少量白色泡沫痰,近 5 年气促明显,体检:气促,但不发绀,桶状胸,双肺呼吸音降低,肺底少许湿罗音,X 线胸片示重度肺气肿,心影不大,血气分析:PaO2:75mmHg,PaCO2:

40mmHg,该病例应判定为:

159、女,50 岁,反复咳嗽 10 年,受凉后咯多量粘性脓痰,近三年活动后感轻度气促,体检:轻度气促,紫绀明显,面部浮肿,有广泛干湿罗音,双下肢水肿,X 线胸片示肺纹理增多,紊乱,心影扩大,血气分析:PaO2:55mmHg,PaCO2:60mmHg,该例应判定为:

A、糖皮质激素

B、色甘酸二钠

C、异丙阿托品

D、氨茶碱

E、舒喘灵

160、能舒张支气管平滑肌及冠状动脉的药物是:

161、主要兴奋支气管平滑肌β2 受体的药物是:

A、肺脓肿

B、肺癌

C、肺结核

D、支气管扩张症

162、男,40 岁,近 2 个月来低热,咳嗽,消瘦,X 线示右上肺炎症浸润及空洞性病变,最可能的诊断是:

163、女,28 岁,CO 中毒后发热、咳嗽、一周后咯脓痰,X 线胸片示右下叶背段大片浓密浸润阴影,并有透亮度,最可能的诊断是:

A、改善肺泡通气措施不力

B、使用利尿剂与糖皮质激素不当

C、支气管—肺感染加重

D、应用呼吸兴奋剂与脑细胞活化剂所致

E、液体补充不足

164、慢性肺心病心力衰竭经治疗后,虽然消失,PaO2 升高,但意识状态不见改善,PaCO2 居高不降,其原因考虑是:

165、慢性肺心病急性加重期,经一周积极治疗,尿量增多,水肿消退,但出现烦燥,手抖,反射亢进,血气 PH:7.43,PaCO2:8.67Kpa,病情出现此种变化的常见原因是:

A、抗生素

B、利尿剂

C、强心剂

D、血管扩张剂

E、持续性低流量吸氧

166、女,62 岁,慢性咳嗽,喘三十年,一个月加重,双下肢浮肿,肝大,肋下 4cm,腹水征阳性,治疗应首选:

167、男,65 岁,慢性咳、痰、喘二十年,一周加重,发绀,两肺湿性罗音,血气分析 PH:7.36,PaCO2:60mmHg, PaO2:(62mmHg),治疗首选:

A、肺炎球菌肺炎

B、金黄色葡萄球菌肺炎

C、支原体肺炎

D、绿脓杆菌肺炎

E、病毒性肺炎

168、常是住院期间获得的肺部感染的是:

169、容易并发脓气胸的是:

A、外源性支气管哮喘

B、支气管扩张

C、喘息型慢性支气管炎

D、浸润型肺结核

E、支气管肺癌

170、两肺散在湿性罗音,伴哮鸣音及呼气延长,符合?

171、普遍性哮鸣音,呼气延长,符合

A、肾上腺皮质激素

B、舒喘灵

C、氨茶碱

D、东莨菪碱

E、色甘酸二钠172、主要兴奋β2 受体的是:

173、能抑制磷酸二酯酶,增加心肌收缩力,静注过快可引起心律失常,甚至死亡的是:

174、能抑制α受体,稳定细胞溶酶体膜,可提高腺苷环化酶和抑制磷酸二酯酶的作用的是: A、异烟肼 B、利福平 C、链霉素 D、吡嗪酰胺 E、乙胺丁醇

175、能杀灭吞噬细胞内酸性环境的结核菌,可有高尿酸血症,关节痛,胃肠道反应和肝损害的是:

176、能抑制结核菌脱氧核糖核酸的合成,阻碍细胞壁合成,能杀灭细胞内外代谢活跃或近乎静止的结核菌的是:

177、抑制结核菌体 RNA 多聚酶,对细胞内外代谢旺盛或偶尔繁殖的结核菌均有杀灭作用的是: A、氨茶碱 B、肾上腺素 C、糖皮质激素 D、速尿 E、吸氧

178、支气管哮喘持续状态不宜用?

179、支气管哮喘合并冠心病不宜用?

A、心源性哮喘

B、喘息型慢性支气管炎

C、过敏性肺炎

D、支气管哮喘

E、支气管肺癌

180、反复喘息 7 年,春秋季节好发,发病 1 周,期间喘息加重,两肺散在哮鸣音,应诊断为?

181、高血压病 20 余年,近 2 年来,常夜间阵发性喘息,咳嗽,胸闷,端坐呼吸,两肺底可闻湿罗音,及少量哮鸣音,应诊断为?

A、青霉素

B、氨基甙类抗生素

C、红霉素

D、磺胺类药物

E、利福霉素182、克雷白杆菌肺炎抗生素治疗,首选药物是?

183、肺炎支原体肺炎抗生素治疗,首选药物是? A、呼吸性酸中毒 B、呼酸合并代酸 C、呼吸性碱中毒

D、呼吸性酸中毒合并代谢性碱中毒

E、呼吸性碱中毒合并代谢性酸中毒184、肺心病患者感冒后呼吸困难,尿少,下肢浮肿,经治疗后浮肿消退,但患者出现兴奋躁动,手足抽搐,血清钾及氯均低,酸碱失衡的变化是:

185、慢性肺心病急性加重期并发感染性休克,尿少,酸碱失衡的变化是: A、支气管哮喘 B、支气管扩张 C、胸廓畸形 D、慢支并发肺气肿 E、肺血管疾病

186、引起慢性肺心病最常见的病因是:

187、在慢性肺心病的病因中甚少见的是:

A、失代偿呼酸

B、呼酸合并代碱

C、呼酸合并代酸

D、呼碱

E、代碱188、肺心病急性加重期给糖皮质激素和利尿剂后常见?

189、肺心病急性加重期机械通气后可见?

190、肺心病急性加重期感染合并休克常见? A、发热、咳嗽、咯粉红色乳状痰B、大量脓性痰,有臭味伴发热 C、端坐呼吸,咳嗽,咯粉红色泡沫痰 D、发热、咳嗽、伴胶状砖红色痰 E、发热、咳嗽、胸痛伴铁锈色痰,

191、克雷白杆菌肺炎的表现为?

192、金黄色葡萄球菌肺炎的表现为?血气分析的结果为:

A、PH:7.30 PaCO2:75mmHg BE:+2.5mmol/L

B、PH:7.22

PaCO2:60mmHg BE:-8mmol/L C、PH:7.50 PaCO2:65mmHg

BE:+15mmol/L D、PH:7.48 PaCO2:30mmHg BE:-6mmol/L E、PH:7.38 PaCO2:56mmHg BE:+5mmol/L

193、失代偿性呼吸性酸中毒的表现为:

194、呼吸性酸中毒合并代谢性碱中毒的表现为?

195、呼吸性碱中毒的表现为?

A、肺炎球菌肺炎

B、金黄色葡萄球菌肺炎

C、肺炎支原体肺炎

D、肺炎杆菌肺炎

E、病毒性肺炎

以下 X 线表现符合上述哪种疾病?

196、X 线表现片状阴影伴空洞和液平

197、X 线表现多发性蜂窝状肺脓肿,叶间隙下坠

198、X 线表现肺段、肺叶实变、在实变阴影中可见支气管道症?

A、激活腺苷环化酶使 CAMP 形成增加

B、抑制鸟苷环化酶使 CGMP 形成减少

C、抑制磷酸二酯酶阻止 CAMP 衍变成 5’—AMP

D、抑制α肾上腺素能受体阻止 ATP 分解为 ADP

E、稳定肥大细胞膜

199、茶碱的平喘作用是?

200、β2 受体激动剂的平喘作用是?

二.循环内科试题

A题型

1、25 岁,女,心悸、气促、反复咯血,体检:心尖部可闻舒张期隆隆样杂音,P2 亢进,双肺可闻湿性罗音,最可能的诊断为:

A 风心病、二尖瓣狭窄并左心衰竭

B 急性心肌梗塞

C 急性病毒性心肌炎

D 支气管扩张

E 肺梗塞

2、原发性扩张性心肌病的临床表现以下哪一项是错误的?

A 起病缓慢

B 可有气急、甚至端坐呼吸

C 可有肝大、浮肿

D 心电图无病理性 Q 波

E 可有第三心音或第四心音

3、患者男性 36 岁,乏力、晕厥史三天,心电图 P 波与 QRS 波无关系,P 波频率 90 次每分,QRS 波 32 次每分,首选哪种治疗方法?

A 含化硝酸甘油

B 立即同步电复律

C 静脉注射西地兰

D 静脉注射心得安

E 安装临时起搏器

4、患者主诉心悸,常规心电图示窦性心律,75 次每分,多次检查有时出现宽大 QRS 波群,时间 0.12 秒,前后的 R-R 间期为 1.6 秒,心电图诊断最可能为

A 窦性心律、房室传导阻滞

B 窦性心律、房性期前收缩

C 窦性心律、室性期前收缩

D 心房颤动、室性期前收缩

E 窦性心律、右束支传导阻滞

5、原发性高血压的发病机理中,最主要的是下列哪一项:

A 盐酸量过多

B 精神过于紧张

C 肾脏病变

D 钙离子代谢紊乱

E 是一种多因疾病

6、慢性左心功能不全常见的症状和体征不包括

A 夜间阵发性呼吸困难

B 第三心音奔马律

C 双肺底湿罗音

D 咳嗽

E 颈静脉怒张

7、风心病二尖瓣狭窄病人最常见并发症是:

A 脑栓塞

B 阵发性心动过速

C 亚急性感染性心内膜炎

D 心房颤动

E 过早搏动

8、以下哪种器质性心脏病最不可能触及震颤:

A 二尖瓣狭窄

B 二尖瓣关闭不全

C 主动脉瓣关闭不全

D 肺动脉瓣狭窄

E 急性心肌梗塞室间

隔穿孔

9、男性、52 岁、每晨 5 时左右胸骨后疼痛,查体:心脏不大,无杂音,发作时心电图呈 I、II、AVF 导联 ST 段抬高,约 30 分钟后恢复正常,其诊断最可能的是:

A 冠心病心肌梗塞

B 中间综合症

C 稳定型心绞痛

D 卧位型心绞痛

E 变异型心绞痛

10、55 岁患者,发现高血压 10 余年,检查:BP 23.4/13.39kP(180/106mmHg),心尖搏动呈抬举样,心电图呈左室肥厚伴劳损,尿蛋白阳性,血钾 4.2mol/L,最可能诊断为:

A 高血压病缓进型 I 期

B 高血压病缓进型 II 期

C 高血压病缓进型

III 期 D 肾性高血压

E 原发性醛固酮增多症

11、急性高血压的诊断哪项是错误的:

A 病程两年

B 舒张压大于 130mmHg

C 年龄 40 岁

D 无高血压家庭史

E 眼底无出血、渗血

12、下列哪一项体征表示心包大有积液:

A 有心包摩擦音

B 心脏向两侧扩大

C 心音遥远

D 有奇脉

E 心脏扩大并有心包填塞表现

13、下列哪项表现对诊断肥厚型心肌病最可靠:

A 胸骨左缘听到收缩期杂音

B 有心肌病家族史

C ECG 示室间隔非对称性肥厚

D 心电图左室肥大,病理性 Q 波

E 胸痛,运动时昏厥

14、感染性心内膜炎的治疗原则首选:

A 细菌培养阳性后才用抗生素治疗

B 用抑菌抗生素治疗

C 及早用大剂量杀菌剂做长期治疗

D 及早输血

E 体温下降后即可停用抗生素

15、一名 37 岁的女性风心病患者,发热近三周,有关节痛,心率 110 次每分,绝对不齐,脾可触及,球结膜下有出血点,Hb:9.0g/dl,尿蛋白(十),最可能合并:

A、风湿活动 B 亚急性细菌性心内膜炎 C 肺部感染 D 肾小球肾炎

E 关节炎16、二尖瓣分离术最适应于:

A、55 岁以上风心二狭患者

B、心功能 IV 级风心二狭患者

C、晚期妊娠风心二狭患者

D、3 月内无风湿活动的单纯二狭患者

E、6 月内无风湿活动的单纯性二狭患者

17、男性,50 岁,多年高血压病史,二天来因情绪波动,突感头痛加剧,伴恶心、呕吐,视物模糊,检查: BP :30. 7/17.3kPa(230/139mmHg),心率 90 次每分,期前收缩 2-4 次每分,肺叩清音,颈软,目前应采取下列哪项治疗措施为最佳:

A、静脉点滴心律平

B、静注速尿

C、静点硝普钠

D、静注安定

E、静注利多卡因

18、患者,男,35 岁,血压升高经常在 24/17.3kPa(180/105mmHg),无高血压家族史,体检:上腹部闻及血管杂音,应首先选择下列哪种检查以确诊:A、静脉肾盂造影 B、放射性核素肾图 C、肾动脉造影

D、尿中 VMA 测定

E、尿 17-羟、17 酮测定

19、风心病二尖瓣狭窄患者,随着右心衰竭的加重,下列哪项临床表现将最可能减轻: A、肝肿大压痛 B、心率增快 C、急性肺水肿发作

D、心尖区舒张期隆隆样杂音

E、胃肠道淤血

20、主动脉瓣关闭不全无下述哪种体征:

A、股动脉枪击音

B、主动脉瓣区舒张期杂音

C、脉压增大

D、Graham-Steehh 杂音

E、Austin-Flint 杂音

21、男性,19 岁患扁桃腺炎 7 天,感心悸,心率 90 次每分,心电图示 P-R 间期 0.24 秒,应诊断为: A、窦性心动过速 B、窦性心动过缓伴不齐C、II 度 I 型房室传导阻滞 D、II 度 II 型房室传导阻滞 E、

I 度房室传导阻滞

22、56 岁,男性患者,近一个月每天晨起时,胸骨后闷压感,持续数分钟缓解,即时心电图显示 II、III、

avF 导联 ST 段抬高 0.1-0.5mv,最合适的治疗是:

A、心得安

B、氨茶碱

C、地高辛

D、心痛定

E、巯甲丙脯氨

23、男性,21 岁,血压持续增高,阵发性加剧半年,血压升高时测定血中肾上腺素、去甲肾上腺素及其代谢产物(尿 VIA)显著增高,最可能诊断为:

A、肾实质性高血压

B、原发性醛固酮增多症

C、嗜铬细胞瘤

D、皮质醇增多症

E、高血压危象

24、女性,50 岁,半年来上坡感心悸,气急,2 小时前剧烈活动后突感心悸、气急加重,咯粉红色泡沫痰,查体:端坐位,口唇紫绀,颈静脉未见,左室增大,心率为 140 次每分,双肺广泛湿罗音,首选诊断为:

A、支气管哮喘急性发作

B、急性支气管肺炎

C、急性肺水肿

D、自发性气胸

E、全心衰竭

25、洋地黄中毒时最常见的心律失常是:

A、心房颤动合并 III 度房室传导阻滞

B、室性早搏呈二联律

C、阵发性室上性心动过速

D、非阵性性交界性心动过速

E、阵发性房性心动过速伴 II 度房室传导阻滞

26、男,26 岁,半月前曾患严重感冒,近日来有心悸感,活动后尤为明显,查体:心率 104 次每分,有早搏约 10 次每分,第一心音减弱,心尖部可闻及收缩期杂音 3 到 6 级,心电图示:频发室早,I 度房室传导阻滞,非特异性ST-T 波改变,诊断可能性最大的是:

A、急性心包炎

B、风心病二尖瓣狭窄合并关闭不全

C、病毒性心肌炎

D、充血性原发性心肌病

E、冠心病

27、以下哪种情况最有洋地黄治疗指征:

A、缩窄性心包炎伴肝大、腹水

B、甲亢伴心动过速

C、急性风湿热伴心动过速

D、房颤伴心室率快及心功能不全

E、肺心病伴心功能不全

28、急性下壁心肌梗塞最易合并:

A、肺水肿

B、心源性休克

C、室上性快速性心律失常

D、室壁瘤

E、房室传导阻滞

29、男性,58 岁,三小时前突感胸痛,压榨样,伴冷汗淋漓,气短,来院急诊,查体:气稍促、心界不大、律齐,心率 90 次每分,无明显杂音,此时最主要应立即检查:

A、心脏超声心动图

B、抽血查心肌酶谱

C、心电图检查

D、X 线胸部摄片

E、全身 CT(计算机 X 线体层扫描)

30、不是心包填塞的表现是:

A、颈静脉怒张

B、血压下降

C、奇脉

D、心音低远

E、心包摩擦音

31、男性、65 岁,有高血压病史 20 余年,平时不间断服用降压药物,1 小时前突然出现明显头痛,烦燥心悸,多汗,恶心,呕吐、视物模糊、面色潮红,BP 为 33.0/16.9kPa(260/130mmHg),本病例的治疗首先考虑选用:

A、利尿剂口服

B、限制钠摄入

C、硝普钠静滴

D、β受体阻滞剂

E、复方降压片

32、急性心肌梗塞病人,突然出现心尖区响亮的收缩期杂音,无震颤,心衰明显加重,最可能的原因是: A、室壁膨胀瘤 B、左心室扩大二尖瓣关闭不全 C、合并亚急性感染性心内膜炎 D、乳头肌功能失调或断裂

E、室间隔穿孔

33、急性心肌梗塞最常见的病因为:

A、冠状动脉栓塞

B、冠状动脉炎

C、冠状动脉先天畸形

D、冠状动脉痉挛

E、冠状动脉粥样硬化

34、26 岁风心病二尖瓣狭窄伴二尖瓣关闭不全患者,发烧二周,体检:T 38.3 度,口腔粘膜及下肢皮肤瘀点,心率 104 次每分,律齐,二尖瓣区返流样 II 级收缩期杂音及舒张期杂音,主动脉瓣区舒张期杂音,

腹软,脾左肋下 2cm,有压痛,该患者应进行下列哪项检查最能明确诊断?

A、ESR+AEO

B、尿常规加肾功能

C、血培养

D、心电图

E、RF 因子

35、鉴别慢性充血性心力衰竭与门脉性肝硬化所致腹水,下列哪一项检查最可靠: A、肝脏肿大程度 B、双下肢浮肿

C、静脉压升高

D、腹水常规化验检查

E、心率增快

36、下列哪种疾病最可能引起右心室后负荷增加: A、主动脉瓣关闭不全

B、阻塞性肺气肿

C、三尖瓣关闭不全

D、房间隔缺损

E、贫血性心脏病

37、一患者,女性,28 岁,自述心脏病 8 年,查体:心尖部舒张期隆隆样杂音,主动脉瓣区 III 级收缩期喷射样杂音向颈部传导,胸骨左缘三、四肋间舒张期吸气样杂音向心尖部传导,最正确的诊断应是:

A、二尖瓣狭窄合并主动脉瓣关闭不全

B、主动脉瓣狭窄并主动脉关闭不全

C、二尖瓣狭窄并相对性肺动脉瓣关闭不全

D、二尖瓣狭窄并主动脉瓣狭

窄 E、二尖瓣狭窄并主动脉瓣关闭不全及主动脉瓣狭窄。

38、左心功能不全最早期的体征为:

A、舒张早期奔马律

B、肺水泡音

C、颈静脉搏动

D、肝肿大

E、下肢浮肿

39、肝颈静脉回流征阳性最常见于:

A、右心室肥大

B、肺气肿

C、右心功能不全

D、肝硬化

E、左心功能不全

40、室上速伴差异与室速在心电图上常难区分,如发现以下条件中哪一项可确诊为室速: A、心室率 180 次每分B、QRS 间期大于 0.11 秒 C、心室率绝对不规则D、有室性融合波

E、P 波不见

41、62 岁男性,陈旧性心肌梗塞患者,反复心绞痛发作 8 天,如果 ECG 不能肯定这次是否有急性心肌梗塞存在,应首先进行下列哪种酶学检查:

A、ALP

B、CPK-MB

C、SGOT

D、SGPT

E、LDH

42、男性,35 岁,发热,胸部持续性痛二天,胸痛于仰卧时加剧,向左臂放射,用硝酸甘油无效,心音减低,伴舒张期附加音,BP14.7/10.7kPa(110/80mmHg),下肢浮肿,静脉压 180mm 水柱,ECG:ST 段抬高,弓背向下,未见 Q 波,诊断最可能为:

A、急性心肌梗塞

B、缩窄性心包炎

C、变异型心绞痛

D、稳定型心绞痛

E、急性渗出性心包炎

43、下述哪一种表现不同于右心功能不全的体征: A、双肺满布中小水泡音

B、颈静脉怒张

C、胸水和腹水

D、肝脏肿大

E、下肢凹陷性水肿

44、风湿性心脏病二尖瓣狭窄时,心脏听诊无下述哪一种体征: A、心尖区隆隆样舒张期杂音 B、肺动脉瓣区第二心音亢进 C、心尖区粗糙的收缩期杂音 D、心尖区第一心音亢进 E、心尖区二尖瓣开放拍击音

45、有一例风湿性心脏病联合瓣膜病患者,心尖部有舒张期隆隆样杂音,以下哪一种体征对联合瓣膜病的诊断最有意义:

A、胸骨右缘第二肋间收缩期杂音

B、胸骨下端 III 级收缩期吹风样杂音

C、心尖部舒张期震颤

D、胸骨左缘第 3 肋间舒张期吹风样杂音

E、P2 亢进

46、男性,30 岁,体检发现心尖部低调舒张期杂音,胸骨左缘第三肋间与胸骨右缘第二肋间可闻递减型舒张期杂音,血压 18.6/6.7Kpa(140/50mmHg),为明确诊断,首选下列哪项检查:

A、心电图

B、胸部 X 线照片

C、超声心动图 D 放射性核素心血管造影

E、CT

47、下列哪一种血清酶对急性心肌梗塞最具特异性:

A、LDH

B、LDH1

C、SGOT

D、CPK-MB

E、CPK-BB

48、男性、50 岁,6 小时前突发剑突下剧列疼痛,约持续 15 分钟,并向左肩部放射,伴呕吐,冷汗及气促,血压 25.3/14.5KPa(190/110mmHg),腹平软无压痛,心电图 V3、V4 导 T 波高达 2.5MV,ST 段抬高 0.3mV,下列哪项检查最有诊断价值:

A、血常规

B、血沉C,肝胆 B 超D、血清心肌酶测定E、超声心动图

49、急性心肌梗塞时,心源性休克的最主要原因是: A、呕吐、多汗造成的低血容量B、室性早搏

C、心肌收缩力减弱,心排出量降低

D、剧烈疼痛神经反射引起周围血管扩张

E、乳头肌功能不全

50、动脉粥样硬化病变最易累及冠状动脉哪支?

A、左冠状动脉主干

B、右冠状动脉窦房结支

C、左冠状动脉窦房结支

D、左冠状动脉前降支

E、左冠状动脉回旋支

51、一急性下壁心肌梗塞患者,病后两天,突感头晕,血压:9.2/6Kpa

(70/50mmHg),心律齐,心率 41 次每分,应首先考虑为:

A、窦性心动过缓

B、窦性心动过速 C 房性早搏 D、室性早搏 E、III 度房室传导阻滞

52、血压骤然升高,剧烈头痛,昏迷的患者,诊断最可能属于下列哪一项: A、高血压病第一期B、高血压病第二期 C、高血压病第三期D、急进型高血压 E、高血压脑病

53、男、40 岁、间断胸痛 1 年,10 分钟前于睡眠中突发胸痛,痛醒伴冷汗不敢活动,心电图 V1-4 导联 ST 段抬高 0.1 至 0.4MV,ST 段与 T 波升支融合,含服硝酸甘油后先进行下列哪项处理:

A、连续观察心电图变化

B、测定血清心肌酶

C、立即拍胸部 X 线片

D、作超声心动图检查

E、连续观察心电图及血清心肌酶变化

54、女性、25 岁,心悸气促半年入院,查体:颈静脉怒张,心界扩大明显,S1 钝、心尖 SM2/6 级,肝肋下 3 厘米,下肢浮肿,心电图示频发多源性室性早搏,最可能的诊断是:

A、风心病心衰

B、心包积液

C、缩窄性心包炎

D、扩张型心肌病

E、限制型心肌病

55、急性心肌梗塞实验室检查,诊断的特异性最高的是: A、GPT(谷草转氨酶)

B、血沉增高

C、血清肌酸磷酸酶的同工酶 CPK-MB

D、外周血象白细胞增高

E、LDH(乳酸脱氢酶)升高

56、心电图诊断为房性早搏,在 II 导联中出现下列哪项表现,最符合房性早搏的心电图诊断 A、提早出现变形异位 P 波,P‘—R间期 0.16秒

B、提早出现侧置异位 P 波,P‘—R间期 0.10秒

C、QRS 波之前隐约可见 P 波,P—R间期 0.06秒

D、提早出现正常 QRS 波,其后可见倒置 P 波,P—R间期 0.20 秒

E、提早出现宽大畸形的 QRS 波

57、心源性哮喘与支气管哮喘鉴别困难时应首先选用下述哪种药物?

A、麻黄素

B、吗啡

C、茶碱

D、肾上腺素

E、甲丙基肾上腺素

58、急性心肌梗塞时,下述哪种酶学变化既准确又能判断梗塞图: A、CPK

B、CPK-MB

C、GOT

D、LDH

E、LDH2

59、肥厚型梗阻性心肌病时,下述哪种措施最可能使胸骨左缘 3-4 肋间的喷射性收缩期杂音减弱:

A、屏气动作(试验)

B、含化硝酸甘油

C、给α受体阻滞药

D、给β受体阻滞药

E、静注慢心律

60、一位 43 岁女教师,时常头晕,心悸,体检发现亢奋,血压 22.6/15.1KPa (170/114mmHg),左室肥厚,眼底视网膜动脉变细变窄,临床上最可能属于:A、重度高血压B、第三期高血压C、第二期高血压 D、恶性高血压

E、轻度高血压

61、男、56 岁,因胸骨后压榨样疼痛 1 小时,急诊来院,体检:血压 100/60mmHg,心率 194 次每分,心音低,心电图显示:P 波辨不了,QRS 波宽大畸形,连续节律基本规则,下列哪一种药为急用首选:

A、心律平

B、乙胺碘呋酮

C、利多卡因

D、心得安

E、溴苄胺

62、高血压脑病最严重的临床表现是:A、血压突然增高 B、视力模糊 C、恶心呕吐 D、心悸气短 E、剧烈头痛伴抽搐

63、男性,60 岁,阵发头昏头痛,心悸、气促已 5 年,血压 21.3/14.6Kpa (160/110mmHg),血清尿素氮

9mmol/L,肌酐 220mmol/L,尿蛋白(++),治疗时下列哪类药物不宜使用: A、钙拮抗剂 B、血管紧张素转换酶抑制剂C、α肾上腺素能拮抗剂 D、β受体阻滞剂 E、血管扩张剂

64、患者,男,66 岁,三个月前患心肌梗塞,出院后反复出现心悸、气喘,不能平卧,心尖部听及 III—IV 级期杂音,无震颤,心率 110 次每分,双肺听及湿性罗音,上述情况最大可能是:

A、心梗合并室间隔破裂

B、心梗合并乳头肌断裂

C、心梗合并肺部感染

D、心梗合并风心二尖瓣关闭不全 F、再梗塞

65、下列哪一项不符合心房颤动的心电图特点:

A、无独立存在的 P 波

B、有频率和振幅不等的 f 波

C、R—R 间期绝对不等

D、QRS 波群振幅绝对不等

E、QRS 波形形态略有差异

66、下述哪一种疾病不出现心功能不全:

A、急性心肌梗塞

B、扩张型心肌病

C、高血压性心脏病

D、心血管神经官能症

E、主动脉瓣关闭不全

67、左冠状动脉降支闭塞最可能引起以下哪一组心肌梗塞:

A、左室广泛梗塞

B、左室前壁、心尖部、前间隔

C、左室隔面

D、左室高侧壁

E、左室隔面、左室高侧壁

68、男性、52 岁、患冠心病一个月,近三日腹泻,呕吐,8 小时尿量 50ml,烦燥不安,血压 10.6/7.9KPA

(80/60mmHg),心率 100 次每分,PCWP 为 6mmHg。最恰当的治疗是: A、静滴硝普钠B、静注西地兰

C、静滴去甲肾上腺素

D、静滴低分子右旋糖酐

E、静滴异搏定

69、左心功能不全引起的肺淤血和肺动脉瓣区第二心音亢进可因出现右心功能不全而减轻,其主要原因是: A、静脉压增高B、动脉血压下降C、右心排血量减少

D、左心排血量增加

E、右心室扩张

70、下列哪种的临床表现和缩窄性心包炎最相似:

A、肥厚型梗阻性心肌病

B、血液粘度增加

C、限制型心肌病

D、肺源性心脏病

E、缺血性心脏病

71、高血压病所致的左室后负荷增加,与以下哪项的关系最为密切: A、主动脉顺应性降低B、血液粘度增加C、相对性主动脉狭窄 D、外周血管阻力增加 E、动脉血容量增加

72、风心病二尖瓣狭窄及主动脉关闭不全患者,合并房颤,心率 130 次每分,在应用洋地黄诊疗过程中,下列哪些情况应立即停止用药

A、心电图出现鱼钩样 ST—T 改变

B、心率 58 次每分

C、血压轻度降低

D、QT 间期缩短

E、夜间尿量增多

73、患者男性 58 岁,诊断急性前壁心肌梗塞,发病第二天突然出现短阵意识丧失,抽搐,心电图上最可能出现的心律失常是哪一种:

A、房性早搏

B、心室颤动

C、心房颤动

D、室性早搏

E、窦性心律不齐

74、男性,24 岁,阵发性心慌 10 年,每次心慌突然发生持续半至 3 小时不等,本次发作来急诊室检查,心律齐,心率 200 次每分,心电图 QRS 波群形状正常,P 波不明显,诊断最可能为:

A、心房颤动

B、窦性心动过速

C、心房扑动

D、阵发性室性心动过速

E、阵发性室上性心动过速

75、突发心前区疼痛一小时来院急诊,疑为急性心肌梗塞,下列哪项可以不出现: A、动脉下降 B、心率加快 C、白细胞升高D、病理性 Q 波E、高大T 波

76、下列哪一项是 AMI 溶栓疗法的禁忌症:

A、68 岁女性

B、近期有手术史

C、有心肌梗塞史

D、有糖尿病

E、发病 6 小时

77、一青年女患者,因心悸咯血 2 小时来院急诊,查体:双肺布满湿罗音和哮鸣音,心尖部闻及舒张期杂音,肺动脉瓣区第 2 心音亢进,ECG 示窦性心率,心率 106 次每分,B 超示二尖瓣口面积小于 1.0cm,立即自理应首选哪一项:A、阿托品皮下注射B、西地兰静注C、速尿静注D、垂体后叶素静注

E、多巴酚丁胺静滴

78、导致肺水肿的最直接的原因:

A、肺动脉压力增高

B、右心室压力增高

C、肺静脉压增高

D、右心室舒张末压升高

E、右心房压力增高

79、男性患者,62 岁,急性前壁心梗 20 小时后出现右心功能不全表现,临床上为了较正确的评价左心室功能状态,以便指导治疗的最佳措施是:

A、心电图

B、测肘静脉压

C、漂浮导管监测

D、胸部 X 光片

E、超声心动图

80、下列哪种疾病最可能有奇脉:

A、原发性限制型心肌病

B、肥厚型梗塞性心肌梗塞

C、原发性扩张性心肌病

D、病毒性心肌炎

E、二尖瓣狭窄

81、关于老年人高血压:

A、不易发生心力衰竭

B、血压波动性较大

C、以舒张压增高为主

D、不易发生体位性低血压D、患病率不随年龄增长而增加

82、对原发性心肌病的诊断下列的哪项检查为首选:

A、X 线检查

B、心电图检查

C、超声心动图

D、放射性核素检查

E、心脏CT

83、女性、23 岁,因风心病二尖瓣狭窄首次就诊查体,心脏大,心率快,哪项处理合适:

A、地高辛

B、西地兰静注

C、心得安

D、强的松

E、乙胺碘呋酮口服

84、一多年高血压病患者,心电图检查示陈旧型前壁心肌梗塞,频发室性早搏,尿常规检查红细胞(+),颗粒管型(+),眼底检查有动静脉交叉现象,血压

21.3/12KPA(160/90mmHg),患者最可能属于:

A、高血压 I 期

B、高血压 II 期

C、高血压 III 期

D、恶性高血压

E、以上都有是

85、重度主动脉瓣狭窄最常见的临床表现是: A、心绞痛与房室传导阻滞

B、房室传导阻滞与晕厥

C、心绞痛与晕厥

D、快速心律失常与心衰

E、瘁死与心源性休克

86、男性,46 岁,近 4 年诊断为“临床性高血压”,这类患者的主要临床表现为: A、血压波动,有时血压正常B、早期发生左心功能不全 C、易出现体位性高血压 D、常合并冠心病与脑血管硬化 E、常呈现短暂性脑缺氧表现(TIA)87、女、23 岁,风湿性心脏病心衰,用地高辛及双氢克尿塞治疗 5 天,气促加重,心电图呈现二联律,下列治疗哪项是错误的:

A、停用地高辛

B、补钾 C 加用利多卡因D、加用速尿 E、加用血管扩张剂

88、男性、60 岁,两年来劳动强度大时心悸、胸闷,休息或减轻强度后可缓解,

检查血压 16/10.7KPA

(120/80mmHg),心率 80 次每分,律齐,心界不大,无杂音,本次为初诊,按序首先检查: A、胸部 X 片 B、超声心动图C、常规心电图D、动态心电图

E、运动心电图

89、下列哪项不是心功能代偿的表现:

A、心率加快

B、心脏增大

C、静脉压增高

D、血液重新分布

E、有效血容量增加

90、一名 60 岁的男性患者,劳力性呼吸困难 3 年,双下肢水肿半月,查体:口唇发绀,颈静脉怒张,心

率 110 次每分,律齐,S1、S2 均减弱,P2>A2,心尖部闻及室性奔马律及 3/6 级收缩期杂音肝脾肿大,双下肢水肿,为正确诊断,最需做的是:

A、心电图

B、超声心动图

C、测静脉压

D、心电向量图

E、胸部X 线检查

91、疑为冠心病心绞痛最可靠的检查手段是:

A、ECG

B、HOTER

C、UCG

D、心肌酶学检查

E、选择性冠状动脉造影

92、慢性充血性心力衰竭的诱发因素最常见的是:

A、严重的心率失常 B 妊娠与分娩C、过度的情绪激动 D、各种感染E、输液过快过量

93、预激综合症合并快速房颤时,下列哪项治疗是错误的:

A、西地兰

B、利多卡因

C、奎尼丁

D、普鲁卡因酰胺

E、乙胺碘呋酮

94、男性 40 岁,阵发性血压增高二年,发作时伴头晕,心悸,出汗,面色苍白,心动过速,恶心呕吐,晕厥,体检:心脏浊音界向左下扩大,腹部未闻及杂音,组织胺激发试验阴性,ECG 示快速心律失常,血压达 34.6/17.3KPA (260/130mmHg),酚妥拉明试验血压降到 120/70mmHg,尿蛋白(+),24 小时 VWA (50umol),诊断首选:

A、原发性醛固酮增多症

B、肾动脉狭窄

C、原发性高血压

D、慢性肾炎

E、嗜铬细胞瘤

95、心绞痛发作中疼痛发生机理推测最可能是:

A、电生理改变

B、心脏收缩功能障碍

C、多肽类刺激心脏内传入神经末梢

D、冠状血管痉挛引起

E、目前尚不清楚

96、男性 36 岁,三周前感冒,发热咳嗽流涕,一周而愈,近几天来,又感头晕,胸闷气短,心率 106 次每分,律齐,心尖区闻及二期收缩期杂音,WBC:10.8X10^9/L,ESR:25mm/H,CPK 及 LDH 均升高,心电图 T 波普遍倒置,诊断最可能的是:

A、风湿性心肌炎

B、病毒性心肌炎

C、亚急性感染性心内膜炎

D、冠心病、心肌缺血

E、心内膜下心肌梗塞

97、下列哪项是测量左心室前负荷最直接的方法:

A、肺毛细血管楔嵌压

B、心输出量

C、左心室舒张末期压

D、血压

E、平均左心房压

98、女性 19 岁,半月前上感,2 天来心悸气短,体温 38.1 度,两肺底闻及湿罗音,肝肋下 3CM,触痛,双下肢水肿,心电图:窦性心律,P—P 间期 0.48 秒,P—R 间期 0.24 秒,II、III、avF 导联,V3 导联 ST 段压低 0.10—0.15MV,T 波平坦或浅倒置,有关此病人,下列哪项不正确:

A、病人有全心衰竭伴房室传导阻滞

B、可考虑诊断为病毒性心肌炎

C、病因不能除外风湿性心肌炎

D、应作血清学检验:CPK、GOT、LBIL、CK—MB

E、如血清心肌酶明显增高则应考虑为无 Q 性急性心肌梗塞。

99、男性、35 岁,主动脉瓣区闻及 DM 及二尖瓣区 SM 二级,近月出现乏力,心悸,气促加重,不发热,体检发现皮肤有瘀点,主动脉区除 DM 外还有 SM,脾可触及,HB:70g/L,最可能的诊断的是:

A、风心病、心衰

B、风心病、风湿活动

C、风湿病伴亚急性心内膜炎

D、贫血性心脏病

E、先天性主动脉瓣病变

100、诊断二度房室传导阻滞最主要的依据是:

A、QRS 波群脱落

B、P—R 间期延长

C、R—P 间期不规则

D、心室率缓慢 E 房室脱节

101、预激征候群患者最常见的并发症的心律失常是: A、窦性心动过速

B、房室传导阻滞

C、心房扑动

D、室上性心动过速

E、心房颤动

102、心肌炎病人反复出现阿—斯综合征,心电图示 III 度房室传导阻滞,最恰当的处理方法是: A、口服心的安B、口服硝酸甘油C、安装人工心脏起搏器 D、静滴氢化可的松E、静滴阿托品

103、某 32 岁的女性,血压增高 3 年,伴乏力,多尿,查血钠 142mmol/L,血钾 2.9mmol/L,PH7.455,最可能的诊断为:

A、高血压病

B、嗜铬细胞瘤

C、皮质醇增多症

D、原发醛固酮增多症

E、慢性肾炎

104、二尖瓣狭窄患者,正确的是:

A、出现急性肺水肿由于左心室排血量降低

B、最常见的心律失常是房性早搏

C、声音嘶哑常见

D、最早症状是游走性关节疼痛

E、后期大咯血反而少见105、女性、21 岁,受凉及劳累而出现畏寒,胸闷胸痛 12 小时入院,血压

105/75mmHg,咽充血,心界正常,心率 106 次/分,律齐,心音低钝,心电图I、II、III、avF、ST 呈弓背上抬达 1.5mv 并于前支融合,血清 CPK:1046U/L,CPK—MB:480U/L,入院后 24 小时突然四肢抽搐,心电活动消失,既而呼吸停止,立即行复苏手术,安装临时起搏器,加用激素等治疗,复苏成功,4 天后血清酶正常,1 周后心电图正常,三周后出院,最可能的诊断是:

A、急性心肌梗塞

B、急性前壁心肌梗塞

C、休克型肺炎

D、急性病毒性心肌炎爆发症

E、急性心包炎

106、一名 20 岁的女学生 5 周前咽部疼痛,体温 38 度,心悸气短,活动后加重,心率快,心脏扩大,心尖部闻及奔马律,何检查可明确心脏病性质:A、十二导联心电图B、二维超声心动图C、心肌酶学检查 D、血清病毒抗体滴定E、核素心肌显像检查

107、肾脏有严重损害的高血压病,不宜用:

A、硝普钠

B、心痛定

C、双肼苯达唪

D、胍乙啶

E、心得安

108、男性、72 岁,发热、咳嗽 4 天,病后进食少,既往经常心悸、胸闷,求诊于当地保健院,诊断为肺部感染,给予抗感染、补液等治疗,10 小时后突然发生心悸、咳嗽加重,气急,被迫坐起,最大可能由于:

A、发热

B、肺部感染加重

C、输液反应

D、输液不足

E、急性肺水肿

109、以下哪项说法最正确:

A、正常时心包腔压力高于大气压

B、正常时心包腔压力高于心房压

C、正常时心包腔压力低于心室舒张压

D、少量渗液可使心包内压力明显上升

E、心包腔内渗液超过 200ml 时,才会使心包内压力迅速上升。

110、男性、20 岁,经常感冒,最近有心悸,不能平卧,下肢水肿,查体:颈静脉稍充盈,心界轻度扩大,心尖部第一心音减低,有病理性第三心音和杂音,诊断最可能为:

A、急性风湿热

B、病毒性心肌炎

C、结核性心包积液

D、冠心病心衰

E、肥厚型梗阻性心肌病111、某女性患者,28 岁,血压持续升高 25.33/16.00KPA(190/120mmHg),B 超和腹部 X 线检查结果为右肾明显缩小,应首先选择以下哪项检查明确诊断:A、测定血浆肾素与儿茶酚胺含量B、快速联系静脉肾盂造影

C、肾动脉造影

D、反复多次尿培养及肾功能检查

E、肾脏区计算机断层X 线摄影

112、临床最常见的继发性高血压是:

A、嗜铬细胞瘤

B、原发性醛固酮增多症

C、主动脉弓狭窄

D、肾血管性高血压

E、慢性肾脏病

113、风心病二尖瓣狭窄病人最常见的并发症是:

A、脑栓塞

B、阵发性心动过速

C、亚急性感染性心内膜炎

D、心房颤动

E、过早搏动

114、女性、24 岁,已婚,体检发现心尖区局限性隆隆样杂音,DM 第一心音亢进呈拍击样,为避免病情加重其首要措施是:

A、避免过劳及妊娠

B、预防和治疗链球菌感染

C、瓣膜置换

D、地高辛

E、硝酸甘油

115、早搏的代偿音期是否完全首要决定于:

A、窦房结起搏频率加快

B、窦房结节律受到干扰否

C、早搏起源部位不同

D、早搏出现的早晚

E、交界区传导速率的快慢

116、下列哪一项判断最正确:

A、主动脉瓣狭窄、左房左室增大显著,心衰发生晚

B、二尖瓣狭窄、左房大显著,心衰发生晚

C、主动脉瓣关闭不全、左房左室大显著、心衰发生早

D、二尖瓣关闭不全、左房左室大显著,心衰发生晚

E、二尖瓣脱垂,左房左室大显著,心衰发生早

117、(略)

118、男、64 岁,患高血压多年,一年来血压常为 22.6/14.6KPA (170/110mmHg)左右,近一周呼吸困难,咳嗽,X 线呈左室扩大,肺瘀血,眼底检查出血、尿常规正常、最可能诊断为:

A、高血压病第一期

B、高血压病第二期

C、高血压病第 III 期

D、恶性高血压

E、高血压危象

119、对于心肌梗塞(MI)下列哪种说法最正确:

A、血清 CPK 正常可排除 MI

B、血清 CPK 增高程度不能反映梗塞范围

C、ECG 正常不能排除 MI

D、心向量图诊断 MI 不如 ECG 敏感

E、无病理性 Q 波 MI 都是累及心室壁全层

120、慢性充血性心力衰竭的主要原因为:

A、前负荷

B、后负荷增加

C、前后负荷均增加

D、心肌收缩力减弱

E、心肌舒张期顺应性下降

121、一名 49 岁的女性,突感心悸,立即做心电图,QRS 波群时限小于 0.12 秒,频率 164 次每分,每个 QRS 波群后可见逆行 P 波,R—P 间期为 0.14 秒,P 波在 II、III 及 AVF 导联倒置,AVR、I 直立,最可能的诊断是:

A、窦性心动过速

B、房性阵发性心动过速

C、交界性阵发性心动过速

D、心房扑动

E、室性阵发性心动过速

122、男性、71 岁,患高血压病 25 年,平时血压最高可达 24/15KPA

(180/112mmHg),本次因情绪波动诱发头痛、恶心、视物模糊、少尿、咳粉红色泡沫痰,不能平卧,现出血及视乳头水肿,此患者最可能属于高血压病的哪一种临床类型:

A、恶性高血压

B、高血压危像

C、高血压脑病

D、临界性高血压

E、急进性高血压

123、60 岁男性,因心前区压榨性闷痛 1 小时入院,临床症状不除外心肌梗塞,为确定诊断采取下列哪一步骤最重要:

A、观察血压变化

B、进行超声心动图检查

C、观察含服硝酸甘油的效果

D、严密观察心电图演变

E、抽血测肌酸磷酸激酶

124、风心病房颤患者,近一周服用地高辛 0.25mg 和速尿 20mg,每日二次,突然倒地,人事不醒,四肢抽搐,心音消失,最可能的原因是:

A、快速性房颤

B、脑栓塞

C、室颤

D、III 度房室传导阻滞

E、心力衰竭

125、男性、66 岁,因胸骨后疼痛 2 天,出冷汗,皮肤凉 1 小时来院就诊,测BP:10.6/6.6KPA(60/50mmHg), ECG 示:V1—V6、AVL 导联的 ST 段明显抬高,并有深而宽的 Q 波,血清 CPK—MB 峰高出正常的 12 倍,该患者的冠脉病变,最可能的原因是:

A、左冠脉前降支

B、左冠脉回旋支

C、左冠脉主干

D、左冠脉回旋支+左室前支

E、左回旋支+中隔支

126、肥厚型心肌病心电图出现病理性 Q 波的最可能机理是: A、心肌过度肥厚引起心肌缺血 B、心肌缺血、坏死 C、左室肥厚、劳损

D、心肌间质水肿

E、肥厚的室间隔除极向量增大

127、糖尿病患者,60 岁,胸骨后疼痛 1 天入院,查:BP21.33/13.3KPA

(160/100mmHg),心脏不大,无杂音,心电图示 ST 段除 AVR 外,余导联普遍水平压低 0.2mv,首选最佳检查方法是:

A、血沉

B、心肌酶谱

C、放射性核素扫描

D、超声心动图

E、冠状动脉造影

128、一名 16 岁女性患者,因胸闷、心悸就诊,二周前患上感,体检:心率 100 次每分,律不齐,心杂音阴性,两肺阴性,EKG:P‘—QRS—T 波群提前出现,II、III、AVR 导联 P’波倒置,P‘—R 间期 0.08 秒,QRS 波群正常,有完全性代偿性间隙,心电图诊断最可能为:

A、房性早搏

B、室性早搏

C、窦性心律不齐

D、交界区性早搏

E、间歇性预激

129、45 岁男性,发热 36.5 度,心前剧痛 8 小时,伴有咳嗽气短,ECG 示除AVR 外 ST 段均有抬高,哪个诊断的可能性最大:

A、夹层主动脉瘤合并感染

B、急性广泛前壁心肌梗塞

C、急性心包炎

D、急性心肌炎

E、变异性心绞痛合并感染

130、急性下壁及右室心肌梗塞者,血压 10.6/6.6KPA(80/50mmHg),心率 62 次每分,心律整,双肺呼吸音正常,PCWP:8mmHg,下列哪项措施为首选:

A、用硝酸甘油

B、给予补液

C、利尿剂

D、给予多巴胺升压

E、给β—受体阻滞剂

131、男患者 25 岁,心悸气促,反复咯血,查梨形心影,心尖区有舒张期隆隆样杂音,肺动脉瓣第二心音

亢进,肺底部有湿罗音,此次就诊时大咯血,BP(150/90mmHg),治疗首选:A、西地兰B、阿托品C、速尿D、垂体后叶素E、止血敏

132、引起亚急性感染性心内膜炎最常见的致病菌为: A、金黄色葡萄球菌

B、肠球菌

C、革兰氏阳性细菌

D、厌氧菌

E、草绿色链球菌133、患者女性,18 岁,学生,因感冒查体发现心尖部舒张期隆隆样杂音,伴开瓣音,心率 72 次每分,律齐,肺阴性,肝脾未及,下肢不水肿,超声心动图示:二尖瓣瓣口面积 1.6 平方厘米,平时活动不受限,应首选作哪项处理:A、避免重体力活动B、二尖瓣分离术C、洋地黄治疗 D、利尿剂治疗

E、青霉素预防、定期随诊

134、男性、14 岁,间断胸痛,气短 2 年,半小时前于起立时,意识丧失,即

刻清醒,查体:心脏轻度增大,闻及第四心音,胸骨左缘 3,4 肋间喷射性收

缩期杂音,心电图 II,III,AVF 导联 Q 波大于 1/4R 波,宽 0.02 秒,有关此病人下列哪项不正确:

A、晕厥可能因严重的心律失常所致

B、心脏杂音是左心室流出首狭窄所致

C、

应诊断为急下壁心肌梗塞并发室间隔穿孔 D、应注意其家族史 E、应提醒病人避免剧烈运动

135、男性、65 岁,有心绞痛史 10 余年,2 年来反复发作心动过速,一日来

持续发作 20 多小时,检查心电图为阵发性房性心动过速,采用刺激迷走神经方法无效,现不应选用哪种治疗:

A、甲氧胺

B、心律平

C、奎尼丁

D、异搏定

E、西地兰

136、女性 30 岁,持续高血压 3 年,常为 180-190/110-120mmHg,感四肢无力、麻木,尿量增多,为确诊高血压的原因最有价值的检查是:

A、血儿茶酚胺测定

B、血钾、尿钾测定

C、尿 VMA 测定

D、血糖、

尿糖E、血清 T3、T4、TSH

137、有关心绞痛的药物治疗,下列哪项最正确: A、β受体阻滞剂与硝酸酯有

协同作用,但两者抗心绞痛的机理不同,所以使用时不须减速量 B、长期服用

β受体阻滞剂的病人,一旦发生明显心动过缓,应立即停药 C、在应用β受体阻滞剂的病人,如发生冠状动脉痉挛,可同时应用维拉帕米(异搏定) D、治

疗变异型心绞痛以钙拮抗剂的疗效最好,如需停药时应逐渐减量,以免发生冠

状动脉痉挛 E、潘生丁是一种强冠状动脉扩张剂,心绞痛发作时,只要用足够剂量常可改善心肌血供。

138、一男性,80 岁,既往有高血压病史,近日患胃肠炎,在输液过程中突然

气促,不能平卧,剧烈咳嗽,咯大量粉红色泡沫痰,现最好使用:

A、利血平

B、胍乙啶

C、心得安

D、硝普钠

139、下列哪一项检查最有助于病毒性心肌炎的诊断:

A、心电图检查

B、心内膜心肌活检

C、血沉检查

D、白细胞计数 F、血清中和抗体滴度检查

B 型题:

A、嗜铬细胞瘤

B、原发性醛固酮增多症

C、皮质醇增多症

D、肾动脉狭窄

E、主动脉缩窄

140、上肢高血压、下肢血压降低为上述何病的临床表现:

141、高血压伴低血钾、高血钠及代谢性碱中毒为上述何病的临床表现:

A、安体舒通

B、速尿

C、甘露醇

D、双氮酰胺

E、甘氮酰胺

142、急性肺水肿首选的利尿剂为:

143、伴有心功能不全的脑水肿首选的利尿剂:

A、降低心脏前负荷

B、降低心脏后负荷和抑制心肌收缩力

C、扩张阻力性小血管和抗血小板聚集力

D、抑制大脑皮层达到镇痛

E、减速慢心率和抑制心肌收缩力

144、硝苯吡啶可:

145、硝酸甘油可:

A、利多卡因

B、吗啡

C、地高辛

D、双氢克尿塞

E、维拉帕米

146、患者、男性、60 岁,咳嗽咯白色泡沫样痰二小时,有高血压,心脏病史,体温正常,呼吸急促,双肺闻及湿性罗音,心率 110 次每分,心律规整,首选哪种药物:

147、患者女性,31 岁,突然发作心慌胸闷一小时,心电图示心率 180 次每分,节律规则,QRS 波与窦性相同,逆行 P 波,首选哪种药物治疗:

A、心电图

B、X 线胸卡

C、核素心功能

D、心脏 CT 检查

E、超声心动图148、患者女性,心尖区可闻及粗糙 III 级收缩期杂音,为进一步明确瓣膜情况,首选哪一项检查:

149、患者男性,58 岁,吸烟,糖尿病史,持续性胸骨后痛 4 小时,神志清楚,出汗,第一心音减弱,首选哪一种检查方法:

A、突然胸痛、心电图示:病理性 Q 波,ST 段弓形上抬,T 波倒置

B、突然胸痛,气促、紫绀、心电图示:电轴右偏,肺型 P 波

C、心电图示病理性 Q 波及 ST 段弓形上抬已持续 6 个月

D、心前区痛,心电图除 AVR 导联外,多数导联 ST 段弓背向下型抬高

E、突然胸痛,心电图示 V4—V6 导联 ST 段下移,T 波倒置

150、心室壁瘤最符合以上哪一组特点:

151、急性肺动脉栓塞最符合以上哪一组特点:

A、西地兰 0.4mg 以葡萄糖液稀释至 20ml,缓慢静脉注射

B、口服地高辛

0.25mg,每日一次

C、地高辛 0.25mg,每日三次,二天后每日一次

D、暂时不用洋地黄

E、先用半量西地兰(0.2mg),稀释后缓慢静脉注射,必要时 2 小时后可重复152、急性心肌梗塞 24 小时内伴左心衰竭

153、高血压患者,夜间出现左心衰竭,近二周未用过洋地黄

154、风湿性心脏病,伴房颤,原每日口服地高辛 0.25mg,近 7 天因故未服,心衰加重,心率增快 A、收缩期负荷过重B、舒张期负荷过重C、AB 均有

D、AB 均无

E、心肌耗氧量增加

155、风湿性二尖瓣狭窄引起的右心衰竭是由于:

156、风湿性主动脉瓣狭窄及关闭不全引起的左心衰竭是由于:

157、二尖瓣脱垂、重度二尖瓣关闭不全引起的左心衰竭是由于:

A、硝普钠

B、硝苯地平(心痛定)

C、卡托普利

D、血利平

E、哌唑嗪158、钙拮抗剂

159、α受体阻滞剂

160、ACE 抑制剂

A、左房粘液瘤

B、二尖瓣狭窄

C、二叶式主动脉瓣

D、主动脉瓣关闭不全

E、二尖瓣脱垂

161、Austin—Flint 杂音最常见于:

162、开瓣音最常见于:

A、心尖区 II 级收缩期杂音

B、肝颈静脉返流征阳性

C、左心室肥大

D、心尖区舒张期奔马律 F、粘液性水肿

163、最能提示左心功能不全的表现是:

164、最能提示右心功能不全的表现是: A、原发性心肌收缩力减弱

B、左心室前负荷过重

C、左心室后负荷过重

D、右心室前负荷过重 F、心室充盈受限

165、主动脉瓣及二尖瓣关闭不全的病因是:

166、扩张型心肌病的病因是: A、心肌细胞肥大、变性、纤维化等混合病变为主 B、心肌细胞肥大、形态奇异,排列紊乱 C、心肌细胞融解,间质水肿,单核细胞浸润等 D、心内膜增厚,心内膜下心肌纤维化 E、心肌细胞排列不规则伴间质纤维化

167、肥厚型心肌病的表现是:168、病毒性心肌炎的表现为:

男性,62 岁,急性前壁心肌梗塞,如发生下列情况首选如何处理: A、室性早搏,二联律 B、室颤C、阵发性室性心动过速 D、III 度房室传导阻滞

E、I 度房室传导阻滞

169、非同步直流电击复律:

170、不予处理:

A、室性心律失常

B、房室传导阻滞

C、室性心律失常加房室传导阻滞

D、右心衰竭加休克

E、室间隔穿孔

171、下壁心肌梗塞最易引起:

172、前壁心肌梗塞最易引起:

A、血压:180-170/80-85mmHg

B、血压:250/150mmHg

C、血压有极大波动

D、晨起后血压升高,体力活动后下降为正常或低于正常

173、上述哪一项提示有动脉粥样硬化?

174、上述哪一项是恶性高血压?

175、上述哪一项有间脑功能紊乱:

A、劳累型心绞痛

B、自发型心绞痛

C、持续性胸痛

D、胸部闪电式刺痛176、心痛定用于:

177、消心痛用于

178、吗啡用于:

A、4-6 平方厘米

B、1.5-2.0 平方厘米

C、6-8

D、1-1.5

E、2-3 179、正常二尖瓣口面积是:

180、二尖瓣中度狭窄面积是:

A、奇脉

B、Graham—Steell 杂音

C、DE—Musset 征

D、Ewart 片

E、Rotch 征

181、主动脉瓣关闭不全病可有:

182、风心病二尖瓣狭窄致相对性肺动脉瓣关闭不全出现: A、急性渗出性心包炎B、急性胰腺炎C、急性胆囊炎 D、急性心肌梗塞E、急性肺动脉栓塞

183、男性、68 岁、既往高血压病史,因上腹痛,恶心呕吐就诊,面色苍白,大汗淋漓,脉搏细弱,有抬举性心尖搏动,心率 120 次每分,律齐,心音低钝,BP(82/52mmHg),血清淀粉酶测定(180U),ECG 示窦性心动过速,II、III、AVF 导联 ST 段上抬 0.3MV:

184、22 岁,女性患者,伴心慌,气短,发烧 2 月,体检:心界扩大,心音遥远,心律齐,心率 120 次每分,BP:90/72mmHg,胸透:心脏搏动减弱,肺野清晰,EKG:低电压,普遍导联 T 波倒置:

A、急性非特异性心包炎

B、急性病毒性心肌炎

C、结核性心包炎

D、亚急性感染性心内膜炎 F、肿瘤性心包炎

185、抗感染治疗原则,早期、联合、适量、规律、全程用药,适用于:

186、抗感染治疗原则,早期、足量、长疗程用杀菌药适用于:

A、硝普钠

B、可乐定

C、肼苯哒嗪

D、卡托普利

E、阿替洛尔

187、合并冠心病的高血压:

188、恶性高血压:

A、草绿色链球菌

B、霉菌

C、肺炎球菌

D、流感嗜血杆菌

E、柯萨奇病毒189、从临床表现看下述病例的病原体,以哪种可能性最大:

男、40 岁,患风湿性心脏病已 10 年,近一个月不规则发热,心尖部 III 级收缩期杂音,主动脉瓣 II 级拍水样杂音,肝脾肿大,右足拇趾发生坏疽,白细胞 15X10^9,尿常规 RBC++,血糖 3.5mmol/L:

190、男,8 岁,因淋雨后发热 4 天,伴咽痛、咳嗽,经治疗 1 周后症状减轻,但疲倦无力,又出现心悸,胸闷,面色苍白,心电图示 II 度 AVB,心率 110 次每分,GOT:150 单位;WBC:10X10^9/L:

A、β受体阻滞剂

B、双氢克尿塞

C、血管紧张素转换酶抑制剂

D、硝普钠

E、复方降压片

以下病例治疗时首选上述哪种药物:

191、男性,65 岁,干部,剧烈头痛,伴有恶心呕吐半天入院,血压(260/130mmHg),心率 88 次每分,甘油三脂 3.5mmol/L,空腹血糖:8.2mmol/L,尿蛋白、尿糖+

192(缺)

A、房室交界区的自律性增高或触发活动

B、折返发生于房室结、心房、房室结

C、折返回路是正常房室传导通道与旁道的结合

D、心房自律性增高

E、心房内多个异位节律点自律性增高

193、房室折返性心动过速

194、自律性房性心动过速

195、房室结折返性心动过速

A、Graham—Steell 杂音

B、Austin—Flint 杂音

C、胸骨左缘 3-4 肋间收缩期杂音

D、心尖区收缩中晚期喀喇音

E、胸骨左缘第二肋间连续性杂音

196、二尖瓣脱垂

197、肥厚型梗阻性心肌病:

A、奇脉

B、水冲脉

C、交替脉

D、短绌脉(脉搏短绌)

E、重脉

198、心肌梗塞:

199、主动脉关闭不全:

200、冠心病左心功能不全:

三.消化内科试题

A题型

1、男性、55 岁,上腹痛 10 年,餐后痛,服药有效,近三个月来,腹痛变为无规律,食欲减退,查体:轻贫血貌,上腹压痛,未扪及包块,多次粪便隐血,应考虑下述哪一项诊断最合适:

A、胃溃疡活动期

B、胃溃疡并慢性穿孔

C、胃溃疡并胃炎

D、胃溃疡恶性变

E、胃溃疡并出血

2、急性胰腺炎一般在起病后几小时后血清淀粉酶开始升高:

A、0.5 小时

B、2 小时

C、8 小时

D、24 小时

E、48 小时

3、克隆氏病最常累及的部位:

A、空肠

B、回肠末段

C、升结肠

D、降结肠

E、直肠

4、为预防急性胰腺炎的发生应注意哪项:

A、吸烟

B、少食脂肪

C、戒酒及避免暴饮暴食

D、餐后剧烈运动

E、精神紧张

5、慢性腹泻患者解粘液脓血便伴里急后重,多次粪便常规与培养均未见致病细菌及原虫,经水杨酸偶氮磺胺吡啶治疗症状明显改善,最应考虑:

A、溃疡性结肠炎

B、克隆病

C、结肠癌

D、肠结核

E、慢性细菌性痢疾

6、患者诊断肝硬化并自发性腹膜炎,治疗应首选:

A、注射大量青链霉素

B、注射先锋铋

C、注射庆大霉素加先锋 IV

D、注射氯霉素

E、注射先锋霉素 IV 加灭滴灵

7、女性,昨晚餐后突感上腹剧痛,恶心、呕吐,今晨腹痛向腰部放射,为尽快明确诊断,首选哪项检查: A、血常规B、血电解质C、血尿淀粉酶

D、腹部 B 超

E、胃镜检查

8、男性,40 岁,突然上腹剧痛,伴恶心,后延至脐周、全腹,体检:全腹压痛,尤其以上腹明显,且浊音消失,肠鸣音减弱,血淀粉酶 500 单位(快速法,正常是 1000 单位),最可能的诊断为:

A、急性胰腺炎

B、胆囊炎、胆石症

C、急性小肠梗阻

D、消化性溃疡穿孔

E、急性胃扭转

9、肝硬化肝功能代偿期的临床表现最早出现的是:

A、恶心、厌油

B、乏力和食欲不振

C、腹胀、腹痛

D、上腹不适或恶心

E、腹泄

10、男性,59 岁,有慢支和肝炎史,近一周精神错乱伴少尿,体检:昏迷状态,口有臭味,心肺阴性,腹膨隆,腹水征阳性,扑击样震颤未引出,首选考虑:A、肺性脑病B、肝性脑病C、糖尿病酮症酸中毒 D、尿毒症E、药物中毒

11、下列哪项与肝性脑病主要诱因无关:

A、感染

B、上消化道出血

C、便秘

D、低钙血症

E、尿毒症

12、关于甲胎蛋白的临床意义哪项不正确:

A、动态观察有助于诊断早期原发性肝癌

B、对判断原发性肝癌手术治疗效果有价值

C、慢性活动性肝炎亦可有暂时性增高

D、阴性结果可排除原发性肝癌诊断 F、对估计原发性肝癌的愈后有价值

13、下列哪项临床表现对诊断门脉高压有特征性价值:

A、脾肿大

B、肝缩小

C、肝质地硬化

D、腹水

E、侧支循环的建立和开放

14、肝肾综合征与下列何种因素最有关:

A、肾皮质血流减少

B、肾皮质坏死

C、肾乳头坏死

D、肾间质炎症改变

E、胆红素对炎症的毒性作用

15、判断肝硬化肝细胞功能最有用的指标是:

A、血清胆固醇

B、血清 ALP

C、血清转肽酶

D、血清白蛋白及凝血酶原时间

E、LDH

16、男性,32 岁,近三天解柏油样便,每日 2-3 次,以往常有返酸,上腹痛,晚间重,进食可缓解,查体贫血貌,肝脾不大,右上腹压痛,最可能的诊断:A、肝硬化并胃底静脉曲张出血 B、胃癌并出血C、胃炎并出血 D、溃疡病并出血 E、胆石症并出血

17、肝硬化腹水时哪一类最常见:

A、低钾低氯血症

B、低钠血症

C、高钾血症

D、低钠合并碱中毒

E、低钙血症

18、肝硬化引起脾大的主要原因:

A、门脉压力增高

B、肝静脉压力增高

C、肝动脉压力增高

D、毒性产物的刺激

E、腹水的压迫使脾血液回流障碍

19、急性有机磷中毒的机理,哪项正确: A、局部刺激作用B、缺氧

C、麻醉作用

D、干扰细胞膜或细胞器的生理功能

E、抑制胆碱脂酶的活性

20、青年男性上腹痛,经胃镜检查确诊为复合性溃疡,判定其发病机理最可能为: A、遗传因素 B、幽门排空障碍致胃窦潴流,胃泌素分泌增加所致 C、十二指肠液返流,胃粘膜屏障受损,H 离子回渗所致 D、壁细胞总数增加,高胃酸分泌所致E、长期吸烟

21、病人在昏迷抽搐,瞳孔明显缩小,皮肤湿泠,多汗,呕吐,流涎,分泌物增多,两肺可闻及湿罗音,吸吸困难,下列何种疾病的可能性最大:

A、CO 中毒

B、巴比妥药物中毒

C、中毒

D、有机磷农药中毒

E、苯胺中毒

22、诊断慢性胃炎,首选下列哪项最为适宜:

A、胃液分析

B、胃镜检查

C、临床表现

D、X 线检查

E、酶学检查

23、女性、45 岁,心窝部疼痛反复发作一年,体检上腹部轻压痛,最可能诊断为: A、胃癌B、胃息肉C、胃溃疡D、胃平滑肌瘤 E、胃体胃炎

24、下列哪一项不是诊断肝硬化的依据: A、发热、腹胀、乏力B、门脉高压的临床表现 C、肝功能减退的临床表现D、肝活检有假小叶形成 E、食道吞钡,X 线检查示食道静脉曲张

25、女性,20 岁,服农药 105g,昏迷 2.5 小时入院,查体:血压(60/40mmHg),昏迷多汗,小便失禁,两瞳孔小如针尖,全身肌肉阵发震颤,应用阿托品治疗,下列哪项错误:

A、心须早期注射足量阿托品

B、静脉给药,尽快阿托品化

C、达阿托品化后立即停药

D、与胆碱脂酶复能剂合用时阿托品剂量应减少

E、积极观察全身反应,瞳孔大小,随时调整阿托品计量

26、下列哪项是假神经递质:

A、多巴胺

B、β羟酪胺

C、甲基多巴

D、5-羟色胺

E、酪胺酸

27、下列哪项检查对肝硬化的肝功能判断最有意义:

A、血清酸性磷酸酶

B、血清腺苷脱氨酶

C、白蛋白/球蛋白比例倒置

D、血清单胺氧化酶

E、血清丙氨酸氨基转换酶

28、男性、35 岁,上腹隐痛半年,无节律性,一个月前钡餐检查,见胃角部有

2.5cm 突出腔外的龛影,近

3 天来每天解成形黑便一次,量不多,血压正常,血红蛋白 95g/L,最可能的诊断是: A、慢性浅表性胃炎B、十二指肠球部溃疡 C、胃角溃疡

D、胃癌

E、胃粘膜脱垂症

29、肝硬化病人,近一周发烧、腹痛,且水明显增加,腹水检查结果:淡黄色,比重 1.018,蛋白 25g/L,李凡他试验阳性,细胞总数 609/ul,其中,中性点:80%,MAO:60mmol/h,下列哪种疾病可能性最大:

A、结核性腹膜炎

B、自发性腹膜炎

C、门静脉血栓形成

D、原发性肝癌

E、功能性肾衰

30、男性 44 岁,近五年来经常上腹部烧灼感,返酸、腹泻,(每日 4-5 次水泻),常有夜间上腹痛症状,血清胃泌素为 560ug/ml,胃液分析,BAO:15mmol/h,MAO:60mmol/h,下列哪种疾病可能性最大:

A、慢性萎缩性胃炎

B、胃溃疡

C、十二指肠溃疡

D、胃粘膜脱垂

E、胃泌素瘤

31、以下哪一项不是代偿期肝硬化的临床表现:

A、乏力

B、食欲不振

C、鼻衄出血

D、肝脏轻度肿大

E、脾脏轻度肿大

32、患者突然呕血约 700ML,查体:腹部满,肝肋下未触及,脾肋下触及 1.5cm,腹水征可疑,下肢浮肿

(+),首选下列哪种药物: A、甲氰咪胍(西米替丁)B、垂体后叶素

C、氢氧化铝凝胶

D、维生素 K

E、6—氨基已酸33、患者因消化道出血住院三天,今晨起感头昏,心慌、查血压(80/60mmHg),脉搏 118 次每分,血尿素氮 12.4mmol/L,最可能的原因是:

A、肾功能不全

B、电解质紊乱

C、酸中毒

D、继续出血

E、低血糖

34、患者右下腹痛三月余,常有腹泻偶有便秘,大便呈糊状,无脓血,无里急后重感,X 光线钡餐检查回肠末端及盲肠可见激惹征象,最可能的诊断是:A、溃疡性结肠炎 B、溃疡型肠结核 C、慢性肠炎 D、阿米巴痢疾E、结肠癌

35、有机磷农药中毒病人的临床表现中下列哪一项是不符合的:

A、呼出气有蒜味

B、意识障碍

C、腺体分泌增多

D、瞳孔散大

E、肌纤维颤动

36、关于萎缩性胃炎,下列哪项最可能是癌前病变的病理改变: A、肠腺上皮化生 B、假幽门腺化生 C、重度不典型增生 D、粘膜固有腺体萎缩E、腺体萎缩伴局部增生

37、男性、70 岁,突起右侧肢体瘫痪,讲话不清,于次日清晨呕血,黑便数次,既往无腹痛病史,上消化道出血最可能的病因是:

A、溃疡病活动期

B、食管静脉曲张破裂出血

C、胃癌

D、急性糜烂性胃炎

E、急性单纯性胃炎

38、男性,30 岁,酗酒后突感上腹持续钝痛 2 小时,向腰背部呈带状放射,体检:上腹有中度压痛,无反跳痛,双肾区无叩击痛,EKG 及血常规正常,为明确诊断,应首选哪项检查:

A、淀粉酶测定

B、血清脂肪酶

C、肝功能检查

D、肾功能检查

E、尿常规检查

39、刺激壁细胞分泌盐酸的物质哪一个是错误的:

A、组织胺

B、乙酰胆碱

C、胃泌素

D、蛋白食物

E、西米替丁

40、关于 A 型萎缩性胃炎的临床特点,下面哪个是错误的:

A、病变在胃体

B、胃酸在减少

C、胃泌素增加

D、伴贫血多见

E、抗壁细胞抗体阴性多见

41、男性患者 70 岁,因偏瘫昏迷就诊,诊断为脑溢血,患者随后出现呕血、黑便,最可能的病因是: A、胃溃疡B、十二指肠球部溃疡 C、胃癌

D、肝硬化食管胃底静脉曲张破裂出血

E、急性糜烂性胃炎

42、十二指肠球部溃疡患者,三天上腹痛加重,反复呕吐,为胃内容物及咖啡样物,共 3 次约 500ML,无宿食,吐后疼痛缓解,最可能合并:

A、幽门梗阻

B、上消化道出血

C、穿孔

D、癌变

E、弥漫性腹膜炎

43、32 岁,男性,因关节疼痛,经常服止痛片已 3 年,近日上腹部不适,昨晚突然呕暗红色血液约 150ML,下列哪项措施最有价值:

A、急诊钡餐透视

B、吞线试验

C、急诊内窥镜检查

D、胃部 CT 检查

E、腹部

B 超检查

44、肝肾综合征时,下列各种变化哪项是错误的:

A、自发性少尿或无尿

B、氮质血症

C、稀释性低钠血症和低尿钠

D、肾小球滤过率正常

E、肾脏缺乏重要的病理改变

45、肝性脑病时下列哪项不正确:

A、轻度性格改变和行为异常,脑电图无明显异常

B、倦睡,行为异常,脑电图无常

C、意识错乱,行为异常,脑电图不正常

D、昏睡或精神错乱,脑电图不正常

E、神志完全丧失,不能唤醒,脑电图不正常

46、肝性脑病诱因下列哪项可能性不大:

A、上消化道出血

B、大量排钾、利尿、放腹水

C、高蛋白饮食

D、肾上腺蛋白质激素增多

E、便秘、腹泻、感染

47、在急性糜烂性胃炎的病因中,下列哪项是错误的: A、大面积烧伤

B、颅内病变

C、创伤和/或休克

D、受凉和/或进食不当

E、重危疾病或器官衰竭

48、男性,44 岁,上腹部饱胀,隐痛不适作胃镜检查:肝窦部粘膜红白相间,以白为主,病理切片示:胃粘膜显示慢性炎症,粘膜肌层增厚,粘膜固有腺体减小,HP(幽门螺杆菌)感染,采取哪种方案最合适:

A、H2 受体阻滞剂

B、1%稀盐酸合剂

C、铋剂

D、硫糖铝

E、碱性药物

49、48 岁女性,上腹部不适 3 年,近 2 年来上腹不规则疼痛,食欲减退及体质减轻,T37.8 度,HB85g/L, ESR:78mm/H,粪便急血试验++,下列哪项检查最有必要:

A、胃液分析

B、B 型超声波检查

C、胃镜检查

D、血管造影检查

E、CT 检查

50、上消化道大出血最常见的原因是:

A、肝硬化食管胃底静脉曲张破裂

B、糜烂性出血性胃炎

C、消化性溃疡

D、胃癌

E、胃粘膜脱垂

51、男性 35 岁,右上腹空腹痛反复发作 5 年,近一段加重,近三天黑便(成形),为进一步明确诊断应首选检查:

A、胃肠钡餐透视

B、胃液分析

C、粪便稳血

D、便查钩虫卵

E、胃镜

52、男,30 岁,三年来反复右上腹疼痛,午夜尤甚,先后发生 3 次上消化道大出血,胃镜检查胃及十二指肠球部未见异常,体检肝脾不大,右上腹压痛:

A、胃粘膜脱垂症

B、球后溃疡

C、穿透性溃疡

D、慢性胃炎

E、幽门管溃疡

53、女性,20 岁,发现昏迷,尿失禁半小时入院,血压(150/90mmHg),呼吸正常,两瞳孔缩小,多汗,面肌颤动,鼻孔有泡沫样分泌物,应首选哪种有效抢救措施:

A、吸氧

B、阿托品

C、

D、

E、

54、男性、30 岁,3-4 次/日稀便,有时带有粘液及血,伴里急后重二年,近一周加重,并发烧,左下腹有压疼,应首选哪项检查:

A、粪便常规

B、粪便细菌培养

C、血常规

D、纤维结肠镜检查

E、血清蛋白电泳

55、女性患者,28 岁,低热,腹胀半年,查体:消瘦,颈静脉无怒张,心肺未见异常,腹部膨隆,肝大剑突下 2.5CM,脾未触及,腹水征阳性,脐至两侧髂上棘距离相等,下肢无浮肿,血沉 68mm/小时,血清蛋白电泳 A60%,γ24%,谷丙转氨酶 25U,最可能的诊断是:

A、肝硬化失代偿期

B、结核性腹膜炎

C、巨大卵巢囊肿

D、心包炎

E、心功能不全

56、28 岁,间歇性上腹痛三年,近来腹胀痛加重,伴呕吐,吐出物为隔餐食物,抗酸剂治疗无效,检查胃部有震水音,转动体位症状不能缓解,最可能的诊断是:

A、消化性溃疡合并幽门梗阻

B、良性十二指肠瘀滞症

C、粘膜脱垂症

D、胃癌

E、胃下垂

57、男性、45 岁,乙型肝炎 30 年,近二个月来,乏力腹泻,睡眠倒置,性格与前不同易急燥,化验 GPT125U, A/G:2.5/3.2,B 超示:肝回声不均匀、增强,门脉内径 1.6CM,少量腹水,下列哪些是最正确的:

A、服镇静药、卧床休息

B、应用抗菌素控制腹泻

C、不必用降血氨药物

D、积极应用利尿剂

E、禁止蛋白质饮食

58、女性,26 岁,腹泻 1 年,每日 2-3 次,进食蔬菜、蛋白后腹泻加重,多

次查粪常规,可见红、白细胞,严重时可见肉眼脓血便,粪培养多次阴性,为明确诊断,最有效的诊断方法是:

A、依靠临床表现

B、粪常规及培养

C、X 线钡剂灌肠

D、纤维结肠镜检

E、血清免疫球蛋白

59、男性,65 岁,三个月来持续上腹隐痛,多次查大便外观黄色,血阳性,食欲差,消瘦明显,面色苍白,上腹压痛阳性,未触及包块,首先应做哪项检查对确诊最有帮助:

A、X 线钡餐检查

B、胃镜检查

C、胃液分析

D、胃泌素测定

E、癌胚抗原(CEA)检测

60、男性,32 岁,反复上腹痛二年余,向背部放射疼痛加重,曾二次大出血,经抢救治愈,最可能的诊断: A、十二指肠球部溃疡 B、胃溃疡C、幽门溃疡D、球后溃疡E、胃癌

61、男性、40 岁,因上腹部不适及黑便而作胃镜检查:胃窦部粘膜严重出血、水肿,小弯处有多处糜烂及粘膜下出血点,病理切片下有中性粒细胞、淋巴细胞浸润,血管破坏及出血,曾因关节疼痛服用过消炎痛等药物,应考虑诊断何病最适宜:

A、急性糜烂性胃炎

B、胃窦部多发性胃炎

C、萎缩性胃炎

D、胃泌素癌

E、肥厚性胃炎

62、男性 30 岁,经常发生周期性上腹部疼痛,服用铁剂不能缓解,采用左侧卧位缓解,最可能的诊断为: A、十二指肠溃疡B、胃溃疡

C、胃窦部炎症

D、胃粘膜脱垂症

E、胆石症

63、一男性 45 岁,上腹部疼痛 5 年,以餐后半小时疼痛明显至下次餐前可缓解,近一周节律性疼痛消失呈顽固性疼痛,腹部放射至背部,下列哪项诊断可能性最大:

A、幽门管溃疡

B、球后溃疡

C、复合性溃疡

D、穿透性溃疡

E、癌性溃疡

64、女性 26 岁,腹胀、午后发热一个月,腹部触诊柔韧感,移动性浊音阳性,肝大,肋下 1.5CM,脾未触及,下腹部可触及边缘不整的肿块,最可能的诊断:A、慢性肾炎B、卵巢囊肿C、结核性腹膜炎 D、肝硬化

E、腹腔内肿瘤

65、在我国急性胰腺炎的病因最常见的:

A、药物诱发

B、腮腺炎合并

C、高血钙症

D、胆囊炎、胆结石

E、壶腹部肿瘤

66、下列哪些与肝臭关系最密切:

A、胆碱

B、胆红素

C、甲基硫醇

D、支链氨基酸

E、α-酮戊二酸

67、男性 40 岁,突然呕血 300ml,暗红色并解黑便二次,查体:蜘蛛痣、肝肋下 1.5cm 质硬,脾肋下 3cm,质韧,少量腹水,首选治疗方法是:

A、静脉注射 Vit K

B、输白蛋白

C、法莫替丁静注

D、服用氢氧化铝凝胶

E、静脉滴注垂体后叶素

68、急性有机磷农药中毒最主要的死亡原因:

A、肺水肿、呼吸肌瘫痪

B、抽搐、全身衰竭

C、心律失常、心跳骤停

D、阿托品治疗过量

E、休克

69、肌肉颤动的表现,有助于哪种中毒的诊断:

A、CO 中毒

B、敌敌畏中毒

C、中暑

D、安眠药中毒

E、铅中毒

70、男 25 岁,来诊前 10 小时自服敌百虫溶液 60ml,经用阿托品后转来,患者昏迷,瞳孔放大皮肤潮红,干燥抽搐,双肺散在干鸣,心率 160 次/分,全血胆碱脂酶活力 30%,10 小时无尿,此患者给予双复磷外,还要停用哪种药:

A、毛果芸香碱

B、阿托品

C、西地兰

D、回苏灵

E、速尿

71、肝性脑病的发病机理:

A、蛋白质代谢紊乱,氨基酸不平衡

B、水电解质平衡紊乱

C、脂肪代谢紊乱

D、慢性肝病导致大脑敏感性高

E、多种因素综合作用的结果

72、以下哪一项与胃粘液—粘膜屏障功能无关:

A、促进粘膜的血供

B、阻碍胃腔内 H+反向弥散

C、维持胃腔与粘膜间酸度差

D、保持粘膜内外的电位差

E、润滑粘膜不受机械磨损

73、CO 中毒的发病机理最主要的是:

A、高铁血红蛋白的蓄积

B、交感神经过度兴奋

C、抑制胆碱脂酶

D、迷走神经过度兴奋

E、碳氧血红蛋白蓄积

74、下列哪项试验指标对有机磷中毒病人的诊断和中毒程度的估计等最为重要:

A、血气分析

B、血电解质

C、血胆碱脂酶活力

D、血乳酸脱氢酶

E、血碱性磷酸酶

75、男性、50 岁,工人,右上腹隐痛伴低热二月余,突发右上腹剧痛,继而全腹痛 1 小时,查体:T38 度,巩膜微黄,腹壁有抵抗感,肝肋下 3 指,质硬,有压痛,脾肋下 1 指,压痛不明显,全腹有弥漫性压痛并移动性浊音:

A、肝硬化并门静脉血栓形成

B、肝硬化并自发性腹膜炎

C、消化性溃疡穿孔

D、胆囊炎胆石症合并胆囊穿孔

E、原发性肝癌破裂

76、女、35 岁,三个月来乏力、纳差、发热、盗汗,伴胃痛,腹泻,查体:右下腹压痛,血 WBC:7.2X10^9,血沉 65mm/H,SGPT 正常,盲肠 X 线钡餐检查发现回盲部呈激惹征象(Stierlin‘ sign),乙状结肠管见肠粘膜大致正常,此病人采取以下措施最好:

A、剖腹探查

B、糖皮质激素

C、SASP

D、灭滴灵

E、抗痨药试验治疗

77、有机磷农药中毒的机理最可能是:

A、乙酰胆碱活性降低,胆碱酯酶积聚

B、胆碱酯酶活性增高,乙酰胆碱减少

C、胆碱酯酶活性降低,乙酰胆碱积聚

D、乙酰胆碱被水解为胆碱及乙酸

E、胆碱酯酶不能被磷酸化

78、重度有机磷家药中毒急性肺水肿,最重要的抢救措施是: A、肌注杜冷丁

B、肌注速尿

C、静注大剂量阿托品

D、静注大剂量解磷定

E、静注西地兰

79、消化性溃疡的发病因素中最主要的是:

A、幽门螺杆菌

B、非甾体消炎药

C、胃蛋白酶

D、前列腺素缺乏

E、胃酸

80、关于消化性溃疡的病理下述哪项不正确: A、溃疡呈圆形或椭圆形,直径小于 2.5CM B、溃疡边缘整齐,具有炎性水肿和结缔组织增生等病变 C、溃疡的底部洁净,覆盖有灰白色的纤维渗出物 D、溃疡发展时或累及肌层和浆膜层,有时穿透浆膜层,引起穿孔 E、溃疡愈合后不遗留疤痕

81、患者上腹部不适二个月,自觉消瘦,查体巩膜明显黄染,皮肤有抓痕,腹软,胆囊可触及,血总胆红素 333.2umol/L,直接胆红素为 238umol/L,尿胆红素阳性,B 超示肝内外胆管扩张,胆囊肿大,胰管稍扩张,最合适的诊断是:A、肝细胞性黄疸B、先天性非溶血性黄疸C、胆石症 D、原发性硬化性胆管炎 E、胰头癌

82、上消化道出血部位是指:

A、食管、胃出血

B、食管、胃、十二指肠球部出血

C、屈氏韧带以上部位出血

D、空肠以上部位出血 F、回盲瓣以上部位出血

83、男、50 岁,患胃溃疡已十年,近三个月来,胃纳减弱,X 线钡餐检查,胃窦部可见 2.5X3.0cm 龛影,过缘不齐,粪便隐血三次阳性,首先应考虑的疾病是:A、胃溃疡出血B、胃溃疡合并胃息肉C、胃溃疡合并胃粘膜脱垂

D、胃溃疡癌变

E、胃溃疡合并幽门梗阻

84、下列哪项对诊断肝硬化最具有参考价值:

A、脾脏肿大

B、蜘蛛痣

C、黄疸

D、肝脏质地坚硬

E、侧枝循环建立

85、关于甲胎蛋白(AFP)哪项是错误的: A、正常值<20mg/ml

B、生殖腺胚胎癌

C、AFP>200mg/ml 持续 8 周可以诊断为原发性肝癌

D、AFP 用扩散法与对流法测定,原发性肝癌的假阳性极多

E、AFP 升高在临床症状出现前 6 个月或更早发现肝癌

86、一肝癌病人,肿瘤直径为 7cm,血胆红素总量 25.5umol/L,直接胆红素:

11.9umol/L,B 超示有少量腹水,首选的治疗方法是:

A、尽早手术切除肿瘤

B、先改善肝功能再做手术

C、行肝动脉栓塞及介入治疗

D、化学药物治疗(静脉用药或口服)

E、中医中药治疗

87、消化性溃疡的预防措施中,哪项是错误的:

A、戒除烟酒

B、避免服用对胃有刺激的药物

C、消除胆汁返流

D、呋喃哩隔维持量,预防复发

E、在好发季节特别注意有规律的饮食起居

88、肝硬化上消化道出血后,为预防肝昏迷的发生最应采取的措施为: A、积极大量输血 B、左旋多巴C、复方氨基酸静滴 D、弱酸液灌肠E、抗炎症冶疗

89、男性、55 岁,有消化性溃疡史 20 年,其间多次复发,近一周来上腹痛,返酸,胃镜:十二指肠溃疡伴胃窦炎,幽门螺杆菌阳性,采用下列哪组治疗有可能减少复发:

A、西米替丁+吗丁淋

B、硫糖铝+胃复安

C、三钾二椽络合铋+法莫替丁

D、西米替丁+氢氧化铝凝胶

E、奥美拉唑+吗丁啉

90、有机磷中毒患者经抢救后肺湿罗音消失,仍昏迷,瞳孔散大,心率 160 次/分,高热,这时进一步的抢救措施应首选:

A、物理降温

B、给予西地兰

C、停用阿托品

D、给予解磷定,加大阿托品剂量

E、用甘露醇降颅压

91、下列哪项不是肝性脑病的诱发因素:

A、口服安体舒通

B、便秘

C、低钾

D、感染

E、腹泻

92、一例大量呕血与黑粪的急诊患者,测血压 10.5/8.5Kpa(80/60mmHg),P134 次/分,出血不止,病因不明,当前的首要措施是:

A、补充血容量

B、急诊胃镜检查

C、急诊手术治疗

D、立即输注止血药物

E、立即三腔二囊管压迫止血

93、关于肝硬化食管静脉曲张破裂大出血,下述哪项最正确: A、出血后腹水减少B、出血后脾脏缩小C、多有明确诱因 D、以黑粪为主要表现

E、全血细胞减少

94、男性、60 岁,因关节痛服消炎痛三片,今晨起上腹不适,随即呕吐咖啡样物,伴鲜血共约 500ml,既往有胆石症病史,其呕血最可能的原因是:

A、胃溃疡并出血

B、胆道出血

C、十二指肠球部溃疡并出血

D、急性胃粘膜病变

E、胃癌并出血

95、下列哪项对肝癌的诊断最有价值:

A、肝区疼痛

B、肝区血管杂音

C、肝质硬

D、肝触痛

E、肝边钝

96、急性昏迷病人,口有特殊气味,体检:巩膜黄染,双侧肢体肌张力对称增高,双侧瞳孔等圆等大,对光反射迟钝,体检检查:血糖:6.5mmol/L,血尿素氮 7.5mmol/L,该患者最可能的诊断是: A、脑血管意外B、糖尿病高渗性昏迷 C、尿毒D、肝性脑病 E、镇静药过量

97、女性患者,54 岁,口服甲拌磷药 1 小时昏迷入院,经阿托品、解磷定等治疗患者清醒,36 小时后停用阿托品,停用阿托品后患者再次昏迷,出冷汗,两肺有湿罗音,该患者再次昏迷最可能的原因是: A、脑血管意外B、肺部感染 C、癫痫发作 D、阿托品用量不足E、低血糖

98、男性,30 岁,胃溃疡患者下列哪项药宜慎用:

A、甲氰咪胍

B、氢氧化铝胶剂

C、普鲁本辛

D、硫糖铝

E、痢特灵

99、男性,41 岁,呕鲜血 300ml,而以上消化道出血急诊入院,既往有胃病史,查体腹部膨隆,移动性浊

音(+),肝未触及,脾肋下 1.5cm,最可能的诊断是:

A、消化性溃疡

B、肝硬化食道静脉曲张破裂出血

C、急性糜烂性胃炎胃粘膜脱垂E、食管—贲门撕裂症

100、一位疑诊肝硬化消化道出血患者,经用垂体加压素静脉滴注,效果不明显,可选择下列哪项检查帮助确诊:

A、插胃管逐段低压吸取消化液,观察有无血迹

B、吞线试验

C、X 线钡餐检查

D、纤维胃镜检查

E、选择性动脉造影

101、女性,18 岁,持续性低热,腹痛,糊状便及闭经 1 个月,体检:腹部饱满,略紧张,全腹轻压痛及反跳痛,右下腹隐约触及不规则肿物较固定,腹部移动性浊音阳性,最可能诊断是:

A、结肠癌

B、结核性腹膜炎

C、克隆氏病

D、侧卵巢囊肿

E、阑尾脓肿102、男,40 岁,患肝硬化腹水二年,近二周来发热、腹痛,尿少,全腹压痛伴轻反跳痛,腹水征阳性,腹水常规为渗出液(白细胞分类以多核为主),周围血白细胞计数增高,应首先采取的治疗措施是: A、积极治疗原发病

B、早期联合足量应用抗菌素

C、加强支持治疗输白蛋白

D、按腹水培养结果选用抗菌素

E、利尿、止痛对症处理103、肠结核和结核性腹膜炎最主要的鉴别点是: A、结核中毒症状B、腹痛腹泻C、腹部肿物

D、腹水

E、白细胞计数及分类

104、对肝硬化门脉高压的诊断最有价值的是:

A、脾大腹水

B、腹壁静脉曲张

C、脾大、脾功能亢进

D、顽固性腹水

E、食道下段静脉曲张

105、十二指肠溃疡患者近几天剧烈呕吐,呕吐物中有宿食酸臭味,查血清钾2.3mmol/L,血清钠 116mmol/L,血清氯化物 86mmol/L,应选择下列哪项措施最好:

A、生理盐水+10%氯化钾静滴及阿托品肌注

B、10%葡萄糖氯化钠静滴及口服胃复安

C、5%葡萄糖+10%氯化钾静滴加阿托品肌注

D、乳酸钠+生理盐水静滴及胃肠减压

E、生理盐水+10%氯化钾静滴及胃肠减压

106、男性、45 岁,上腹痛 2 年余,周期发作,最近一个月持续性疼痛,体检:左上腹局限性压痛,钡餐提示胃角龛影,直径 1cm,胃酸分析稍低于正常,经内科保守治疗 8 周后痛略轻,应首选哪项:

A、继续保守治疗,加用生胃酮

B、继续保守治疗,反复查粪便隐血试验

C、纤维胃镜+活检

D、手术治疗

E、复查钡餐

107、男性,65 岁,进行性黄疸,并感上腹及腰背部隐痛,皮肤瘙痒,消瘦,查体:肝肋下 1.5cm,质韧,脾未及,Courvoisier 征阳性,TB:136umol/L,DB:80umol/L,尿胆红素(+++),尿胆元(-),肝功能正常,为确诊应首选下列哪项检查:

A、放射性核素扫描

B、CT

C、胃肠钡餐透视

D、肝穿

E、B 型超声

108、以下各项对肠结核最有确诊意义的是:

A、粪便抗酸杆菌培养阳性

B、X 线示回肠末段有局限性肠腔狭窄和近端肠段扩张

C、X 线示回盲瓣结构紊乱,有激惹征象

D、瘘管形成

E、回盲端肉芽肿性病变+肠系膜淋巴结干酪样坏死

109、关于急性上消化道出血,下述哪项说法是错误的:

A、出血量超过 50ml,即可出现柏油样便

B、出血早期血压、血红蛋白可正常

C、咖啡渣样液是血液经胃酸作后后形成正铁血红素所致

D、48 小时内可作急胃镱检查

E、大量出血就是食管胃底静脉曲张破裂出血

110、男性 32 岁,慢性腹泻 7 年,大便每 2-3 次,常带少量脓血,粪便镜检红白细胞均 15 个/HP,纤维结肠镜示隆结肠以下粘膜充血,假息肉形成,少数散在浅溃疡,请选择最合适的初始治疗药物:

A、琥珀酸氢化考的松保留灌肠

B、氟哌酸

C、强的松

D、柳氮磺胺吡啶

E、磺胺类药物保留灌肠

111、女性 20 岁,因昏迷,抽搐,被家属发现并送来急诊室,体检:双瞳孔等大为 2mm,皮肤湿冷,多汗,呼吸困难,最可能的诊断为:

A、糖尿病酮症酸中毒

B、脑血管意外

C、低血糖昏迷

D、急性安定中毒

E、急性有机磷家药中毒

112、以下哪一项更符合肝硬化失代偿期的临床表现:

A、蜘蛛痣、腹水、月经失调

B、黄疸、齿龈出血脾大

C、肝掌、腹水、性功能减退

D、黄疸、脾大、胃底静脉曲张

E、腹水、脾大,下肢浮肿

113、有关幽门螺旋杆菌下列哪种说法不正确: A、是革兰氏阴性杆菌B、产生尿毒酶,促进氢离子逆弥散 C、是厌氧菌 D、对胃粘膜有侵袭力 E、产生局部免疫反应

114、患者因肝硬化并上消化道出血住院,经治疗出血好转,但逐渐尿少,每日在 500ml,尿常规少许蛋白,血钾 5.0mmol/L,血钠 129mmol/L,血氯

90mmol/L,血尿素氮 24.1mmol/L,最可能的诊断是:

A、电解质紊乱

B、酸中毒

C、肝肾综合症

D、消化道出血 E 慢性肾炎复发

115、某溶血性黄疸病人,下列检查结果哪项是最符合的:

A、血总胆红素 102umol/L(6mg),直接胆红素 51umol/L(3mg),尿胆原++,尿胆红素++

B、血总胆红素 102umol/L(6mg),直接胆红素 68umol/L(4mg),尿胆原++,尿胆红素++

C、血总胆红素 102umol/L(6mg),间接胆红素 17umol/L (1mg),尿胆原+++,尿胆红素+

D、血总胆红素 102umol/L(6mg),直接胆红素 85umol/L(5mg),尿胆原-,尿胆红素+++

116、男性、60 岁,患高血压病已五年,酒后突然昏迷,查:瞳孔缩小,皮肤积汗,见肌纤维震颤,无瘫痪,最可能的诊断是:

A、脑血管意外

B、酒精中毒

C、有机磷中毒

D、中毒性痢疾

E、安眠药中毒

117、女性、40 岁,上腹胞胀不适,明显厌食,体重减轻,上腹部有轻度压痛,结膜苍白,胃肠钡餐检查未发现异常,经用硫糖铝治疗效果不佳,最可能的诊断为:

A、慢性萎缩性胃窦炎

B、慢性萎缩性胃体炎

C、十二指肠炎

D、功能性消化不良

E、胃癌

118、男性、24 岁,因球后溃疡而胃大部切除术后一个月,上腹部疼痛,呈钝痛,无规律性且经常有腹泻,用制酸药和痢特灵等对症治疗,效果不佳,为进一步明确诊断可选择下列哪项检查最为适宜:

A、便常规+粪便隐血检查

B、胃肠钡餐透视检查

C、B 型超声检查

D、纤维胃镜检查

E、用五肽胃泌素作刺激试验

119、有机磷农药中毒经大剂量阿托品治疗,已达阿托品化,但患者神志又由清醒变狂燥,最后转入昏迷,体检:体温 39.5 度,面部潮红,心率速,肺无罗音,你考虑应如何处理最为适宜:

A、再加大阿托品剂量

B、维持阿托品原剂量

C、暂停阿托品

D、阿托品维持

E、加用大剂量兴奋剂

120、女性、36 岁,过去有胃溃疡病史,饮酒后上腹痛 6 小时,进行性加剧,并蔓延至全腹,检查:有明显腹膜刺激症,以上腹部为主,肝浊音界正常,移动性浊音阳性,血清淀粉酶 200 单位,血钙 1.62mmol/L,最可能的诊断是:A、消化性溃疡穿孔B、急性出血坏死性胰腺炎C、急性化脓性胆囊炎穿孔 D、门静脉血栓形成E、急性高位阑尾炎穿也性腹膜炎

121、溃疡型肠结核的最常见临床表现是:

A、多有结核毒血症

B、常有腹泻,多为血便

C、易并发急性肠穿也

D、常引起结肠出血

E、多不伴有肠外结核

122、女性、56 岁,因上腹部不适,腹胀,纳差,返酸,嗳气,行胃镜检查,镜下所见,胃粘膜红白相间,粘膜下毛细血管可见,幽门口可见大量胆汁返流入胃腔,选用以下哪种药物治疗最适宜:

A、甲氰咪呱

B、胃舒平

C、多潘立酮

D、生胃酮

E、山莨菪碱 654—2

123、关于幽门螺旋杆菌(HP)下列哪项是最正确的:

A、常见于正常胃粘膜

B、常侵犯至细胞内

C、炎症程度与该菌数量不成比例

D、不直接破坏上皮细胞

E、胶体铋对此菌无作用

124、胃镜显示慢性浅表性胃炎有十二指肠液返流,经用制酸药物治疗效果不显,可选用下列哪种药物治疗较适宜:

A、多潘立酮(吗丁啉)

B、呋喃唑酮

C、胶体铋(Demol)

D、普鲁本辛

E、硫糖铝

125、男性,30 岁,腹痛,粘液脓血便 2 月,应用多种抗菌素治疗无效,体重减轻 3 公斤,肛门直肠指检未见异常,目前,最适宜做下列哪项检查:

A、X 线钡餐检查

B、X 线钡剂灌肠检查

C、乙状结肠镜检查

D、纤维结肠镜检查

E、腹部 X 线平片检查

126、男性,29 岁,近 1 个月低热,腹痛,腹泻和便秘交替,腹泻时每日排便2-4 次,粪便呈现糊状,血沉 60mm/h,其父亲曾患”肺结核”,目前,最适宜的检查是:

A、结核菌素试验

B、乙状结肠镜检查

C、腹部超声波检查

D、X 线钡餐检查

E、腹部 CT 检查

127、男性,40 岁,患高血压三年,今晨突然呕鲜血 800 毫升伴血块,继之排柏油便两次,BP140/100mmHg, P110 次每分,肝肋下未及,脾肋下 2cm,腹部移动性浊音阳性,请指出下列治疗措施中,哪项是错误的:

A、三腔二囊管压迫止血

B、PAMBA 静脉滴注

C、垂体后叶素静脉滴注

D、输新鲜血

E、先晶体后胶体为输液原则

128、男性,30 岁,腹泻 6-7 次/日,稀便带粘液脓血,二年来时轻时重,近 20 天来发热,时有便血里急后重,左下腹压痛,曾服痢特灵,氯霉素无效,为明确诊断应首选哪项检查:

A、便常规+找阿米巴样滋养体

B、便培养

C、X 线钡餐检查

D、纤维结肠镜检查

E、血清蛋白电泳分析

129、肝硬化腹水患者,近日尿少,腹胀加重,脉搏 60 次每分,血钾 7.6mmol/L,血钠 130mmol/L,目前首要的治疗是:A、补给钠盐B、注射葡萄糖胰岛素 C、输白蛋白 D、输血E、利尿剂

130、男、50 岁,肝硬化住院治疗中突然呕鲜血 300 毫升,既往患冠心病,下列治疗哪项是错误的: A、三腔二囊管压迫 B、6 氨基已酸静脉滴注C、垂体后叶素静脉滴注 D、凝血酶口服E、去甲基肾上腺素胃管滴注

131、中年女性患者,上腹部隐痛半年,无规律性,近三天来排黑色成型便一次/日,量中等,血压正常,血 hb105g/L,应立即选哪组治疗措施:

A、禁食+输液+止血剂

B、输血+6 氨基已酸+去甲肾上腺素

C、流质+输液+制酸剂+止血剂

D、禁食+输液+制酸剂+冰盐水洗胃

E、禁食+手术治疗

132、青年男性,餐后排便,起身突然晕倒在地,无类似既往史,T:正常,P:124 次/分,BP:10/8KPA

(80/60mmHg),神志清,面色苍白,肢冷出汗,心律齐,无杂音,腹平软,肝脾未触及,肠鸣音亢进,血血素 90g/L,最佳治疗选哪组:

A、静推高渗葡萄糖溶液

B、静滴抗菌素+升压药

C、升压药+强心剂

D、补充 5%碳酸氢钠

E、输血+抑酸剂

133、萎缩性胃炎分为 A 型和 B 型,哪项不是主要依据: A、胃粘膜腺体萎缩的有无B、血清胃泌素的含量

C、恶性贫血的有无

D、血清壁细胞抗体的有无

E、血清内因子抗体的有无

134、女,38 岁,饱餐后 3 小时感右上腹部疼痛,逐渐加重,呕吐二次为食物,4 小时后来诊,既往无上腹部疼痛史,血压 11/9KPA(85/70mmHg),体温 38 度,心肺无异常,上腹部肌紧张,压痛明显,反跳痛阳性,血清淀粉酶 250u(somogyi 法),尿淀粉酶 32u(winsow 法),白细胞 1.2X10^9/L,留观输液禁食 4 小时后,以上症状不缓解,下列哪项检查最有助于诊断:

A、纤维胃镜

B、血清脂肪酶测定

C、复查血清淀粉酶

D、腹部透视

E、心电图

135、十二指肠球部溃疡患者近二天来每天解成型黑便一次,黑便约 50 克,下列哪项组合处理最佳: A、禁食、输液、冷盐水洗胃、雷尼替丁注射B、流质、输液、口服去甲肾上腺素、硫糖铝 C、禁食、输液、注射止血敏、雷尼替丁注射D、流质、输液、硫糖铝,雷尼替丁口服 E、禁食、输液、注射止血敏、雷尼替丁注射

136、因镇痛呕吐、腹泻四天入院,肝脾不大,HbsAg 阳性,给予禁食、输液等治疗稍好转,今晨开始精神萎糜,乏力、肌肉痉挛,继而神志不清,昏迷,为明确诊断,首先应查:

A、血氨

B、脑脊液

C、血电解质

D、脑 CT

E、血淀粉酶

137、男性,腹痛、腹泻二月余,心肺无异常,腹软,左中腹压痛,为明确诊断,首先应检查: A、纤维结肠镜B、腹部 B 超C、钡剂灌肠D、粪便隐血

E、腹部平片

138、关于急性一氧化碳中毒,下述哪项是错误的:

A、老年人较易得病

B、中枢神经系统病变最为显著

C、重度中毒者应立即现场抢救

D、应迅速纠正缺氧

E、重症者可出现神经系统并发症

139、男性、35 岁,有肝炎病史 10 年,近感肝区痛加重,血 ALT 40u,AFP 100u,HbsAg(+),B 超示肝右后叶内侧有 4-5cm 低密度区,下一步应首先做哪一种检查或处置:

A、腹部肝 CT 扫描

B、血管造影

C、腹腔镜检查

D、手术探查

E、一月后重复 B 超及 AFP 等检查

B 型题

血清胆红素一分钟胆红素尿胆原尿胆红素 A 增高

增高+ +

B 增高正常 +——

C 增高

正常— + D 增高增高—

+++ E 增高增高 + —

140、肝硬化所致黄疸的实验室检查指标为:

141、胆道完全梗阻性黄疸的实验室检查指标为:

A、抗胆碱能药物

B、H2 受体拮抗剂

C、胃动力药(提高幽门张力,加速胃窦收缩)

D、制酸剂

E、质子泵抑制剂

142、奥美拉唑(洛赛克):

143、多潘立酮(吗丁啉)

A、应用阿托品

B、应用解磷定

C、阿托品加解磷定

D、应用强心剂

E、应用镇静剂

144、有机磷中毒,经治疗后仍有腺体分泌亢进症状,肺水肿:

145、有机磷中毒,经治疗后仍有骨骼肌兴奋症状,肌颤动: A、消瘦、乏力、腹胀B、贫血、出血倾向C、腹部蜘蛛痣 D、腹水伴脾肿大E、肝脏进行性肿大,肝区疼痛

146、肝硬化门脉高压最常见的临床表现:

147、肝癌常见的临床表现: A、间接胆红素增高,无贫血,肝功正常、尿胆原阴性 B、直接胆红素增高,肝功正常,尿胆红素阳性 C、间接胆红素增高、分血、尿胆红素阴性、网织细胞增多 D、间接胆红素增多,无贫血,尿胆红素阴性,肝功正常 E、间接胆红素增高,无贫血,肝功能不正常

148、溶血性黄疸可出现:

149、梗阻性黄疸可出现:

A、肝炎

B、肝癌

C、肝脓肿

D、脂肪肝

E、瘀血性肝肿大

150、男性,42 岁,右上腹肌紧张,肝脏肋下 2cm,质软,右肋间明显压痛,最可能诊断是:

151、女性,37 岁,查体,肝肋下 4cm,质中,边钝,有触痛,肝颈回流征+,诊断: A、血清淀粉酶B、血钙C、血清脂肪酶D、血清淀粉酶肌酐清除率比率 E、血清正铁血红蛋白

152、水肿型急性胰腺炎时升高,出血性坏死型胰腺炎可正常的是:

153、水肿型胰腺炎阴性,出血坏死型胰腺炎阳性的是: A、铜沉积于肝脏 B、抗线粒体抗体+ C、铁沉积于肝脏 D、肝纤维化 E、乙肝表面抗原+

154、血色病是因为:

155、肝豆状核变性是由于:

A、X 线钡餐检查、回盲部呈现 Stingsing(线样症)

B、X 线钡餐检查、回盲部呈现 Stierlin’sSing(跳跃症)

C、X 线钡餐检查、结肠边缘不规则,呈锯齿状

D、X 线钡餐检查、回盲部呈充盈缺损,肠腔狭窄

E、X 线钡餐检查、结肠袋消失,肠腔狭窄

158、159(缺)

A、胃得乐

B、三钾二橼络合铋

C、阿托品

D、法莫替丁

E、硫糖铝160、治疗十二指肠溃疡时应首选:

161、治疗胃溃疡并 HP 感染应首选:

A、CT 示胰腺钙化症

B、B 型超声示胆总管增宽及胆囊扩大

C、ERCP 示肝内胆管系统狭窄、胰管正常

D、尿胆原(+)、尿胆红素(+)ALT:

120u

E、血清淀粉酶 620u

162、胆总管结石并胆囊炎:

163、急性胰腺炎:

A、肌束震颤至全身抽搐、呼吸肌麻痹

B、头晕、共济失调、谵妄、昏迷

C、血压增高、心律失常

D、癫痫样抽搐,瞳孔不等大

E、瞳孔缩小,流涎、肺水肿164、有机磷中毒时烟碱样症状为:

165、有机磷中毒时毒蕈碱样症状为:

A、多为上腹正中或偏右节律性疼痛

B、多为剑突下正中或偏左节律性疼痛

C、上腹疼痛无典型节律性、呕吐多见

D、上腹持续性剧疼痛、放射至背后

E、右上腹节律性疼痛,夜间痛和背部疼痛多见且突出

166、十二指肠球部溃疡可出现:

167、幽门管溃疡可出现:

168、球后溃疡可出现:

169、胃溃疡可出现:

170、穿透性溃疡可出现

A、法莫替丁

B、丙谷胺

C、奥美拉唑

D、山莨菪碱

E、氢氧化铝

171、H2 受体拮抗剂是指:

172、泵阻滞剂(质子泵抑制剂)是指:

A、抗线粒体抗体

B、抗内因子抗体

C、G 细胞抗体

D、抗核抗体

E、抗平滑肌抗体

173、对原发性胆汁瘀积性肝硬化最有诊断价值的是:

174、胃体胃炎(A 型胃炎)合并恶性贫血时可能检出: A、肝炎后肝硬化

B、血吸虫性肝纤维化

C、酒精性肝硬化

D、原性性胆汁性肝硬化

E、瘀血性肝硬化

175、可么生羽毛样变性、坏死的是:

176、临床上以门脉高压为主要表现的是::

A、血甘油三酯

B、血淀粉酶

C、淀粉酶肌酐清除率比值

D、血脂肪酶

E、血钙

177、对急性胰腺炎晚期诊断最有帮助的是:

178、判断急性胰腺炎预后不良的指标为:

A、氢氧化铝凝胶

B、硫糖铝

C、法莫替丁

D、654-2

E、丙谷胺

179、男性、25 岁,近一个月常感觉上腹部疼痛,尤以空腹为甚,进食后缓解,X 线钡餐检查见球部有钡斑,最好选用哪种药物:

180、男性,50 岁,近几个月感上腹部疼痛,无明显规律性,伴腹泻,X 线钡餐透视见胃窦、胃体及胃底多个溃疡,BAO/MAO=65%,最好选用哪种药物:A、皮肤金黄色B、皮肤瘙痒C、皮肤色素沉着、蜘蛛痣 D、尿色加深、粪色黄E、酱油色尿

181、急性溶血性黄疸可出现:

182、胆道结石完全梗阻性黄疸可出现:

A、瞳也缩小

B、多汗

C、肌束震颤

D、肺水肿

E、共济失调

183、烟碱样症状为:

184、中枢神经系统症状为:

A、胃肠淤血、消化吸收障碍、菌群失调

B、肝细胞进行性或广泛性坏死

C、雌激素增多

D、雄激素增多

E、醛固酮增多

185、肝掌、蜘蛛痣是由于:

186、中毒性鼓肠是由于:

A、胃溃疡癌变

B、溃疡穿孔

C、幽门梗阻

D、慢性穿透

E、上消化道出血

187、胃溃疡患者上腹痛失去节律性,粪便隐血持续阳性,最应考虑:

188、十二指肠溃疡患者餐后上腹痛,有胃蠕动波、呕吐物含酸酵宿食物,无胆汁、最应考虑:

189、球后壁溃疡患者出现上腹及背部持续性疼痛,药物治疗无效,最应考虑:A、Brunner 腺B、主细胞C、壁细胞D、胃粘膜上皮细胞E、胃窦部 G 细胞

190、分泌胃酸的细胞是:

191、分泌胃泌素的细胞是:

192、分泌内因子的细胞是:

A、减少肠道氨气的形成和吸收

B、降血氨

C、纠正氨基酸代谢不平衡

D、补充脑部正常神经递质

E、防治脑水肿

193、肝性脑病,乳果糖的作用机制是:

194、肝性脑病,谷氨酸钠的作用机制是:

195、肝性脑病,左旋多巴的作用机制是:

A、胃糖膜不同程度炎性细胞浸润

B、胃固有腺体不同程度破坏性伴炎细胞浸润

C、胃粘膜 G 细胞增生

D、胃粘膜炎性细胞浸润,坏死组织及纤维组织

E、胃粘膜见印戒细胞呈巢状

196、慢性萎细胞性胃炎:

197、慢性浅表性胃炎:

A、疼痛—>进食疼痛,无间歇,无规律

B、进餐疼痛—>缓解—>进餐疼痛

C、疼痛—>进餐缓解、空腹及夜间疼痛

D、进餐疼痛—>呕吐

E、进餐—>夜间痛

198、球部溃疡:

199、胃溃疡:

200、胃癌;

四.泌尿内科试题

A题型

1、女性 25 岁,发烧伴血尿、尿频、尿急及腰痛三天,双眼睑稍浮肿,查体:心肺未见异常,腹软肝脾未及,两肾区叩痛阳性,尿常规:比重:1.016,蛋白+,RBC:++,WBC:+++,颗粒管型及细胞管型+,为明确诊断,下列哪一检查为首选?

A、查血白细胞总数及分类

B、泌尿系 B 超检查

C、腹部平片

D、泌尿系 X 线计算机扫描

E、中段尿培养及菌落计数

2、男,16 岁,头痛,乏力,两眼浮肿,尿少一周,血压(105/100mmHg),发病前十天,曾有感冒咽痛史,首先应采取哪种检查最有利于诊断?

A、查血白细胞总数及分类

B、查尿常规+比重

C、查空腹血尿素氮

D、查内生肌酐清除率

E、查血、尿渗透压

3、下列哪一项不是原发性肾病综合征 I 型的临床表现?

A、水肿

B、低蛋白血症

C、高脂血症

D、尿蛋白大于 3.5g/24 小时

E、尿中 C3 和巨球蛋白阳性

4、男,68 岁,因高血压 25 年,乏力 5 个月入院,入院后双下肢不自主运动,查体 BP:180/110mmHg,皮肤少许出血,心界左扩大,实验室检查:血Hb:78g/L,BUN:28.6mmol/L,血Ca:1.9mmol/L,P+++:2.8mmol/L,Na:135mmol/L,该患者下

肢不自主运动的最可能的原因是:

A、脑血管意外

B、帕金森氏病

C、高血钾症

D、低钙血症

E、尿毒症性神经病变

5、急性肾炎出现轻度贫血的最主要原因是:

A、缺铁

B、促红细胞生成素减少

C、水钠潴留、血液稀释 D 骨髓受抑制 E、红细胞寿命缩短

6、下列哪项是诊断急性肾功能衰竭最可靠的指标?

A、高钾血症

B、少尿

C、尿钠>20mmol/L

D、血尿素氮增高

E、血肌酐增高

7、长期血透病人的最主要的死亡原因是:

A、消化道出血

B、心血管合并症

C、神经系统合并症

D、透析肾病

E、感染

8、尿毒症时,下列哪项临床表现最主要是由于肾脏内分泌功能障碍所致? A、氨质血症 B、贫血 C、代谢性酸中毒 D、胃肠道症状 E、心包摩擦音

9、预防急性肾小球肾炎的最有效措施是:

A、加强营养

B、注意休息

C、保暧防潮

D、积极治疗链球菌感染

E、定期尿常规检查

10、女,58 岁,发热,腰痛 1 天,无尿路刺激征,体温 50 摄氏度,心肺阴性,腹软,脐右侧腹直肌外缘处压痛,左下肢皮肤有小破损一处,尿常规:蛋白+,红血球++,白血球++,尿亚硝酸盐还原试验,尿细菌增减为葡萄球菌,菌落为 1000 个/ml,临床诊断最应考虑为:

A、急性阑尾炎

B、急性膀胱炎

C、急性肾小球肾炎

D、急性肾炎

11、人体有两肾脏,由约多少万个肾单位组成? A、10 B、20 C、30

D、50

E、100

12、关于急性肾盂肾炎,下列哪项是错误的?

A、有尿液化

B、肾区啊击痛阳性

C、一般无寒战发热全身不适

13、女,23 岁,已婚,突发性尿频、尿痛、无发热,检查:双肾区无叩击痛,尿常规:尿蛋白—,脓球++,红细胞少许,中段尿培养为大肠杆菌,菌落数大于 10 个,最可能的诊断是:

A、急性肾炎发作

B、慢性肾炎急性发作

C、急性膀胱炎

D、泌尿系统结石并感染

E、泌尿结核

14、女,35 岁,浮肿、高血压三年,尿蛋白++,长期低盐饮食,并用双氢克尿噻利尿治疗,近一周尿量减少,厌食,恶心呕吐,意识障碍,血压:105/70mmHg,尿素氮:9.9mmol/L,血钾:3.7mmol/L ,血钠:

110mmol/L,下列哪种处理最为合适:

A、口服氯化钾

B、3%氯化钠 100ml 缓慢静脉滴注

C、冬眠灵 25mg 肌注

D、10%G 静脉滴注

15、下列哪项内分泌激素不是肾脏所分泌的?

A、肾素

B、前列腺素

C、1 羟化酶

D、促红细胞生成素

E、抗利尿激素

16、引起肾盂肾炎的致病菌中以下哪种细菌最多见?

A、大肠杆菌

B、副大肠杆菌

C、变形杆菌

D、肠形菌

E、金黄色葡萄球菌

17、男,76 岁,患糖尿病 25 年,两年前发生肾病综合症,两个月来病情加重,恶心,呕吐,口中异味感加重,并出现夜间轻度呼吸困难,每日尿量 1000—1500ml,查体:(200/90mmHg),Scr:760umol/L,BUN:

40mmol/L,Hb:68,首先的治疗方案是:

A、血液滤过

B、CAVH(持续性动脉血液滤过)

C、CAPD(持续性不卧床性腹膜透析)

D、肾移植

E、血液灌流

18、肾病综合症最重要的改变是:

A、高度浮肿

B、高血压

C、高脂血症

D、大量蛋白尿

E、低蛋白血症

19、导致肾盂肾炎最常见的感染途径是:

A、上行性

B、血源性

C、淋巴播散

D、邻近组织感染直接蔓延

20、女,30 岁,头晕,伴恶心、呕吐半个月,血压 150/105mmHg,Hb:60g/L,尿检查,比重 1.010,蛋白

++++,红血球++,白细胞+,蜡样管+,BUN84mmol/L,本例的诊断可能是? A、急进型高血压伴急性肾功能不全尿毒症期 B、慢性肾盂肾炎伴慢性肾功能不全氮质血症期 C、慢性肾小球肾炎伴慢性肾功能不全氮质血症期 D、慢性肾盂肾炎伴慢性肾功能不全尿毒症期 E、慢性肾小球肾炎伴慢性肾功能不全尿毒症期

21、与环磷酰胺毒性无关的是:

A、股骨头无菌性坏死

B、骨髓抑制

C、中毒性肝炎

D、性腺抑制

E、出血性膀胱炎

22、以下为患者 23 小时尿蛋白定量的化验值,哪种情况不属于蛋白尿? A、40mg/24 小时尿 B、160mg/24 小时尿 C、0.2g/24 小时尿 D、

0.25g/24 小时尿 E、0.3g/24 小时尿

23、临床上最常见的急性肾衰的病因是?

A、重症肾盂肾炎

B、急性间质性肾炎

C、大量出血

D、重症肾小球肾炎

E、急性肾小管坏死

24、有关慢性肾功能不全的病因和发病机理中下列哪一项是错误的? A、各种慢性肾脏疾病均可导致肾功能不全 B、蛋白质代谢产物均可导致肾功能不全C、肾功能不全发展至尿毒症的过程通常用”健存”肾单位来解释 D、矫枉失衡学说补充了尿毒症的发病机理 E、”健存”肾单位减少后所引起的肾小球毛细血管高灌流、高滤过状态可加速肾小球硬化

25、慢性肾盂肾炎的最早出现下列哪项异常?

A、血尿素氮

B、血肌酐

C、血β2 微球蛋白

D、尿β2 微球蛋白

E、内生肌酐清除率

26、肾小球源性血尿的主要特点是:

A、肉眼血尿

B、尿中红细胞大于 10 个/HPC

C、尿中红细胞小于 10 个/HPC D,以畸形红细胞为主 E、以正常红细胞为主

27、女,16 岁,三周前猩红热,三天来眼睑浮肿,血压(130/70mmHg),尿蛋白++,比重 1.028,镜下可见红细胞管型,血尿素氮 7.1mmol/L,血肌酐:

1.88mmol/L,治疗的最主要的措施是?

A、透析疗法

B、加强营养

C、休息并控制感染

D、激素

E、抗凝疗法

28、尿毒症时,高血钾最有效的治疗方法是:

A、输入碳酸氢钠

B、输入钙剂

C、输入高渗葡萄糖加胰岛素 E、进行血液透析 E、口服负型离子交换树脂

29、下列哪种药物造成肾小管尿液高渗状态,减少水、钠的重吸收而利尿? A、速尿 B、双氢克尿塞 C、安体舒通 D、安定 E、甘露醇

30、男,26 岁,一个月前患上感,近日浮肿,持续少尿,肾功能进行性恶化,血压 150/90mmHg,尿蛋白

+++,红细胞 15—20 个/HP,血红蛋白 10g/dl,肾病理特点最可能是? A、弥漫增殖性病变 B、系膜和基质增生成毛细血管袢的双轨现象 C、系膜增殖性病变 D、局灶节段性硬化 E、广泛肾小球腔内新月体形成

31、男,36 岁,轻度浮肿 3 年,2 周来恶心,乏力,血压:180/106mmHg,血红蛋白 50g/L,尿蛋白+,比重 1.010,为明确诊断,首先应检查:

A、骨髓涂片检查

B、血浆铁测定

C、血浆蛋白测定

D、血气分析

E、血尿素氮测定

32、下列哪组肾脏疾病在用了包括糖皮质激素在内的免疫抑制剂治疗后病情最可望得到改善? A、原发性肾小球肾病,CGN 高血压型,急进型肾炎 B、原发性肾小球肾病,CGN 肾病型,狼疮性肾炎 C、原发性肾小球肾病,急性肾炎,糖尿病肾病 D、高血压良性肾小动脉硬化,原发性肾小球肾病,慢性肾功能不全 E、原发性肾病、妊娠中毒症,急性肾功能衰竭

33、慢性肾盂肾炎的临床表现,下列哪项是错误的?

A、尿沉渣可见到成团的白细胞

B、在急性期内,中段尿定量细菌培养数超过 10 万/ML

C、尿浓缩功能减退

D、反复发作不易治愈者,常有尿路不通因素

E、不会发展成为高血压

34、肾病综合症最主要的诊断依据是:

A、高度水肿伴胸腹水

B、高脂血症,血浆白蛋白小于 30g/L,

C、尿蛋白

3.0g/日,血浆白蛋白小于 3.5g/L D、血β球蛋白相对增高 E、尿蛋白>3.5g/日,血浆白蛋白小于 3.0g/L

35、尿毒症患者必有的症状为:

A、少尿

B、高血压

C、恶心呕吐

D、分血

E、气喘

36、尿毒症最早和最常出现的症状是:

A、胃肠道症状

B、血液系统症状

C、泌尿道症状

D、神经系统症状

E、循环系统症状

37、下列哪一项不符合原发性肾病综合征 II 型的临床表现: A、常有程度不等的水肿 B、常有程度不等的高血压 C、常呈选择性蛋白尿 D、常有变形红细胞 E、常有程度不等的肾功能受损

38、男,40 岁,发热,流涕三天,继以恶心呕吐,少尿乏力,血压 160/100mmHg,血红蛋白:48g/L,B 超双肾长轴约 8cm,本例最可能诊断为:

A、急进型肾炎

B、急性肾功能衰竭

C、急性肾炎

D、慢性肾功能衰竭急性加剧

E、脱水引起氮质血症

39、男性,14 岁,全身浮肿两个月加重 1 周,尿蛋白++++,颗粒管型偶见,血浆白蛋白 20g/L,尿蛋白电泳白蛋白占 90%以上,最可能的诊断是:

A、肾小球肾病

B、急性肾炎

C、慢性肾炎

D、过敏性紫癜肾炎

E、系统性红斑狼疮性肾炎

40、高度浮肿,尿蛋白++++,Hb:140g/L,血浆白蛋白 14g/l,血压:90/60mmHg,首选的治疗是: A、肾上腺皮质激素 B、全血 C、血浆 D、速尿 E、环磷酰胺

41、尿毒症最早出现的症状是:

A、失眠、头痛乏力

B、咳嗽、气短、呼吸深长

C、厌食、恶心、呕吐

D、贫血

E、皮肤干燥瘙痒

42、女,57 岁,浮肿,间断血尿 18 年,查血压 190/120mmHg,化验 BUN:

27mmol/L,CO2CP:18mmol/l,血清钾:7.7mmol/L,血清钙:1.9mmol/L,应首先考虑以下哪种治疗?

A、输血

B、补钙

C、静滴葡萄糖加胰岛素

D、血透

E、静滴必需氨基酸

43、男,32 岁,平素健康,2 周前发热,咽痛,一周前出现少尿,全身水肿,血压 130/90mmHg,尿蛋白++,红细胞满视野,入院 6 周内尿量少于 20mmol/日,血尿素氨从 9mmol/l 升至 38mmol/L,肌酐从 177mmol/L 升至 884mmol/L,最可能的诊断是:

A、急性肾小管环死

B、急性肾小球肾炎

C、急进性肾小球肾炎

D、慢性肾小球肾炎急性发作

E、急性肾盂肾炎

44、男,30 岁,尿少,浮肿一个月,血压:120/82mmHg,尿蛋白+++、红细胞 2—5 个 HP,血浆白蛋白:20g/L,球蛋白:25g/l,胆固醇 7.2mmol/L,肌酐:176mmol/L,尿素氮:7.1mmol/L,首选的治疗是:

A、强的松

B、环磷酰胺

C、静脉输注白蛋白

D、静脉输注甘露醇

E、速尿

45、肾小球滤过膜的组成包括?

A、内皮细胞、系膜细胞、基膜

B、上皮细胞、系膜细胞、基膜

C、内皮细胞、上皮细胞、系膜细胞

D、内皮细胞、上皮细胞、基膜

E、上皮细胞、足突细胞、基膜

46、急性肾小球肾炎的临床表现,哪项是错误的?

A、大量尿蛋白

B、肾功能不全

C、早期少尿或无尿

D、镜下或肉眼血尿

E、早期出现贫血

47、以下哪项不支持肾盂肾炎的诊断?

A、尿蛋白量随病情变化而增加

B、尿中白细胞增多及白细胞管型

C、大量蛋白尿及低蛋白血症

D、肾小管功能损害为主

E、X 线或同位素肾图示两侧病变对称

49、男,18 岁,尿常规化验蛋白+++,红细胞蛋白 125g/L,血脂增高,尿蛋白成份分析以白蛋白为主,为选择性蛋白尿,最主要的是出于?

A、电荷屏障破坏

B、分子屏障破坏

C、电荷屏障、分子屏障均破坏

D、电子屏障、分子屏障均未破坏

50、关于肾小球毛细血管的滤过屏障,作用最重要的是:

A、内皮细胞层

B、基膜层

C、上皮细胞层

D、系膜细胞层

E、内透明层

51、女,32 岁,因双腰背酸痛,尿频、尿急、尿痛七天,就诊时:体温 39 度,双肾叩击痛阳性,尿检蛋白阳性,白细胞+++、红细胞+,最可能诊断为:

A、急性肾炎

B、急性膀胱炎

C、肾结核

D、急性肾盂肾炎

E、慢性肾盂肾炎

52、急性肾盂肾炎的典型临床表现:

A、发热、膀胱刺激症、蛋白尿

B、发热、膀胱刺激症、肾区叩痛、尿中白细胞增多

C、发热、膀胱刺激症、浮肿、尿中白细胞增高

D、发热、膀胱刺激症、浮肿,尿中白蛋白增高

53、男,46 岁,剧烈呕吐三天,无尿一天,CO2CP:32,BP:20/14Kpa,双下肢浮肿明显,以下检查哪种对诊断意义最大?

A、血浆白蛋白

B、BUN、SCR

C、血红蛋白

D、尿沉渣、蜡样管型

E、

24 小时尿蛋白定量

54、急性肾功能衰竭少尿期或无尿期,引死亡的最常见原因的是: A、氮质血症 B、高钾血症 C、出血倾向 D、高镁血症 E、低钠血症

55、尿毒症加重最常见的诱发因素是:

A、使用了肾毒性药物

B、代谢性酸中毒

C、心力衰竭

D、各种感染

E、高蛋白食物

56、下列哪项不是原发性肾病综合症 I 型的临床表现:

A、高度水肿

B、高蛋白血尿

C、高血脂

D、高粘滞血症

E、尿中纤维蛋白降解产生增高

57、男,18 岁,近年来腰酸乏力,双下肢出现凹陷性水肿,尿检查蛋白+++,白细胞 3/HP,红细胞 1/HP,血浆白蛋白 28g/L,尿蛋白定量 4g/日,肾功能正常,治疗首选以下哪种药?

A、强的松

B、潘生丁

C、地塞米松

D、环磷酰胺

E、甲基强的松龙

58、女,35 岁,5 年来间有低热腰痛,无尿频尿痛,近一年来夜尿增多,每晚达 3—4 次,BP160/90mmHg,尿蛋白+,RBC:1/HP,BUN:7mmol/L,尿培养二次阴性,诊断最可能是:

A、慢性肾炎普通型

B、慢性肾盂肾为

C、肾结核

D、慢性肾炎高血压型

E、隐匿型肾炎

59、慢性肾炎、肾功能不全已数十年,出现尿毒症、酸中毒入院,尿量显著减少,强效利尿效果不佳,血压 180/112mmHg,血尿素氮:30mmol/L,CO2CP:30,给予小剂量碳酸氢钠静滴后,出现呼吸困难,心率

120 次每分,肺底出现少许水泡音,此时进一步处理最应该的是: A、改用口服碱性药物+强心、降压药 B、改用乳酸钠静滴+强心药、降压药 C、改用利尿合剂+强心、降压药 D、强心降压处理后行血液透析 E、用 THAM+强心、降压药

60、急性肾炎的四联疗法包括:

A、强的松+环磷酰胺+消炎+抗凝

B、强的松+环磷酰胺+尿激酶+潘生丁

C、强的松+环磷酰胺+抗凝+潘生丁

D、强的松+环磷酰胺+抗凝+尿激酶

E、强的松+环磷酰胺+抗凝+雷公藤

61、慢性肾盂肾炎史 20 年,三年来出现慢性肾功能不全,尿少三天入院,肾小球滤过率 0.08ml/S,血肌酐 792mmol/L,其最佳治疗方法是:

A、透析

B、饮食治疗

C、氨基酸疗法

D、氧化淀粉治疗

E、强力利尿治疗

62、原发性肾病综合症 II 型经激素治疗后完全缓解,半年又复发,应首选下列哪一治疗方案? A、消炎痛+雷公藤+潘生丁 B、潘生丁+消炎痛+环磷酰胺C、强的松+消炎痛+肝素 D、强的松+消炎痛+潘丁生 E、强的松+环磷酰胺+潘生丁

63、肾盂肾炎最常见的感染途径是:

A、血行感染

B、淋巴道感染

C、上行感染

D、直接感染

E、器械感染

64、下列哪项是诊断急性肾衰竭可靠的指标?

A、高钠血症

B、高钾血症

C、少尿

D、尿素氮突然升高

E、肌酐突然增高

65、男性,35 岁,已确诊为尿毒症晚期;血压 24/14KPA,心率 130 次每分,高度浮肿,24 小时尿量 400 毫升,下列哪项治疗最有效?

A、50%葡萄糖加西地兰 0.4mg 静注

B、补充碱剂

C、使用大量利尿剂

D、血液净化治疗

E、强心剂加利尿剂

66、选择性蛋白尿的特点是:

A、24 小时尿蛋白量超过 3.5g

B、多为大分子蛋白尿

C、多为球蛋白

D、以白蛋白为主的蛋白尿

E、24 小时尿蛋白不少于 150mg

67、急性肾炎的最主要的病理特点是:

A、弥漫增生性膜肾小球肾炎

B、系膜细胞增生性肾小球肾炎

C、弥漫性毛细血管内肾肾炎

D、膜增生性肾炎

E、膜性肾炎

68、引起急性肾小球肾炎的最常见的病原菌是:

A、溶血性链球菌

B、草绿色链球菌

C、葡萄球菌

D、A 型溶血性链球菌

E、血菌

69、以下哪项对急、慢性肾功能衰竭的鉴别最有助?

A、肾影大小

B、代谢性酸中毒

C、肾滤过功能

D、血电解质

E、尿蛋白电泳

70、下列哪项检查最有助于上、下尿路感染的鉴别?

A、尿沉渣涂片不染到细菌

B、尿沉渣镜检有透明管型 3—5 高倍

C、尿沉渣抗体包裹细菌阳性

D、中段尿菌落计数大于 105

E、一小时尿白细胞大于30 万

71、女性,50 岁,畏寒,高热 1 天,体温达 40 度,右肾区轻叩痛,尿比重 1。025,尿蛋白++,WBC++,颗粒管型 1 个/HP,临床最可能的诊断是:

A、急性膀胱炎

B、慢性肾炎合并感染

C、急性肾盂肾炎

D、慢性肾盂肾炎急性发作

E、急性肾炎

72、急性肾功能衰竭高钾血症的发生不受下列哪个因素影响? A、感染 B、酸中毒 C、水钠潴留 D、外伤 E、输入库存血

73、男,15 岁,少尿、浮肿 1 周,气促不能平卧,伴有粉红色泡沫痰 1 天,全身浮肿明显,端坐呼吸,心率 120 次每分,两肺底湿罗音,BP:22.7/14.7KPA,尿比重 1.025.尿常规:蛋白+++、RBC+++,颗粒管型+, BIN18mmol/L,SCR:320mmol/L,临床最可能的诊断是:

A、急性肾炎的合并心衰

B、慢性肾炎合并心衰

C、高血压心脏病合并心衰

D、原发性心肌病合并心衰

E、肾小球肾病合并心衰

74、静滴注先锋霉素治疗蜂窝织炎,二天后出现肉眼血尿,少尿及肾功能急剧恶化,下列哪项不支持药物性急性间质性肾炎?

A、发热、关节痛

B、尿中嗜酸细胞

C、尿红细胞管型

D、停药后肾功能逐渐恢复

E、强的松治疗可促进肾功能恢复

75、以下哪项不是急性肾衰的透析指征?

A、无尿一天

B、CO2CP:12mmol/L

C、血钾 6.8mmol/L

D、血尿素氮:25mmol/l

E、血肌酐:503mmol/L

76、尿素症酸中毒时?

A、血钙总量正常,游离钙减少

B、血钙总量正常,游离钙增加

C、血钙总量降低,游离钙相对增加

D、血钙总量降低,游离钙更低

E、血钙总量降低,游离钙减少

77、女,65 岁,间断腰酸、尿频 30 余年,无明显尿痛,近 10 年高血压,查体:BP:160/90mmHg,眼底动脉变细,尿常规:蛋白+,RBC1—3 个/HP,WBC:10/HP,肾 B 超:右肾:11*5*3cm,左肾:7*3*2.5cm,左肾表面高低不平,最可诊断为:

A、慢性肾小球肾炎

B、高血压病

C、先天性肾发育不良

D、慢性肾盂肾炎

E、左肾动脉狭窄

78、女,25 岁,突发寒战,高热,腰痛、尿频、尿痛一周,T39.5 度,两侧肋腰点压痛,普通尿培养阴性,应首选哪项检查?

A、腹部平片

B、静脉肾盂造影

C、肾图

D、尿高渗性培养

E、肾活检

79、慢性肾功能不全患者入院后二周出现发热、咳嗽,首先的抗生素应是: A、丁胺卡那霉素 B、四环素 C、链霉素 D、青霉素 E、庆大霉素

80、男,14 岁,全身浮肿伴胸、腹腔积液一月,血压 16/10KPA,尿蛋白++++,白细胞 0—1/HP,血浆白蛋白 15g/L,球蛋白 20g/L,血尿素氮 3.6mmol/L,经严格限制纳盐,并注射大量速尿,尿量增大,但二天后尿量骤减,精神萎糜,四肢冷,血压(60/40mmHg),体温 36 度,尿比重:1.028,血尿素氮:10.7mmol/L, 白细胞计数正常,诊断首先应考虑:

A、急性肾小球肾炎并发肾功能不全

B、慢性肾小球肾炎并发肾功能不全

C、微小病变肾病并发继发感染

D、微小病变肾病并发低血容量性休克

E、急进性肾小球

肾炎

81、男,25 岁,腹痛、腹泻三天,曾用庆大霉素和补液治疗,继之少尿 300ml 每日,面部及双下肢浮肿, BP:185/108mmHg,心界扩大,化验:Hb:60g/L,尿蛋白+,红细胞 3—5/HP,BUN:35mmol/L,SCR:824mmol/L,血磷 2.4mmol/L,最可能的诊断是:

A、急进性肾炎

B、急性肾炎

C、急性肾炎

D、慢性肾炎

E、庆大霉素所致的急性肾小管坏死

82、女,32 岁,反复尿频、尿急一年,偶有腰痛和高热,尿培养曾有大肠杆菌,经治疗后症状消失,尿白

细胞持续存在++++,肾功能正常,为明确反复发作的原因应做下列哪项检查最为适应? A、肾脏 CT B、静脉肾造影 C、同位素肾图 D、尿培养 E、肾活检

83、少尿,伴浮肿三天,气急一天,尿渗透压 300,尿常规:蛋白:+,红细胞+,血肌酐:75.4mmol/L,血磷:1.8mmol/L,应首选诊断为:

A、急性肾小球肾炎

B、慢性肾小球肾炎

C、急性肾盂肾炎

D、急性肾功能衰竭

E、慢性肾功能不全尿毒症

84、男,35 岁,浮肿二年,近二周头晕,乏力,呕吐,尿少,BP:150/105mmHg,HB:(缺)

85、上尿路与下尿路的解剖分界在

A、肾盂和输尿管的交界处

B、膀胱三角

C、膀胱颈处

D、输尿管与膀胱交界处

E、膀胱与尿道

86、下列哪项肾盂肾炎实验室检查标准是错误的?

A、尿含菌数大于 105/ml 有诊断意义

B、尿中粪链球菌数在 1000—10000/ml 之间者,无诊断意义

C、尿白细胞大于 30 万个每小时为阳性

D、尿白细胞数小于 20 万个每小时属于正常范围

E、尿红细胞数大于 10 万个每小时为阳性

87、男性,24 岁,发作性肉眼血尿 3—4 年,尿蛋白+红细胞数 1—5 个/HP,WBC:2 个/HP,检查:血压:

110/70mmHg,无浮肿,尿培养阴性,IVP 造影正常,BUN、SCR 正常,诊断最可能是: A、肾结核 B、肾结石 C、慢性肾盂肾炎 D、隐匿性肾炎 E、急性肾炎

88、急性肾功能衰竭少尿期最常见的死亡原因是: A、感染 B、无尿 C、休克 D、高血钾 E、酸中毒

89、男,20 岁,全身水肿,反复发作二年,有时高血压(160/100mmHg),现血压:136/84mmHg,全身高度肿,24 小时尿蛋白 3.8 克,血清胆固醇:250mg/DL,尿沉渣细胞 10—15 个/高倍视野,颗粒管型+、血浆蛋白 A/G=2.3/2.8,最佳诊断为:

A、肾病综合症 II 型

B、慢性肾炎普通型

C、慢性肾炎高血压型

D、慢性肾炎急性发作

E、肾病综合征 I 型

90、女,25 岁,近年反复尿频尿急,伴右腰阵发性剧痛,且放射至下腹部,四环素治疗未能根治,尿涂片发现革兰氏阴性杆菌,为明确诊断,哪项检查最有意义?

A、尿高渗培养

B、肌酐清除率

C、膀胱镜检

D、静脉肾盂造影

E、尿结核菌造影

91、尿毒症代谢性酸中毒,原因有下列各项,但除外?

A、酸性代谢产物潴留

B、肾小管生成氨、泌氨功能减退

C、呼吸功能不全

D、肾小管加收重碳酸盐减少

E、碱性肠丢失

92 患者,男性 26 岁,一个月前低热:37.3—37.6 度左右,伴关节肿痛,近半个月来,双下肢水肿,眼睑水肿,来院就诊,查体:心肺阴性,腹部有轻微抵抗,轻度压痛及反跳痛,腹水症双下肢可凹性水肿,实验室检查尿蛋白:大于 500mg/24 小时,SCR:300mmol/L,BUN:21.4mmol/L,B 超双肾明确诊断患者需进一步检查:应首选:

A、反复测定内生肌酐清除率

B、腹腔穿刺

C、肾活检

D、骨髓穿刺

E、静脉肾盂造影

93、男,15 岁,少尿、浮肿一周,气促,不能平卧,伴粉红色泡沫痰一天,心率 120 次每分,两肺底湿罗音,BP:168/100mmHg,尿比重:1.025,尿常规蛋白+++、RBC:+++,颗粒管型++,BUN:10mmol/L,SCR:

136mmol/L,为治心力衰竭,最好选用哪种利尿剂:

A、甘露醇

B、低分子右旋糖酐

C、氨苯蝶啶

D、速尿

E、安体舒通

94、以下哪项最符合尿毒症血液系统表现:

A、低色素、小细胞分血

B、贫血常为中、重并且难纠正

C、网织红细胞减少

D、白细胞增高

E、血小板减少

95、男,52 岁,三周前曾有疥疮伴感染,三天来出现少尿,浮肿,血压:

150/90mmHg,尿比重 1.018,蛋白++,红细胞 30—40,白细胞 4—8 个/HP,血红蛋白:120g/L,诊断应首先考虑:

A、急性肾小球肾炎

B、慢性肾小球急性发作

C、急性肾盂肾炎

D、IgA 肾病

E、慢性肾小球肾炎普通型

96、关于尿毒症造血系统的改变下列哪项是错误的?

A、贫血是必有的症状

B、贫血与促红细胞生成素减少

C、贫血与造血原料的缺乏

D、贫血可用反复输血得到纠正

E、贫血与红细胞寿命缩短有关97、尿毒症必有的临床表现是:A、低钠血症 B、高钾血症 C、脱水 D、高度贫血 E、贫血

98、下列哪项不是尿毒症时引起血钙降低的原因 A、磷从肠道排时与钙结合限制了钙的吸收 B、低蛋白血症结合钙减少

C、厌食,使从食物中摄入的钙减少

D、甲状旁腺功能亢进

E、1.25 二羟 D3 生成障碍

99、女,40 岁,反复发生尿频、尿急、尿痛三年,低热,血压:105/75mmHg,尿蛋白阳性,脓细胞++、红细胞+++,尿抗酸杆菌+,普通尿增减选择哪种方案治疗?

A、静滴抗球菌药物

B、静滴抗杆菌药物

C、小剂量抗生素,联合交替使用

D、抗结核治疗

E、合用止血剂

100、男,21 岁,以双侧凹陷性水肿一月余入院,血浆蛋白 50g/L(A:28g/L,G22g/L),尿检查蛋白++, FDP+,尿 G3+,最可能诊断为:

A、急性肾炎

B、肾病综合症 I 型

C、肾病综合症 II 型

D、慢性肾盂肾炎

E、急性肾盂肾炎

101、男,18 岁,明显浮肿两周,尿量持续减少,约 300ml/日左右,BP:

150/120mmHg,尿蛋白++,尿 RBC+++,为进一步确诊,以下哪项检查最有意义:A、PSP 试验 B、CO2CP C、尿蛋白定量 D、观察尿 RBC 形态 E、尿液 PH

102、32 岁,患者,高血压浮肿已多年,因嗜睡伴恶心呕吐一周入院。体检:BP:185/108mmHg,心脏向左扩大,腹水征+,下肢水肿,尿常规:尿蛋白+、RBC+,WBC+,管型++,SCR:495mmol/L,CO2CP:20.8mmol/L,血钾:2.5mmol/L,血钙:1.7mmol/L,EKG 示心率 100 次每分,频发室早经治疗无效,病人突然意识丧失,其死亡原因最可能是:A、酸中毒 B、低钙 C、溶血 D、低钾 E、心功能不全

103、男,14 岁,反复浮肿、少尿二年多,又发浮肿、少尿一个月,查尿常规蛋白++++,定量:6g/24H,血浆白蛋白:25g/L,胆固醇 400mg/dl,经强的松治疗十天后,发病情仍无好转,此时应采取下列哪项措施最为适宜?

A、停用强的松

B、改用地塞米松 C 继续用原剂量强的松 D、加用氮芥 E、加用环磷酰胺

104、女性,26 岁,头痛,腰酸,口干,胸闷、心悸、少尿三天,浮肿外观,表情淡漠,血压 80/60mmHg,心率 120 次每分,腹胀叩诊浊音,双下肢凹陷性水肿胸腔中等量积液,血红蛋白 140g/l,尿蛋白++++,白细胞 4/hP,血浆白蛋白 14g/L,首先应立即处理:

A、5%GS+间羟胺

B、50%GS+西地兰

C、50%GS+速尿

D、输白蛋白

E、放胸腹水

105、女,25 岁,腰酸,阵发性下腹部疼痛,尿频、尿急、尿痛伴发冷发热一天,尿蛋白+,白细胞 10/hp,红细胞 12/HP,诊断最可能是:

A、急性膀胱炎

B、尿道综合征

C、泌尿系结石

D、急性肾盂肾炎

E、急性肾炎

106、男,40 岁,面部及双下肢间歇性轻度浮肿三年,BP:150/90mmHg,尿蛋白++,RBC:4—6 个/HP,颗粒管型 1—2 个/HP,BUN:11mmol/L,诊断最可能为?

A、慢性肾炎普通型

B、慢性肾炎高血压型

C、急性肾小球肾炎

D、慢性肾炎肾病型

E、慢性肾盂肾炎

107、男,患者 40 岁,呼吸深快,T37 度,BP:180/110mmHg,肺底部有湿性罗音,有心包摩擦音、肝脾不大,双下肢轻度浮肿,Hb:60g/L,尿比重:1.012,尿蛋白+,进一步做下列哪项检查,对确诊最有意义? A、血气分析 B、胸片C、血肌酐 D、心电图 E、眼底

108、男,35 岁,诊断慢性肾炎,尿毒症近一周尿少,查:Hb:60g/L,BP:

150/90mmHg,心率:40 次每分,心律齐,尿蛋白++,RBC:4—5/HP,EKG:P —R 间期 0.22,QRS 波 0.12,应作下列哪项紧急检查? A、心脏 B 超 B、血清电解质 C、SCR D、BUN E、二氧化碳结合力

109、慢性肾小球肾炎引起的水肿的机理最主要的是:

A、低蛋白血症

B、心力衰竭

C、醛固酮增多

D、全身毛细血管通透性增加

E、肾小球滤过率下降

110、女性,28 岁,反复浮肿 7 年,近二年来血压明显升高,血 BUN30mmol/L 其血压升高的最主要原因是: A、肾素活性增高 B、前列腺素分泌减少 C、肾动脉狭窄 D、全身小动脉硬化 E、水钠潴留

111、某病人,夜尿多,Hb:68g/L,BUN:21.3mmol/L,血肌酐:484mmol/L,动脉血气分析:HCO3:8mmol/L,在治疗过程中出现手足抽搦,最可能的是:

A、大量利尿

B、严重腹泻

C、血磷增高

D、血浆白蛋白降低

E、补碱纠正酸中毒

112、男,25 岁,三年来反复血尿,偶见红细胞管型,尿蛋白定量:0.9g/24 小时,血肌酐:97mmol/L,尿素 N:5.4mmol/L,为明确诊断,最有价值的进一步检查是:

A、尿找抗结核杆菌

B、腹部平片

C、逆行肾盂造影

D、肾活检

E、中段尿培养

113、男,17 岁,浮肿,24 小时尿蛋白量 10 克,血浆白蛋白 15g/L,血胆固醇:600mg%,经两周足量强的松治疗后,症状明显改善,最可能为:

A、膜性肾病

B、系膜增生性肾炎

C、局灶性肾小球硬化

D、慢性进展性肾小球肾炎

E、光镜下肾小球基本正常

114、女,40 岁,间断腰痛,尿频 5 年,2 年来夜尿 2—3 次,近两周来头痛,头晕,颜面下肢轻水肿,BP:

170/110mmHg,尿蛋白+,尿白细胞 20—30/HP,尿 RBC:0—2/HP,颗粒管型偶见,尿比重 1.011,以下诊断哪项正确?

A、慢性肾普通型

B、慢性肾炎高血压型

C、慢性肾炎急性发作

D、慢性肾盂肾炎

115、男性患者 30 岁,临床诊断肾病综合征,肾活检病理提示早期间质肾病,经肾上腺皮质激素治疗,症状有所好转,但此后突然出现明显尿痛,尿常规:RBC20—30/HP,尿蛋白量明显培养加,血肌酐上升,B 超检查示左侧肾脏体较前增大,以下诊断哪一个是首先考虑的?

A、泌尿系感染

B、急性肾功能衰竭

C、输尿结石肾绞痛

D、肾静脉血栓形成

E、激素治疗用量不足

116、肾小球疾病有 C3 减少时,一般不考虑下列哪个病?

A、急性链球菌感染后肾炎

B、急进型肾小球肾炎 II 型

C、系膜毛细血管增生性肾炎

D、肾淀粉样变

E、狼疮肾

117、基本肾单位应包括:

A、肾小球+肾小球囊+肾小管

B、肾小球+集合管

C、肾小球+肾小管+集合管

D、肾不球毛细血管丛

E、入球动脉+出球动脉+直血管

118、尿毒症患者发生肾性骨病的最主要原因是:

A、尿钙排泄增多

B、长期代谢性酸中毒

C、低蛋白血症及营养不良

D、活性维生素 D 合成障碍

E、继发性甲状腺功能亢进

119、男,17 岁,全身浮肿,尿少两周,BP(100/80mmHg),HB:14g/L,尿蛋白+++,24 小时尿定量 4.0g, BUN6mmol/L,血细胞正常,尿阴性,诊断肾病综合症 I 型,基层医院给速尿后尿量增加,一周后,突然腰痛,尿量明显减少,BUN 升至 18mmol/L,尿镜检红细胞满视野,最可能的诊断为:

A、泌尿系感染

B、原发性腹膜炎

C、肾静脉血栓

D、肾前性氮质血栓

E、Fanconi’s综合症

120、女,28 岁,近三年反复出现尿频、尿急、畏寒。发热、伴左腰部阵发性剧痛且放射至下腹部,曾有全程血尿二次,尿涂片发现革兰氏阴性杆菌,为确诊下列哪项检查有意义?

A、尿细菌培养

B、膀胱镜检查

C、尿路 X 线造影

D、尿结核杆菌无病呻吟养

E、尿培养菌落数

121、男,35 岁,头昏、乏力、纳差半年,近十天出现呕吐,厌食,血压:150/90mmHg,无水肿,血红蛋白 8.0G/L,尿蛋白+、颗粒管型 0—2 个/HP,血白蛋白

3.0g/dl,球蛋白 2.5g/dl,BUN:12.5mmol/L,SCR:

264mmol/L,血钠 125mmol/L,近三个月低盐,低蛋白饮食,目前应首选给予: A、低蛋白,以动物蛋白为主,不限制钠盐摄入 B、低蛋白,以植物蛋白为主,限制钠摄入 C、不限制蛋白,多以动物蛋白,低盐饮食 D、不限制蛋白,多以植物蛋白,高钠饮食 E、低蛋白,多以动物蛋白,高钠饮食

122、女性,寒战高热一天,体温 40 度,右侧肾区叩痛阳性,尿常规比重 1.025,白细胞+++,白细胞管型

0—1/HP,红细胞+,蛋白+,为了尽快选用有效的治疗药物,最应采取的何种检查? A、新鲜尿沉渣涂片染色找细菌 B、亚硝酸盐还原试验 C、尿细胞计数 D、尿细菌增减+药敏试验 E、尿细菌普通培养+菌落计数

123、男,35 岁,反复浮肿腰痛 12 年,尿比重 1.014,HP:70g/L,X 线照片有明显骨质疏松,其最佳治疗应是:

A、利尿

B、补钙

C、口服氢氧化铝

D、肌注苯丙酸诺龙

E、用活性维生素

124、女性,25 岁,妊娠 7 个月,突然腰痛,寒战发热,为明确诊断,首先进行哪项检查? A、血常规 B、尿常规 C、血培养 D、肾脏 B 超 E、尿培养

125、男,68 岁,高血压 20 年,头晕,腰酸,夜尿多,颜面浮肿一周,血压:220/110mmol/L,心界向左下扩大,眼底示三级视网膜动脉硬化,尿蛋白+,白细胞 10/HP,红细胞 8/HP,尿蛋白定量 1g/日,下列哪项治疗原则最主要的?

A、应用降压药物缓慢降压以防肾血流量骤降

B、应用激素治疗控制蛋白尿

C、应用利尿剂消肿和降压

D、应用抗菌素控制尿路感染

E、应用补肾药物治疗腰酸夜尿

126、、女,29 岁,妊娠 4 个月,因尿痛、尿频、尿急二天来诊,无发热与血尿,尿常尿蛋白+,沉渣镜检满视野白细胞,首选哪种治疗?

A、青霉素 80 万肌注每日三次

B、红霉素 0.4 每日两次

C、庆大霉素 8 万肌注每日两次

D、氟哌酸 0.2,每日三次

E、氨苄青霉素 1.0 肌注每日二次

127、一昏迷尿少患者,重度贫血,呼吸深长,BP:(180/120mmHg),有心包摩擦音,肝脾不大下肢浮肿,下述检测哪项对抢救最有意义?

A、血气分析

B、血糖测定

C、脑 CT

D、测钾、钠、氯

E、测血尿素氮、血肌酐

128、有关肾性水肿的发生机理,下列哪项是错误的? A、水肿是肾脏病必备的表现 B、水肿程度与肾脏病严重程度不完全一致

C、急性肾炎水肿的机理是低蛋白血症

D、肾病综合症水肿的主要原因是大量的蛋白尿致低蛋白血症

E、肾脏病变时可有继发性醛固酮增多,加重水肺129、尿毒症患者下列哪种情况最危险:

A、血清钙 1.8mmol/L

B、二氧化碳结合力 15mmol/L

C、血尿素氮 24mmol/L

D、血肌酐 374mmol/L

E、血清钾

130、下列哪项对肾盂肾炎的诊断有意义:

A、尿蛋白

B、尿中白细胞增多

C、红细胞增多

D、真性菌尿

E、尿中可见白细胞管型

131、男,25 岁,上感后 4 天发现肉眼血尿,血尿 6 天后消失,无腰痛,无发热,无浮肿等症状,血压正常,最佳的诊断考虑为:

A、急性肾炎

B、IgA 肾病

C、紫癜性肾炎

D、尿路结石

E、慢性肾炎

132、女、33 岁,反复关节痛,面部红斑 12 年,全身浮肿 4 个月,BP:

150/90mmHg,化验 Hb:77g/L,尿蛋白++++,尿红细胞+,血白蛋白 27g/L,尿蛋白 33 克/L,血浆蛋白电泳 Y 球蛋白 33%,HBsAg、HBeAg 均阳性,ANA 阳性,SDNA 阳性,胸片:心脏扩大,最可能的诊断是:

A、原发性肾病综合症

B、乙肝相关性肾炎

C、狼疮性肾炎

D、风湿热伴热性蛋白尿

E、慢性肾炎、普通型

133、肾病综合症的三高一低表现可见下列哪种疾病,但除外?

A、狼疮性肾炎

B、糖尿病肾病

C、慢性肾盂肾炎、尿毒症

D、急性肾炎

E、过敏性紫癜性肾炎

134、男性,25 岁,尿浮肿 2 月,血压 120/80mmHg,尿比重 1.018,尿蛋白++++,尿红细胞 0—1/HP,尿蛋白成分分析为选择性蛋白尿,血浆总蛋白为

40g/L,白蛋白 15g/L,血肌酐 99mmol/L,血尿素氮 10mmol/L,为消除病人浮肿首选?

A、白蛋白

B、血浆

C、速尿

D、强的松

E、甘露醇

135、患者,男性,20 岁,发烧一日,后出现肉眼血尿,无明显尿痛,略有尿频,第三日尿常规 RBC:20

—30/HP,WBC:1—2/HP,尿蛋白+,B 超示双肾大小形态正常,查体:BP:

130/90mmHg,双下肢微肿,首先考虑下述哪一诊断?

A、泌尿系感染

B、急性肾小球肾炎

C、IgA 肾病

D、泌尿系结石

E、膀胱肿瘤

136、男性,28 岁,半年来常头痛乏力,三天来视力模糊,体检:脸轻肿,血压:180/105mmHg,尿蛋白++,尿沉渣 RBC20/HP,眼底检查:视网膜动脉痉挛,黄斑部有渗出物与出血,视乳头无水肿,B 超:两肾缩小,首选哪个诊断?A、原发性高血压肾损伤 B、恶性高血压肾损害 C、慢性肾炎高血压型 D、急性肾炎 E、肾动脉狭窄

137、最能作为慢性肾功能衰竭和急性肾功能衰竭的鉴别的依据的是哪一项?A、血 BUN B、蛋白尿与低白蛋白血症 C、贫血严重 D、低血钙症和高磷血症 E、肾脏体积缩小

138、女性,50 岁,不洁饮食后出现腹痛、腹泻二天,大便 10—30 次每天,伴呕吐,曾补液庆大霉素治疗,未用利尿剂,治疗三天后出现少尿,30ml/日,双下肢浮肿化验:BUN28mmol/L,SCR:556mmol/L,应首先进行下列哪项检查?

A、测尿钠

B、静脉肾盂造影或肾脏 B 超

C、肌酐清除率

D、尿沉渣

E、以上都不是

139、指出以下哪种方鉴别上下尿路感染准确率最高?

A、尿细菌培养,尿含菌数大于 10/ml

B、发烧

C、尿路刺激症

D、检查尿沉渣

E、膀胱冲洗后细菌尿培养

B 型题:

A、上皮细胞足突融合与消失

B、膜细胞增生基质增多

C、内皮细胞增生

D、广泛肾小球腔内新月体形成

E、肾间质纤维化

140、微小的主要病理改变是:

141、急进性肾小球肾为炎的病理改变是:

A、厌食、腹部不适、恶心、呕吐、腹泻

B、精神萎糜,疲乏、头晕、头痛、记忆减退等

C、贫血

D、容易出现脱水和水肿的现象

E、可有心包摩擦音或心包积液症

142、尿毒症中最早和最常见的症状是:

143、尿毒症病人必有的症状是:

A、颗粒管型

B、蜡样管型

C、脂肪管型

D、红细胞管型

E、白细胞管型

144、急性肾小球肾炎可出现?

145、急性肾盂肾炎可出现?

A、抗菌药物治疗两周,停药观察

B、低剂量药物抗菌疗法

C、系统抗菌药综合治疗

D、数种抗菌药分组轮流使用

E、单剂抗菌药治疗

146、男性,85 岁,二年来常有尿感发作,血压 180/105mmHg,双肾区叩痛,尿脓细胞++,尿培养变形杆菌大于 100000/ml.

147、女性,28 岁,寒战高热三天,伴右腰痛,尿白细胞++,蛋白+,尿培养肠球菌>1000/ml A、强的松+环磷酰胺 B、强的松+青霉素 C、环磷酰胺+青霉素 D、青霉素+对症处理 E、强的松+对症处理

148、男性,18 岁,浮肿,大量蛋白尿,低蛋白血症及高脂血症一周,用强的松 60mg/日,治疗一个月后,浮肿消退,尿蛋白,20 天内强的松减至 30mg/日,一周后尿蛋白又增至+++,此时应采取以上哪组治疗方案最优?

149、男,14 岁,发咽痛,半月后尿少,两眼睑浮肿,腰酸、乏力,查尿蛋白++,镜检:WBC:3/HP,RBC 满布视野,红细胞管型 5—10 个/HP,血 BUN:

3.8mmol/L,SCR:82mmol/L,此时应采取哪组治疗方案最为恰当?

A、肾病综合症

B、周期性麻痹

C、低钙高磷血症

D、膀胱刺激症

E、大量蛋白尿

150、急性肾盂肾炎最常见?

151、尿毒症最常见?

A、急性肾小管坏死

B、肾后性少尿

C、急性间质性肾炎

D、肾前性氮质血症

E、尿潴留

152、少尿三天,B 超双肾均正常,SCR:700mmol/l,BUN:24mmol/L,尿钠:45?153、腹泻二天,少尿一天,导尿时 20ml,SCR:264umol/l,BUN:28.6mmol/L,尿钠 10?

154、排尿迟缓 5 年,少尿一天,B 超示双肾盂积水,SCR:528umol/L,BUN:

28.5mmol/L A、大剂量甲基强的松龙冲击加环磷酰胺+血浆置换

B、强的松 1mg/Kg,环磷酰胺

C、强的松 1mg/d

D、强的松 30mg/d+环孢素 A

E、抗菌素 2 周+卧床休息+对症处理

155、急进性肾小球肾炎

156、感染的急性肾小球肾炎

157、微小病变性肾病综合症

A、远端肾小管酸中毒

B、近端型肾小管酸中毒

C、IV 型肾小管酸中毒

D、尿毒症性酸中毒

E、酮症酸中毒上述哪种代谢性酸中毒的表现应当是?158、血 PH7.353,血 HCO3:16mmol/L,SCR:160umol/L,血钾:3.0mmol/L,FEHCO3:15%,血糖:104mmol/L,

159、血 PH7.353,血 HCO3:16mmol/L,SCR:186umol/L,血钾:3.8mmol/L,FEHCO3:31.5%,血糖:19.7mmol/L,

160、血 PH7.325,血 HCO3:31.2mmol/L,SCR:780umol/L,血钾:5.5mmol/L,FEHCO3:35.5%,血糖:7.8/L,肾功能衰竭病人(SCR<10ml/min)时用药的选择是:

A、不应用

B、剂量不改变

C、需要增加剂量

D、可减量采用

E、慎用161、四环素

162、氨基糖甙类药物

A、钠型阳离子交换树脂

B、碳酸钙

C、叶酸制剂

D、碳酸氢钠

E、安体舒通上述药物适用于下列哪种疾病?

163、慢性肾功能不全,高钾血症

164、慢性肾功能不全,高磷血症 A、乙酰唑胺 B、噻嗪类 C、安体舒通165、髓袢升支皮质部?

166、髓袢升支粗段

A、肾病综合症

B、肾结石

C、肾动脉硬化

D、肾盂肾炎

E、慢性肾炎高血压型

167、尿常规,尿蛋白+,尿沉渣:RBC:8—10 个/HP,WBC:20—30 个/HP,透明管型 0—1/HP,应诊断为

168,尿常规:尿蛋白+++,尿沉渣:RBC4—6wh ,WBC:0&1/HP,脂肪管型 0—1/HP,应诊断为 A、排泄性肾盂造影 B、放射性核素检查(ECT) C、逆行性肾盂造影

D、肾血管造影

E、B 型超声波

169、为了了解肾脏大小、位置、形态、有无肿大和积液。

170、为了解肾脏形态、位置、血流、功能和有无梗阻。

A、苯丙酸诺龙

B、1.25 二羟 VD

C、叶酸

D、安体舒通

E、碳酸氢钠171、慢性肾功能不全高钾血症时宜用?

172、慢性肾功能不全低钙血症时需用?

A、分泌肾素

B、合成前列腺素

C、产生 1 羟化酶

D、生成促红细胞生成素

E、感受远曲肾小管内容量和浓度变化

173、球旁细胞可?

174、致密斑可? A、肾小球弥漫性毛细血管内皮细胞增生、炎性细胞浸润

B、肾小球新月体形成

C、肾小球系膜区 IgA 颗粒沉着

D、肾小球上皮细胞足突融合

E、肾小球基底膜广泛增厚

175、肾小球微小病变为:

176、肺出血肾炎综合征为?

A、尿沉渣涂片找细菌

B、尿培养及菌落计数

C、尿原浆菌株培养

D、尿中细菌血清型

E、尿中细菌表面有无抗体

177、区别膀胱炎和肾盂肾炎采取?

178、区别泌尿系感染是复发还是重新感染采取? A、内生肌酐清除率

179、反映远曲小管功能的是:

180、反映肾小球功能的是:尿镜检中发现有:

A、白细胞管型

B、红细胞管型

C、上皮管型

D、蜡状管型

E、脂质管型最常见于:

181、急性肾小球肾炎

182、微小病变肾病

A、50ml

B、100ml

C、120ml

D、300ml

E、400ml

183、正常人从肾小球滤出的滤液(原尿)每分钟约为:

184、少尿的定义是 24 小时尿量少于?

A、红细胞管型

B、白细胞管型

C、脂质管型

D、上皮细胞管型

E、蜡状管型

185、急性肾小球肾炎可出现?

186、慢性肾小球肾炎可出现?

187、肾盂肾炎可出现?

A、代谢性酸中毒

B、高血钙

C、高血钾

D、低血磷

E、低血镁患以下病症时常见改变为:

188、慢性肾炎尿毒症

189、急性肾功能衰竭少尿期

A、抗菌药物疗程稍长,多采用联合用药

B、用药后症状消失即停药

C、用药后 48 小时无效应考虑更换抗菌药物

D、用糖皮质激素

E、应用消炎痛以下疾病的治疗应是:

190、急性肾盂肾炎

191、慢性肾盂肾炎

A、少尿

B、血尿

C、尿频尿急

D、高度浮肿

E、胃肠道症状下列疾病最突出症状是:

192、肾病综合症

193、尿毒症

194、急性肾炎

A、Alport 综合症

B、狼疮肾

C、肾淀粉样变

D、痛风肾

E、骨髓瘤肾病人因尿蛋白++及镜下血尿 5—8/HP,并有病史或临床表现,首选诊断是什

么?

195、女性,28 岁,有游走性关节痛与血小板减少史

196、男,15 岁,轻度重听,其史已死于尿毒症

A、酚红排泄试验

B、内生肌酐清除率

C、BUN 测定

D、尿浓缩稀释试验

E、血肌酐测定

197、测定肾小球滤过功能最灵敏的方法是:

198、测定肾小管远曲小管功能最灵敏的方法是:

A、强的松

B、消炎痛

C、潘生丁

D、环磷酰胺

E、苯丙酸诺龙上述哪种药物的作用机理为以下所列?

199、抑制前列腺素合成

200、促进蛋白质合成

五.血液内科试题

A题型

1、缺铁性贫血用铁剂治疗后判定有效的早期指标是:

A 自觉症状明显改善B、HB、RBC 计数上升C、网织红细胞计数上升 D、骨髓内外铁恢复正常E、血清铁、总铁结合力恢复正常

2、诊断急性白血病最有力的根据是:

A、贫血、出血、发热

B、淋巴结肿大

C、肝脾肿大

D、胸骨压痛

E、骨髓中有大量原始细胞

3、患者女性,25 岁,头昏,心悸 1 年余,近 1 年来月经增多,喜吃泥土,查体:面色苍白,毛发干燥无光泽,心率 100 次/分,心尖区有 I 到 II 级收缩期吹风样杂音,肝脾未扪及,双下肢轻度凹陷性水肿,化验血象:HB:50g/L,RBC:3.0X10^12/L,WBC:4.0X10^9/L,根据以上表现你考虑哪种疾病可能性最大:

A、风湿性心脏病

B、慢性肾炎

C、缺铁性贫血

D、再生障碍性贫血

E、溶血性贫血

4、确定再生障碍性贫血诊断的主要依据是:

A、全血细胞减少

B、网织红细胞减低

C、外周血不出现幼稚细胞

D、骨髓增生减低,巨核细胞减少

E、中性粒细胞碱性磷酸活性增高5、溶血性贫血患者,脾肿大,CoomBs 试验阴性,血片可见多量的小球形红细胞,应首选治疗为: A、切除脾脏B、糖皮质激素C、输血

D、环磷酰胺

E、6—mp

6、32 岁,女工,2 周来脸色苍白,巩膜轻度黄染,脾左肋下 3cm,血红蛋白80g/L,网织红细胞 16%,红细胞脆性试验正常,CoomBs 试验阳性,首先治疗为:

A、脾切除术

B、低分子右旋糖酐

C、肾上腺皮质激素

D、维生素 B12

E、紧急输新鲜血

7、何杰金病患者,横膈上下均有淋巴结肿大,脏器和骨髓均无侵犯,无痢疾症状,治疗方法首先: A、局部放射治疗B、扩大照射至腹主动脉旁区包括脾区

C、全身淋巴结照射

D、全身淋巴结照射+MOPP 方案化疗

E、化疗为主,局部放疗辅助,

8、特发性血小板减少性紫癜当前首选疗法是:

A、骨髓移植

B、联合化疗

C、糖皮质激素

D、VB12

E、脾切除

9、下列哪一项检查结果不支持缺铁性贫血的诊断:

A、血清铁降低

B、总铁结合力升高

C、血清铁蛋白降低

D、血清铁未饱合度降低

E、骨髓铁染色,铁粒幼细胞 24%

10、女性,29 岁,月经过多三年,皮肤于碰撞部位有瘀斑,刷牙时,有时齿龈出血。查体:HB:90g/L, WBC:0.4X10^9/L,RBC:36X10^9/L,网织红细胞 2.0%,骨髓检查:增生活跃,粒红比为 1:2.3,粒系增生异常,巨核细胞全片有 100 只,原始巨核细胞 15 只,红细胞中心淡染,血清铁蛋白 8ug/L,最可能的诊断是:

A、再生障碍性贫血

B、失血性贫血

C、原发性血小板减少性紫癜伴缺铁性贫血

D、巨细胞性白血病

E、失血性贫血

11、男性,17 岁,头昏,皮肤紫癜月余,近一周来,头痛,呕吐入院,查体:面色苍白,全身浅表淋巴结肿大,颈略有抵抗,肝大肋下 1.5cm,脾大肋下2.0cm,HB5g/L,WBC:22 X10^9/L,骨髓检查:增生明显活跃,原始及早淋巴细胞占 40%,巨核细胞及血小板减少,脑脊液检查潘氏试验阳性,白细胞 20 个/mm3,诊断最大的可能是:

A、传染性单核细胞增多症

B、病毒性脑膜炎

C、结核性脑膜炎

D、急性淋巴细胞白血病并发脑病

E、流脑

12、再生障碍性贫血诊断依据中错误的是:

A、网织红细胞减少

B、全血细胞减少

C、一般无肝脾淋巴结肿大

D、骨髓有核细胞增生

E、骨髓巨核细胞减少

13、一男性患者 60 岁,微热,骨痛,病理骨折,贫血,血沉增快,免疫球蛋白增高,还需做下面哪些检查为首选:

A、血清铁蛋白

B、骨髓穿刺检查

C、血浆蛋白电泳

D、血培养

14、体内缺铁时的最早表现为:

A、血清铁降低

B、血清总铁结合力增高

C、贮存铁减少

D、平均红细胞体积减小

E、平均红细胞血红蛋含量降低

15、下列溶血性疾病的首选治疗,最正确的是: A、遗传性球形细胞增多症——糖皮质激素或其它免疫抑制药物 B、自体免疫溶血性贫血———糖皮质激素C、阵发性睡眠性血红蛋白尿——脾切除 D、遗传性非球形细胞增多症——糖皮质激素及其它免疫抑制药物 E、药物性溶血性贫血——抗凝疗法

16、特发性血小板减少性紫癜的急性型和慢性型有以下区别,但除外: A、常见儿童B、80%病前 1—3 周有上呼吸道感染或病毒感染史 C、起病急而重但可以有自限性

D、血小板常低于 20*10^9/L

E、骨髓中巨核细胞减少

17、下列哪项与急性粒细胞白血病的诊断不符: A、周围血全血细胞减少

B、周围血未见原始或幼稚粒细胞

C、骨髓原始与幼稚细胞占 80%,过氧化酶染色阴性

D、胸骨有压痛

E、中性粒细胞碱性磷酸酶活性降低

18、下列临床表现中,哪一项不出现于缺铁性贫血:

A、吞咽困难

B、嗜泥土癖

C、反甲

D、共济失调

E、口角炎

19、18 岁,女,务农,反复出现自发性皮肤瘀斑,偶有鼻衄,月经量过多一年,Hb:10g/l,WBC:

5.0*10^9/L,分类正常,骨髓巨核细胞 85 个/片,治疗应首选下列哪项: A、雄性激素 B、免疫抑制剂 C、糖皮质激素 D、脾切除 E、维生素 K 20、男性,24 岁,不规则发热三个月,右颈部淋巴结肿大,韧,穿刺涂片见”镜影状”细胞,应首先考虑: A、慢性淋巴细胞白血病 B、非何杰金淋巴瘤 C、

淋巴结核 D、癌肿淋巴结转移 E、何杰金病

21、一 13 岁女学生,头晕,乏力,尿黄一个月,Hb:86g/L,BP:10*10^9/L,Coombs 试验阳性,最可能的诊断为:

A、阵发性睡眠性血红蛋白尿

B、阵发性寒冷性血红蛋白尿

C、自体免疫溶

血性贫血 D、EVANS 综合征

22、下列哪项试验结果最能确定缺铁的诊断:

A、红细胞中心淡染区扩大

B、血清铁低于正常

C、血清铁蛋白低于

正常 D、MCV 减少 E、MCH 减少

23、溶血最直接的证据是:

A、血清直接胆红素增多

B、红细胞寿命缩短

C、网织红细胞增多

D、

骨髓红系细胞比例增高 E、尿中尿胆原增多

24、下列哪项是诊断急性白血病的最主要依据:

A、血红蛋白尿

B、外周血白细胞计数

C、外周血白细胞分类

D、骨

髓细胞学检查 E、细胞染色体检查

25、急性白血病经获得完全缓解后,下一步骤应该是:

A、停止化疗

B、维持治疗

C、强化巩固治疗

D、干扰素治疗

E、

免疫治疗

26、女性,20 岁,月经量增多,伴下肢反复出现瘀点 1 年,肝脾未及,Hb:

90g/L,WBC:4.3*10^9,网织红细胞 3%,骨髓有核细胞增生活跃,巨核细

胞 180 个,巨核细胞形成血小板减少,应首选作哪项检查以助诊断:

A、酸溶血试验

B、血小板相关 IgG

C、凝血素

D、凝血酶原时间

E、血小板凝集试验

27、男患,患慢性粒细胞白血病(慢粒)已 3 年,近来不明原因发热,贫血加重,脾进行性肿大,血小板

90*10^9/L,骨髓中原始细胞 38%,口服马利兰及用抗生素无效,下列哪种情

况最为可能? A、慢粒并感染 B、类白血病反应 C、慢粒加速期 D、慢

粒急变期

28、下列疾病中除了哪种疾病均有贫血,出血、全血细胞减少?

A、再生障碍性贫血

B、脾功能亢进

C、EVANS 综合症

D、阵发性睡

眠性血红蛋白尿 E、急性非白血性白血病

29、青年女性,农民,颜面苍白并感吞咽困难,血红蛋白 55g/L,白红胞及血

小板正常,血片中见红细胞大小不均,以小细胞为主,中心淡染区扩大,首选

治疗药物为:

A、维生素 B1

B、叶酸

C、强的松

D、口服铁剂

30、男,24 岁,颈部淋巴结肿大二个月余,左胸腔积液一个月,脾肿大肋下

4.1cm,骨髓穿刺涂片查到 12

%淋巴肉瘤细胞,该患者的最可能的诊断为:

A、何杰金病

B、急性白血病

C、非何杰金淋巴瘤

D、恶性组织细胞病

E、多发性骨髓瘤

31、男性,20 岁,颈后疖肿破溃,挤压排脓后,高热、寒战,皮肤粘膜出血,Hb:90g/L,WBC:45*10^9/L, PLC:50*10^9/L,凝血酶原时间 20 秒(正

常对照是 15 秒),纤维蛋白原定量 1200mg/L,3P 试验阳性,肝功正常,最

可能的诊断是:

A、播散性血管内凝血(DIC)

B、重症肝病合并凝血功能异常

C、原发

性纤溶症 D、血友病 IV E、感染合并 DIC

32、女性,22 岁,皮肤瘀点,月经量过多余,浅表淋巴结不大,血小板 30*

10^9/L,骨髓象:颗粒型巨核细胞比例增多,产板巨核细胞为 O,该患者最可

能的诊断为:

A、血管性血友病

B、特发性血小板减小性紫癜

C、过敏性紫癜

D、播

散性血管内凝血 E、急性白血病

33、女性,28 岁,乏力半年,贫血貌,无出血,血红蛋白 70g/L,白细胞及血

小板正常,骨髓增生活跃,以红系增生明显,幼红细胞体积小,红细胞中心淡

染区扩大,血肌酐正常,治疖本病最常用的药物是什么? A、叶酸 B、VB12

C、促红细胞生成素

D、枸橼酸铁

E、硫酸亚铁

34、女性,27 岁,一年反复牙龈血,鼻衄,皮肤出血点,月经量多,Hb:96g/L,

血小板:40*10^9/L,骨髓巨核细胞增多,多为未成熟型,应首选下列哪种治疖:

A、输血

B、肾上腺糖皮质激素

C、脾切除

D、免疫抑制剂

E、雄

性激素

35、男性患者,24 岁,一年来头晕、乏力、心悸,偶有鼻衄,贫血状,皮肤有

瘀点,肝脾不大,Hb:60g/L,网织红细胞:0.001,WBC:2.8*10^9/l,PLC:

38*10^9/L,骨髓增生低下,未见巨核细胞,最可能的诊断是:

A、再生障碍性贫血

B、慢性失血性贫血

C、急性白血病

D、特发性血

小板减少性紫癜 E、阵发性睡眠性血红蛋白尿

36、何杰金病的临床首发症状常是:

A、颈部或锁骨上淋巴结肿大

B、全身搔痒

C、肝肿大

D、脾肿

大 E、周期性发热

37、某女性患者,24 岁,一年来反复下肢紫癜,月经过多,病前无服药史,脾

肋下 1cm,肝未及,血红蛋白 100g/L,白细胞:5.4*10^9/L,血小板 25*

10^9/L,肾功能正常,LE 细胞阴性,骨髓增生活跃,可见

31 个巨核细胞,其中幼稚型 6、颗粒型 22,产板型 3,PAIgG 增高,最可能

的诊断为: A、再障 B、营养性巨幼细胞贫血 C、阵发性血小板减少性紫

癜 D、脾功能亢进 E、全身性红斑狼疮

38、一男性 20 岁,头痛,头晕一个月,高热鼻衄一周,全身浅表淋巴结不大,

肝可触及过,脾不大,红

细胞 1.8*10^9/L,白细胞:3.5*10^9/L,血小板:80*10^9/L,骨髓象增生

减低,红系和粒系各阶段比例大致正常,未见巨核细胞,可见组织嗜碱细胞,

最可能的诊断为:

A、急性白血病

B、再生障碍性贫血

C、特发性血小板减少性紫癜

D、颗

粒细胞减少症 E、营养不良性贫血合并感染

39、Auer 小体不见于下列哪种白血症?

A、急性粒细胞白血病

B、急性淋巴细胞白血病

C、急性单核细胞白

血病 D、急性粒单核细胞白血病 E、急性早幼粒细胞白血病

40、女性,24 岁,寒战,高热、咳嗽、胸痛、咯血、尿少,色红,巩膜轻度黄染,皮肤多量瘀点瘀斑,血压 75/50mmHg,Hb:70g/L,WBC:21*10^9/L,血

片可见盔形,三角型红细胞,KPT:60 秒和粒系各 PT18 秒,FDP:32mg/L,血

尿纤维蛋白原:1200mg/L,本例最可能的诊断是:

A、重症肺炎并发溶血性贫血

B、感染性休克并发 DIC

C、败血症并

发血小板减少 D、EVANS 综合症 E、血栓性血小板减少性紫癜

41、女、23 岁,反复出现皮肤瘀点瘀斑,鼻衄 5 月余,脾轻度肿大,Hb:80g/L,WBC:40*10^9/L,出血时间:10 分钟,束臂试验阳性,Coombs 试验阴性,

骨髓检查巨核红胞一张涂片 160 个,颗粒型巨核细胞

90%,产生血小板巨核细胞 2%,本例最可能的诊断是:

病例分析题目及答案内科学

1、[病例摘要] 女性,59岁,间断咳嗽、咳痰5 年,加重伴咯血2 个月。 患者5年前受凉后低热、咳嗽、咳白色粘痰,给予抗生素及祛痰治疗,1 个月后症状 不 见好转,体重逐渐下降,后拍胸片诊为“浸润型肺结核”,肌注链霉素1 个月,口服利福平、雷米封3 个月,症状逐渐减轻,遂自行停药,此后一直咳嗽,少量白痰,未再复查胸片。2个月前劳累后咳嗽加重,少量咯血伴低热、盗汗、胸闷、乏力又来诊。病后进食少,二便正常,睡眠稍差。既往6 年前查出血糖高,间断用过降糖药,无药物过敏史。 查体:T37.4 ℃, P94次/ 分,R22次/ 分,BP130/80mmH,g一般稍弱,无皮诊,浅表淋巴 结未触及,巩膜不黄,气管居中,两上肺呼吸音稍减低,并闻及少量湿罗音,心叩不大, 心 率94 次/ 分,律齐,无杂音,腹部平软,肝脾未触及,下肢不肿。 化验:血HGB 110g/L, WBC 4.5×109/L, N 53%, L47%, PLT 210 ×109/L, ESR 35mm/h,空腹血糖9.6mmol/L ,尿蛋白(-),尿糖(3+) 、诊断及诊断依据(4 分)

二、鉴别诊断(1.5 分) 三、进一步检查(1.5 分) 四、治疗原则(3 分) 2、[病例摘要] 男性,65 岁,持续心前区痛4 小时。 4 小时前即午饭后突感心前区痛,伴左肩臂酸胀,自含硝酸甘油1 片未见好转,伴憋气、乏力、出汗,二便正常。既往高血压病史6年,最高血压160/100mmH,g 未规律治疗,糖尿病病史 5 年,一直口服降糖药物治疗,无药物过敏史,吸烟10年,每日20 支左右,不饮酒。 查体:T37℃,P100次/ 分,R24次/ 分,Bp150/90mmH,g半卧位,无皮疹及出血点,全身浅表淋巴结不大,巩膜无黄染,口唇稍发绀,未见颈静脉怒张,心叩不大,心律100 次/ 分,律齐,心尖部Ⅱ /6 级收缩期吹风样杂音,两肺叩清,两肺底可闻及细小湿罗音,腹平软,肝脾未及,双下肢不肿。 化验:HGB134g/L, WBC9.6 ×109/L, 分类:中性分叶粒72%,淋巴26%,单核2%, PLT 250×109/L, 尿蛋白微量,尿糖(2+), 尿酮体(-) ,镜检(-)

心血管内科学考试试题答案(供参考)

一、A1型题:题干在前,选项在后。有A、B、C、D、E五个备选答案其中只有一个为最佳答案,其余选项为干扰答案。考生须在5个选项中选出一个最符合题意的答案(最佳答案),并按考试规定的方式将相应的位置上的字母涂黑。 第1题急性炎的临床特点 A、疼痛不放射 B、随渗液量的增多而加重 C、前俯位时加重 D、深呼吸时减轻 E、前俯位时减轻 正确答案是:E 第2题下列哪项不属于高级心肺复苏的措施 A、人吸 B、气管插管 C、除颤 D、建立静脉通路 E、药物 正确答案是:A 第3题诊断冠心病最有价值的侵入性检查是 A、心腔内心音图检查 B、心腔内电生理检查 C、冠状动脉造影 D、心腔内心电图检查 E、心内膜心肌活组织检查

正确答案是:C 第4题征(Ewart征)是 A、心尖搏动弱,位于心浊音界左缘的内侧或不能扪及 B、左肩胛骨出现浊音及肺受压所引起的支气管呼吸音 C、在胸骨左缘第3、4肋间可闻及心包叩击音 D、心脏叩诊浊音界向两侧扩大 E、心音低而遥远 正确答案是:B 第5题冠状主要危险因素错误的是 A、 B、血脂异常 C、年龄 D、吸烟 E、血压 正确答案是:A 第6题主动脉-冠状动脉旁路移植术的适应症不包括 A、左冠状动脉主干病变狭窄>50% B、有轻度室性伴左主干或3支病变 C、稳定型对内科药物治疗反应不佳,影响工作和生活 D、冠状动脉3支病变伴左心室射血分数<50% E、左前降支和回旋支近端狭窄≥70% 正确答案是:B

第7题的晚期,可见 A、收缩压升高,脉压正常 B、收缩压和脉压均升高 C、脉压升高,收缩压正常 D、收缩压和脉压均下降 E、收缩压和舒张压均下降 正确答案是:D 第8题持续性室速无显著血流动力学障碍,药物治疗首选 A、西地兰 B、β-受体阻滞剂 C、 D、 E、 正确答案是:D 第9题对诊断有意义的检查是 A 尿17-羟类固醇测定 B 尿中24小时VMA测定 C 肾脏CT扫描 D 血钾测定 E 基础代谢测定 第10题进行直流电除颤的能量是 A、50J

《内科学》问答题及答案

《内科学》问答题及答案 试述肺结核的化学治疗原则P53 答:原则:早期、规律、全程、适量、联合。 1、试述COPD的病程分期和严重程度分级P66 答:COPD的病程分期: 1)急性加重期:在疾病过程中,短期内咳嗽、咳痰、气短和(或)喘息加重,痰量增多,呈脓性或黏液脓性,可伴发热等症状; 2)稳定期:患者咳嗽、咳痰、气短等症状稳定或症状较轻。 COPD严重程度分级: 0级:高危(肺功能检查正常) Ⅰ级:轻度(FEV1/FV C<70%.FEV1≥80%预计值); Ⅱ级:中度(FEV1/FV C<70%.50%≤FEV1<80%预计值); Ⅲ级:重度(FEV1/FV C<70%.30%≤FEV1<50%预计值); Ⅳ级:极重(FEV1/FV C<70%.FEV1<30%预计值)。 2、支气管哮喘的诊断标准P72 答:诊断标准: 1)反复发作喘息、气急、胸闷或咳嗽,多与接触变应原、冷空气、物理、化学性刺激、 病毒性上呼吸道感染、运动等有关。 2)发作时在双肺可闻及散在或弥漫性,以呼气相为主的哮鸣音,呼气相延长。 3)上述症状可经治疗缓解或自行缓解。 4)除外其他疾病所引起的喘息、气急、胸闷和咳嗽。 5)临床表现不典型者(如无明显喘息或体征)应有下列三项中至少一项阳性:①支气 管激发试验或运动试验阳性;②支气管舒张试验阳性;③昼夜PEF变异率≥20% 符合1~4条或4、5条者,可以诊断为支气管哮喘。 4、40岁以上男性长期中度吸烟者,出现哪些情况应警惕肺癌。P131 答: 1)刺激性咳嗽持续2~3周,治疗无效或原有慢支,咳嗽性质改变者。 2)持续痰中带血。而无其他原因解释者。 3)反复发作的同一部位的肺炎,抗生素治疗吸收不完全。 4)原有肺结核,病灶稳定,短期内病灶增大,或其他处出现新病灶。 5)原因不明的四肢关节疼痛及杵状指。 6)X线出现有段、叶性肺不张,单侧肺门块影或孤立性圆形病灶等。 7)血性胸水增长快,抗结核治疗效果不佳。 8)不明原因的肺脓肿,无中毒症状,无异物吸入史,抗炎效果差。 9)单侧性局限性哮鸣音,不因咳嗽而改变。 5、社区获得性肺炎定义及诊断依据是什么P18 答诊断依据:①新近出现的咳嗽、咳痰或原有呼吸道疾病症状加重,并出现脓性痰,伴或不伴胸痛。 ②发热。 ③肺实变体征和(或)闻及湿性啰音。 ④WBC>10×10^9/L或<4×10^9/L,伴或不伴中性粒细胞核左移。 ⑤胸部X线检查显示片状、斑片状浸润性阴影或间质性改变,伴或不伴胸腔积液。 以上1~4项中任何1项加第5项,除外非感染性疾病可做出诊断。

内科学试题库及答案(一)

内科学试题库及答案(一) 1.垂体危象时,下列何种情况最为多见 A.低钾性麻痹 B.低血糖性昏迷 C.谵妄 D.脑梗死 E.高钠高渗性昏迷 正确答案:B 2.结核性腹膜炎治疗中最重要的是 A.穿刺抽腹水 B.加强营养支持疗法 C.手术治疗肠梗阻、肠穿孔 D.加用糖皮质激素 E.联合足量全程的抗结核治疗 正确答案:E 3.类风湿性关节炎中,最先受累的关节组织是 A.骨组织 B.软骨组织 C.滑膜组织 D.韧带 E.关节囊 正确答案:C

4.慢性浅表性胃炎的临床表现哪项是错误的 A.可以引起恶性贫血 B.有时症状酷似消化性溃疡 C.消化性溃疡的发生率增高 D.胃酸偏低 E.易出现嗳气、反酸、腹胀等症状 正确答案:A 5.肾盂肾炎最主要的治疗措施是 A.多饮水或输液 B.卧床休息 C.应用糖皮质激素 D.应用抗生素 E.解痉止痛 正确答案:D 6. Budd-Chiari综合征时肝肿大是由于 A.肝淤血 B.代谢异常 C.肿瘤 D.血液病 E.中毒 正确答案:A 7. 哪种情况可引起全身淋巴结肿大 A.急性咽炎 B.慢性鼻炎

C.再生障碍性贫血 D.链霉素过敏 E.足癣合并感染 正确答案:D 8. 皮下出血面积的直径多大称为紫癜 A.<2mm B.2~3mm C.3~5mm D.>5mm E.以上均可 正确答案:C 9. 引起出血性疾病较常见的因素是 A.血管外因素 B.凝血因子缺乏 C.肝素或香豆类药物 D.抗凝血物质活性增加 E.血小板因素 正确答案:E 10. 轻度肿大的脾脏在仰卧位时触不到,医生可用双手触诊,病人应取哪种体位 A.右侧卧位,右下肢伸直,左下肢屈曲 B.右侧卧位,右下肢屈曲,左下肢伸直

心血管内科试题及答案(二)

心血管内科试题及答案一、A1型题: 第1题急性心包炎胸痛的临床特点 A、疼痛不放射 B、随渗液量的增多而加重 C、前俯位时加重 D、深呼吸时减轻 E、前俯位时减轻 正确答案是:E 第2题下列哪项不属于高级心肺复苏的措施 A、人工呼吸 B、气管插管 C、除颤 D、建立静脉通路 E、药物 正确答案是:A 第3题诊断冠心病最有价值的侵入性检查是 A、心腔内心音图检查 B、心腔内电生理检查 C、冠状动脉造影 D、心腔内心电图检查 E、心内膜心肌活组织检查

正确答案是:C 第4题心包积液征(Ewart征)是 A、心尖搏动弱,位于心浊音界左缘的内侧或不能扪及 B、左肩胛骨出现浊音及肺受压所引起的支气管呼吸音 C、在胸骨左缘第3、4肋间可闻及心包叩击音 D、心脏叩诊浊音界向两侧扩大 E、心音低而遥远 正确答案是:B 第5题冠状动脉粥样硬化主要危险因素错误的是 A、脑栓塞 B、血脂异常 C、年龄 D、吸烟 E、血压 正确答案是:A 第6题主动脉-冠状动脉旁路移植术的适应症不包括 A、左冠状动脉主干病变狭窄>50% B、有轻度室性心律失常伴左主干或3支病变 C、稳定型心绞痛对内科药物治疗反应不佳,影响工作和生活 D、冠状动脉3支病变伴左心室射血分数<50% E、左前降支和回旋支近端狭窄≥70% 正确答案是:B

第7题主动脉瓣狭窄的晚期,可见 A、收缩压升高,脉压正常 B、收缩压和脉压均升高 C、脉压升高,收缩压正常 D、收缩压和脉压均下降 E、收缩压和舒张压均下降 正确答案是:D 第8题持续性室速无显著血流动力学障碍,药物治疗首选 A、西地兰 B、β-受体阻滞剂 C、普罗帕酮 D、利多卡因 E、胺碘酮 正确答案是:D 第9题对诊断嗜铬细胞瘤有意义的检查是 A 尿17-羟类固醇测定 B 尿中24小时VMA测定 C 肾脏CT扫描 D 血钾测定 E 基础代谢测定 第10题进行直流电除颤的能量是 A、50J

内科学期末模拟试题及标准答案

模拟试题六 一、A型题(1~50题,每题1分,共40分) 1.诊断肺结核的主要依据是: A.低热、盗汗、咳嗽 B.血沉增快 C.结核菌素试验阳性 D.痰中找到结核菌 E.体重减轻 2.对于急性肾盂肾炎选择抗菌药物最有指导意义的是: A.尿路刺激征的轻重 B.热度的高低 C.尿中白细胞的多少 D.外周血白细胞增高程度E.尿沉渣涂片细菌革兰染色 3.引起上消化道出血最常见的病因是: A.慢性胃炎B.胃癌C.消化性溃疡 D.肝硬化门脉高压E.胃血管瘤 4.尿毒症患者出现贫血的主要原因是: A.红细胞寿命缩短B.红细胞生成素减少 C.毒素抑制骨髓造血 D.肠道溃疡出血使血液丢失 E.营养不足,缺铁、低蛋白 5.关于慢性肾炎,不正确的是选项: A.仅少数由急性肾炎直接迁延而来 B.部分发病与链球菌感染有关 C.并发感染可用氨基糖苷类抗生素.且剂量要足 D.低蛋白血症时如无氮质血症可不限制蛋白摄人量 E.凡有1年以上蛋白尿、浮肿、高血压、肾功能不全者应考虑本病 6.肾病综合征使用糖皮质激素疗效最佳的是: A.微小病变型肾病B.局灶节段硬化性肾炎C.膜性肾病 D.膜增生性肾炎E.急性肾炎 7. 男,58岁。反复咳嗽、咯白粘痰、喘息6年,冬季加重,1周前上述症状再发,经治疗无效住院。检查:胸廓对称, 两肺可闻及干、湿罗音。X线示:两肺纹理增粗。其可能的诊断为: A.慢性单纯型支气管炎急性发作期 B.慢性单纯型支气管炎迁延期 C.慢性喘息型支气管炎急性发作期 D.慢性喘息型支气管炎迁延期 E.慢性单纯型支气管炎缓解期 8. 下列哪一项不是流脑的诊断条件: A.皮肤粘膜有瘀点、瘀斑B.脑膜刺激征阳性C.白细胞总数及中性粒细胞增高 D.细菌学或免疫学检查阳性E.脑脊液检查糖及氯化物显著增高 9. 肺结核化学药物治疗中,判断疗效最主要的指标是: A.病灶吸收好转B.痰结核菌持续三个月转阴C.血沉恢复正常 D.空洞闭合E.体温恢复正常、体重增加 10. 慢性支气管炎急性发作期最重要的治疗措施是: A.平喘B.祛痰C.控制感染D.镇咳E.免疫治疗 11. 急性心肌梗死时,应用尿激酶治疗的目的在于: A.调节离子代谢B.营养心肌C.溶解冠状动脉内粥样硬化斑块 D.溶解冠状动脉内的血栓E.改善心肌微循环 12.男,26岁。间断右上腹部疼痛半年。半年来反复右上腹疼痛,饭前明显,进食可减轻,有夜间痛,并有反酸、嗳气。 体检:腹部软,右上腹轻度压痛,未触及包块,肝脾未触及。诊断为: A.慢性胃炎B.慢性胆囊炎C.胃溃疡D.慢性胰腺炎E.十二指肠溃疡 13.以下哪项是消化性溃疡合并穿孔的肯定性诊断依据: A.腹部疼痛突然加重B.发热C.腹部有压痛、反跳痛、板状腹 D.肝浊音界缩小E.X线透视见右膈下新月状游离气体 14. 根除Hp的标准治疗方案是: A.氢氧化铝+甲硝唑+四环素B.胶体次枸橼酸铋+阿莫西林+四环素 C.法莫替丁+甲硝唑+四环素D.硫糖铝+ 甲硝挫+克拉霉素 E.奥美拉唑+替硝唑+克拉霉素 15. 确诊伤寒最可靠的依据是 A.肥达反应阳性B.血白细胞减少C.血培养阳性 D.高热、中毒症状E.肝脾肿大、肠出血 16. 男,15岁。消瘦,HbA1c l2.8%,C肽释放呈低平曲线,GAD—Ab阳性。治疗除饮食控制外首选的是: A.继续观察B.格列本脲C.二甲双胍D瑞格列奈E.胰岛素 17. 女,35岁。有糖尿病家族史,空腹血糖6.8mmol/L。为确诊糖尿病,最有意义的检查是: A.重复测空腹血糖B.测餐后2小时血糖C.多次测任意血糖 D. 口服葡萄糖耐量试验E.糖化血红蛋白

肾内科学篇病例分析

题目部分,(卷面共有8题,100.0分,各大题标有题量和总分) 一、病例分析(8小题,共100.0分) (12分)[1] 男,57岁,腰、肩痛数年,发现蛋白尿1周入院。无水肿,血压正常。尿常规蛋白+++,红白细胞少量,血红蛋白50g/L,血尿素氮1O.65mmol/L,肌酐230.85μmol/L 。血钙12.00mg/dl(3.00mmol/L),血沉60mm/h,血碱性磷酸酶正常。 (1)要考虑哪些疾病? (2)进一步做哪些检查才能确定诊断? (14分)[2] 男性,65岁。咳嗽、高热(右肺炎)后2周出现面部及下肢水肿,尿褐色,泡沫多,血压 165/97mmHg。尿蛋白+++,显微镜下尿红细胞3+~4+/HP。经中药治疗1月余,食欲减退,面色苍白,查血肌酐400μmol/L ,血红蛋白78g/L。 (1)临床上要鉴别哪些疾病,应进一步做哪些检查? (2)如果此病例,肾活检为系膜增生,肾小囊内细胞性新月体形成达55%,应如何处理? (8分)[3] 一肾小管酸中毒病人,但尿pH>6.0,为进一步明确其病变部位在近端还是远端小管,进行 了氯化铵负荷试验,服用氯化铵后尿pH降至 6.0以下。其诊断属哪一型肾小管酸中毒?如何处理? (12分)[4] 男性,57岁,患者不明原因出现尿泡沫多、量少伴颜面水肿20余天。无发热,皮疹,关 节肿痛,呕吐,血尿及尿路刺激征,尿量约550ml/d,自服中药方剂,症状无缓解,5天后尿量减至400ml/d,仍服中药,查尿蛋白3+,红细胞3+/HP,为混合型血尿。WBC6.4G/L,血红蛋白142g/L,PLT178G/L,24小时尿蛋白定量 6.4g,血肌酐398μmol/L,BUN30.0mmol/L,血清蛋白13g/L,球蛋白16g/L,胆固醇8.5mmol/L,甘油三酯 2.42mmol/L,免疫球蛋白及补体均正常,尿渗透量350mOsm/kgH O。B超双肾大小正常,实质回声增强,腹腔及双侧胸腔积液(漏出液)。心电图正常。血压131/70mmHg。予抗生素静滴,扩容(低分子右旋糖酐),利尿病情无好转。 问题: (1)诊断考虑什么? (2)其急性肾衰竭的原因? (14分)[5] 男,24岁。咽痛、发热1周后出现全程无痛肉眼血尿。热退后尿常规:显微镜下红细胞++/HP,尿蛋白++++。眼睑轻度水肿。血压150/90mmHg。 问题: (1)临床诊断上应考虑哪些疾病? (2)需进一步做哪些检查? (16分)[6] 患者,男,45岁。因反复颜面下肢水肿、蛋白尿5个月,加重伴肾功能不全3个月入院,起病初无镜下血尿,尿糖阴性,而后逐渐出现尿隐血+~3+,曾在外院行肾活检提示肾小球 轻度系膜增生性病变,免疫荧光示IgA+,IgM+。近1个月曾服用不明成分的中药汤剂。尿 量600ml/d。体检:双下肢水肿,腹部膨隆,移动性浊音阳性。 尿常规:PRO++++,24h尿蛋白定量16.04g/24h。尿蛋白电泳谱:大分子14.6%,中分子53.7%,小分子31.7%,尿沉渣RBC350万~750万/ml,WBC0~1/HP。肾

2017心血管内科出科考试题及答案

一 . 单选题(共100题,每题1分) 1 . 不能用于判断急性心肌梗死后溶栓成功的临床指标为 E A . 胸痛缓解 B . 心电图示ST段下降 C . 频发的室性期前收缩 D . CK、MB峰值前移 E . 窦性心动过速 2 . 二度Ⅰ型房室传导阻滞的心电图特征是 B A . P-R间期进行性缩短,直至一个P波受阻不能下传到心室 B . 相邻P-R间距进行性延长,直至一个P波受阻不能下传到心室 C . P-R何期进行性延长,直至一个P波受阻不能下传到心室 D . P—R问期>0.20秒;P波无受阻 E . P-R间期固定,P波间断受阻不能下传到心室 3 . 一患者于平卧时频发胸痛,且心率达110次/分,血压升高至21.3/12.8kPa(160/96mmHg),并有左室舒张功能减退。宜首选何药治疗 C

A . 维拉帕米 B . 美托洛尔 C . 异山梨酯 D . 卡托普利 E . 地高辛 4 . 发生急性心肌梗死后,预防左室重构的药物为 A A . 卡托普利 B . 美托洛尔 C . 维拉帕米 D . 肼苯达嗪 E . 硝酸甘油 5 . 男,42岁。2年前出现左下肢行走10余分钟后胀痛。休息片刻缓解,再行走后疼痛又出现。无吸烟史,发病前半年左足部外伤已治愈。体格检查:左下肢皮色较苍白,左足背动脉未触及。最可能的诊断是 B A . 动脉粥样硬化性闭塞症 B . 血栓闭塞性脉管炎

C . 雷诺病 D . 多发性大动脉炎 E . 结节性动脉周围炎 6 . 风心病患者因心衰而用地高辛0.25mg,每日2次,共10天,同时并用氢氯噻嗪,出现心动过速,频发室性早搏,血钾31mmol/L,正确处理是E A . 继续服用地高辛 B . 停用地高辛,加用毛花苷丙 C . 停用地高辛,加用利多卡因 D . 停用地高辛,点钾盐,静注利多卡因 E . 停用地高辛,静点钾盐 7 . 下列哪一项是劳力型心绞痛典型的心电图改变 A A . 发作时某些导联ST段呈水平型或下垂型降低,可伴有T波低平或倒置 B . 发作时某些导联ST段呈上斜型降低 C . 发作时某些导联ST段呈抬高型升高,伴高尖T波 D . 某导联ST-T呈鱼钩型降低,伴T波倒置

内科学题库+答案

呼吸系统疾病 肺炎 一、单项选择题 1.下列对肺炎的描述哪个是错误的 A.肺炎是指由细菌感染引起的终末气道,肺泡和肺间质炎症。 B.大叶性肺炎亦称肺泡性肺炎。 C.感染性肺炎的发生主要决定于宿主和病原体的因素。 D.肺炎在X线胸部检查时有明显的肺实质炎性浸润。 E.HAP常发生于高龄,有心肺基础疾病,免疫力低下的人群。 2.经口腔痰定量培养分离的细菌在下列那种情况下可以认为是肺部感染的致病菌: A.细菌浓度>104cfu/ml B.连续两次以上分离到相同细菌,浓度≥104cfu/ml C.细菌浓度<104cfu/ml D.细菌浓度≥107cfu/ml E.细菌浓度105cfu/ml~106cfu/ml 3.感染性肺炎在胸部X片上出现空洞改变,可能性最小的诊断是 A.金葡菌肺炎 B.真菌性肺炎 C.肺炎链球菌肺炎 D.G—杆菌肺炎 E.厌氧菌性肺炎 4.18岁男性患者,既往健康,受凉后突然发热,寒战,胸痛,体检:T 39.5℃,P110次/分,R28次/分,右上肺有管样呼吸音。X片示:右上肺大片密变阴影。WBC 26.0×109/L,N90%,最恰当的诊断为: A.右上肺干酪性肺炎 B.病毒性肺炎 C.支原体肺炎 D.肺炎链球菌肺炎 E.急性肺栓塞 5.肺炎链球菌肺炎经青霉素及对症支持治疗后,其预后最可能是: A.完全吸收,不留痕迹 B.肺部空洞形成 C.肺部留下瘢痕条索影 D.支气管扩张 E.脓胸 6.男性患者,45岁,因高热,咯大量臭脓痰伴咯血住院。X线胸片诊断:右肺下叶背段肺脓肿,先后应用青霉素、甲硝唑等多种抗生素治疗七个月仍有反复咯痰,咯血,咯血量最大可达500ml,下一步最佳的治疗方案是: A.痰及体位引流 B.痰培养+药敏,选用敏感抗生素 C.抗感染同时加强止血剂应用 D.外科手术治疗 E.气管内滴入药物治疗 7.急性吸入性肺脓肿最典型的临床表现是:

儿内科试题及答案

2016年北京市医师定期考核业务水平测评 (儿内科专业试卷) 单位:____________________________________________________ 姓名:_________________ 性别:____________ 身份证号:_____________________________________________________________ 分数 共100分,60分合格。 一.单选题(80分, 每题1分) 1.小儿生长发育的两个高峰是() A.新生儿期和婴儿期 B.婴儿期和幼儿期 C.婴儿期和学龄前期 D.学龄前期和青春期 E.婴儿期和青春期 2.正常2岁~青春期前小儿体重(kg)应按以下哪项公式计算?() A.体重=出生体重+年龄(岁)×2 B.体重=出生体重+年龄(岁)×7 C.体重=年龄(岁)×2+8 D.体重=年龄(岁)×3+7 E.体重=年龄(岁)×3+8 3.正常2岁~青春期前小儿身高(cm)应按以下哪项公式计算?()

A.身高=年龄(岁)×7+65 B.身高=年龄(岁)×7+75 C.身高=年龄(岁)×2+80 D.身高=年龄(岁)×7+85 E.身高=年龄(岁)×7+95 4.一般正常小儿头围的数值正确的是() A.出生时36cm 个月时40cm 岁时46cm 岁52cm 岁54cm 5.一般正常小儿前囱的闭合时间为() 个月个月个月-30个月-36个月 6.关于小儿的生长发育,以下哪项是错误的() 岁时胸围与头围大致相等岁时身体中点位于脐部岁时身体中点位于耻骨联合 岁上臂围大于表示营养良好岁时腕骨骨化中心的数目为7个 7.为预防佝偻病,新生婴儿应在出生后2周开始补充维生素 D ,每天补充的剂量应为() 8.以下哪项不符合水痘() A.潜伏期8~21日 B.皮疹首先在发迹、颈侧部和耳后出现 C.皮疹最后达手掌和足底 D.皮疹一般在3~5日内分别出齐 E.疹退后不会遗留色素沉着 9.流行性腮腺炎最常见的并发症是() A.睾丸炎或卵巢炎 B.胰腺炎 C.耳聋 D.脑膜脑炎 E.心肌炎

2017心血管内科出科考试题及答案

一 . 单选题(共 100题,每题 1分) 1 . 不能用于判断急性心肌梗死后溶栓成功的临床指标为E A . 胸痛缓解 B . 心电图示ST段下降 C . 频发的室性期前收缩 D . CK、MB峰值前移 E . 窦性心动过速 2 . 二度Ⅰ型房室传导阻滞的心电图特征是 B A . P-R间期进行性缩短,直至一个P波受阻不能下传到心室 B . 相邻P-R间距进行性延长,直至一个P波受阻不能下传到心室 C . P-R何期进行性延长,直至一个P波受阻不能下传到心室 D . P—R问期>秒;P波无受阻 E . P-R间期固定,P波间断受阻不能下传到心室 3 . 一患者于平卧时频发胸痛,且心率达110次/分,血压升高至/(160/96mmHg),并有左室舒张功能减退。宜首选何药治疗 C A . 维拉帕米 B . 美托洛尔 C . 异山梨酯 D . 卡托普利 E . 地高辛 4 . 发生急性心肌梗死后,预防左室重构的药物为 A A . 卡托普利 B . 美托洛尔 C . 维拉帕米

D . 肼苯达嗪 E . 硝酸甘油 5 . 男,42岁。2年前出现左下肢行走10余分钟后胀痛。休息片刻缓解,再行走后疼痛又出现。无吸烟史,发病前半年左足部外伤已治愈。体格检查:左下肢皮色较苍白,左足背动脉未触及。最可能的诊断是 B A . 动脉粥样硬化性闭塞症 B . 血栓闭塞性脉管炎 C . 雷诺病 D . 多发性大动脉炎 E . 结节性动脉周围炎 6 . 风心病患者因心衰而用地高辛,每日2次,共10天,同时并用氢氯噻嗪,出现心动过速,频发室性早搏,血钾31mmol/L,正确处理是E A . 继续服用地高辛 B . 停用地高辛,加用毛花苷丙 C . 停用地高辛,加用利多卡因 D . 停用地高辛,点钾盐,静注利多卡因 E . 停用地高辛,静点钾盐 7 . 下列哪一项是劳力型心绞痛典型的心电图改变 A A . 发作时某些导联ST段呈水平型或下垂型降低,可伴有T波低平或倒置 B . 发作时某些导联ST段呈上斜型降低 C . 发作时某些导联ST段呈抬高型升高,伴高尖T波 D . 某导联ST-T呈鱼钩型降低,伴T波倒置 E . 全部导联(除aVR导联外)ST段下移 8 . 阿司匹林一级预防的剂量是 B A . 40~80mg B . 75~100mg

内科学选择题题库+答案

内科选择题题库 题号:1/2345 答题指南:各题选项可能多个正确,只能选择其中最佳一项肝硬化并上消化道出血导致肝昏迷的原理主要是 A.血容量减少 B.肝解毒功能受限 C.脑缺血及缺氧 D.肠道积血致产氨增多 E.肾血流量减少,使排氨减少 答案:D 题号:2/2345 输入异型血液多少毫升即产生溶血反应的症状? A.1~5ml B.5~10ml C.10~15ml D.15~20ml E.20~25ml 答案:C 题号:3/2345 肝脾肿大在急性白血病各型中,以哪一型最为显著: A.急性淋巴细胞性白血病 B.急性单核细胞性白血病 C.红白血病 D.急性粒细胞性白血病 E.以上都不是 答案:A 题号:4/2345 为昏迷病员作口腔护理时,应特别注意: A.压舌板轻轻撑开颊部 B.从外向里擦净口腔及牙齿的各面 C.血管钳夹紧棉球,蘸水不可过多 D.操作时动作要轻 E.观察口腔粘膜

答案:C 题号:5/2345 硝酸甘油,β受体阻滞药,钙拮抗药治疗心绞痛的共同作用是 A.扩张血管 B.减少心肌耗氧量 C.减慢心率 D.抑制心肌收缩力 E.减少心脏容积 答案:B 题号:6/2345 麻疹出疹的特点是: A.发热1-2日出疹,出疹后体温高 B.发热1-2日,热退后出疹 C.发热2-3日出疹,出疹同时有发热 D.发热3-4日出疹,出疹同时体温升高 E.发热3-4日 答案:D 题号:7/2345 盐酸肾上腺素注射剂如何保管 A.冷藏 B.密闭 C.干燥 D.避光 E.温箱 答案:D 题号:8/2345 异位妊娠常发生的部位是下述哪项: A.卵巢 B.输卵管 C.子宫颈 D.子宫角 E.腹腔

内科病例分析题

内科病例分析题 病例1: 女性,67岁,反复咳嗽,咳痰伴喘20年,再发加重一周,意识障碍一天。20年来,常咳嗽、咳痰,以冬春季为甚。近几年体力减退明显,稍动即气促。一周前,受凉后咳嗽加重,咳黄痰,痰不易咳出。三天前出现双下肢浮肿,尿量减少。昨天开始嗜睡,反应迟钝。在当地处理后送至我院。查体:T36℃, R30次/分,BP120/70mmHg。意识模糊,球结膜充血、水肿。胸廓呈桶状,叩诊呈过清音,听诊双下肺细湿罗音。心无异常。腹软,肝肋下2厘米,肝颈回流征阳性。双踝凹陷性水肿。实验室检查:血WBC5.1×10/L,N75%。 动脉血气:PH7.36 PaO2 70mmHg Pa CO2 65mmHg HCO 3 30mmol/L 问题: 1、诊断及诊断依据 2、鉴别诊断 3、进一步检查项目 4、治疗原则

病例1答案: 1.诊断及诊断依据 (1)诊断: 慢性支气管炎急性发作期 慢性阻塞性肺病 慢性肺源性心脏病 II型呼吸衰竭肺性脑病 心功能不全 (2)诊断依据: 1、慢性咳嗽病史,近一周咳嗽加重并有脓痰。 2、有肺气肿症状及体征 3、有大循环淤血表现,且不能用其他病解释。 4、动脉血气提示:II型呼吸衰竭 5、有精神症状 2.鉴别诊断 脑血管意外:基础病及血气检查均不支持,且无定位体征。 电解质紊乱:低钠、低氯血症 3.进一步检查 (1)、胸片

(2)、痰培养及药敏试验 (3)、血电解质 (4)、心电图 4.治疗原则 (1)、控制感染 (2)、保证呼吸道通畅:去痰、扩张支气管。必要时气管插管。 (3)、氧疗 (4)、纠正水盐电解质失衡 5、必要时行CT检查

心血管内科试题及答案解析

题目部分,(卷面共有54题,100.0分,各大题标有题量和总分) 一、标准配伍型(B型题)(3小题,共11.0分) (3分)[1] A、西地兰0.4mg以葡萄糖液稀释至20ml缓慢静脉注射 B、口服地高辛0.25mg,每天1次 C、地高辛0.25mg,每天3次,2天后每天1次 D、暂不用洋地黄 E、先用半量西地兰(0.2mg),稀释后缓慢静脉注射,必要时2小时后可重复 心力衰竭时应用洋地黄,选用方法为: (1)急性心肌梗死24小时内伴左心衰竭( ) (2)高血压病患者,夜间出现左心衰竭,近2周末用洋地黄( ) (3)风湿性心脏病伴房颤,原每天口服地高辛0.25mg,近3天因故未服,心衰加重,心率增快( ) (3分)[2] A、维拉帕米 B、利多卡因 C、苯妥英钠 D、莫雷西嗪 E、普鲁卡因胺 (1)治疗室上性心动过速首选( ) (2)治疗室性心动过速首选( ) (3)治疗预激综合征合并心房颤动首选( ) (5分)[3] A、QRS波型为R型 B、QRS波型为rSR'型 C、QRS波型为rS型 D、QRS波型为QS型 E、QRS波型为qR型 (1)左后分支阻滞Ⅱ、Ⅲ、aVF导联QRS波型为( ) (2)完全性左束支传导阻滞时V V QRS波型为( ) (3)左前分支传导阻滞Ⅱ、Ⅲ、aVF导联QRS波型为( ) (4)完全性右束支传导阻滞时V QRS波型为( ) (5)左前分支传导阻滞Ⅰ、aVL导联QRS波型为( ) 二、病例串型最佳选择题(A3-A4型题)(1小题,共3.0分) (3分)[1] 女,65岁,高血压史8年,因紧张、劳累,出现头痛、焦虑、恶心欲吐、烦躁不安、心悸、多汗、面色苍白、间或潮红、视力模糊,测血压为260/120mmHg,心率120次/分,两肺底湿性啰音。 (1)此时诊断为( ) A、恶性高血压 B、高血压危像 C、高血压脑病 D、肾性高血压 E、老年人高血压

西医内科学考核试卷及答案)

内科学专业考核 姓名:__________ 得分:_______ 一、单选题,以下各题有多个选项,其中只有一个选项是正确的,请选择正确答案(本大题满分40分,每小题1分) 1. 下列疾病中,除哪一种外均常导致症状性高血压:( ) A. 急性肾盂肾炎 B. 急性肾炎 C. 主动脉狭窄 D. 肾动脉狭窄 E. 嗜铬细胞瘤 2. 急性心肌梗死时,下列哪支动脉阻塞最易累及窦房结和房室结:( ) A. 右冠状动脉 B. 以上均不是 C. 左冠状动脉回旋支 D. 左冠状动脉主干 E. 左冠状动脉前降支 3. 甲亢是最有诊断意义的体征:( ) A. 弥漫性甲状腺肿伴血管杂音 B. 突眼 C. 脉压增大 D. 心脏增大 E. 心率加快,第一心音亢进 4. 金葡菌肺炎最具特征的X线表现是:( ) A. 肺实变伴多发性蜂窝样改变 B. 肺段实变伴液气胸 C. 肺段实变伴脓肿形成 D. 浸润阴影易变伴气囊腔形成 E. 多发性肺浸润 5. 心源性哮喘与哮喘难于鉴别时,治疗有选用:( )

A. 吗啡 B. 氨茶碱 C. 硝普钠 D. 洋地黄 E. 利尿剂 6. 我国高血压病引起的死亡原因最常见的是:( ) A. 脑血管意外 B. 尿毒症 C. 伴发冠心病 D. 高血压危象 E. 心力衰竭 7. 结核结节的病理改变与结核杆菌哪种成分有关:( ) A. 矿物质 B. 类脂质 C. 糖 D. 糖蛋白 E. 蛋白质 8. 肝硬化脾肿大的主要原因:( ) A. 肝动脉压力增高 B. 毒物刺激 C. 门静脉高压 D. 腹水压迫使脾血回流受阻 E. 肝静脉压力增高 9. 急性心肌梗死合并室性早搏应首:( ) A. 普萘洛尔 B. 胺碘酮 C. 利多卡因 D. 普鲁卡因胺 E. 给拉帕米 10. 对消化性溃疡诊断最有意义的是:( ) A. 胃液分析为高胃酸 B. 上腹部压痛 C. 周期性的上腹痛 D. 负餐透视局部有压痛,激惹,变形 E. 钡餐可见到龛影 11. 继发性高血压不见于下列哪种疾病:( ) A. 嗜铬细胞瘤 B. 慢性肾盂肾炎 C. 原发性醛固酮增多症 D. 肾上腺皮质功能减退 E. 先天性肾畸形

病例分析题目及答案内科学

女性,59 岁,间断咳嗽、咳痰 5 年,加重伴咯血 2 个月。 患者5年前受凉后低热、咳嗽、咳白色粘痰,给予抗生素及祛痰治疗, 1 个月后症状不 见好转,体重逐渐下降,后拍胸片诊为“浸润型肺结核”,肌注链霉素 1 个月,口服利福平、雷米封 3 个月,症状逐渐减轻,遂自行停药,此后一直咳嗽,少量白痰,未再复查胸片。2个月前劳累后咳嗽加重,少量咯血伴低热、盗汗、胸闷、乏力又来诊。病后进食少,二便正常,睡眠稍差。既往 6 年前查出血糖高,间断用过降糖药,无药物过敏史。 查体:T37.4 C, P94 次/分,R22次/分,130/80,—般稍弱,无皮诊,浅表淋巴结未触及,巩膜不黄,气管居中,两上肺呼吸音稍减低,并闻及少量湿罗音,心叩不大,心率94 次/分,律齐,无杂音,腹部平软,肝脾未触及,下肢不肿。 化验:血110g, 4.5 X 109, N 53%, L47%, 210 X 109, 35 ,空腹血糖9.6,尿蛋白(-),尿糖(3+) 一、诊断及诊断依据(4 分) 、鉴别诊断(1.5 分) 三、进一步检查(1.5 分) 四、治疗原则(3 分)

男性,65岁,持续心前区痛 4 小时。 4 小时前即午饭后突感心前区痛,伴左肩臂酸胀,自含硝酸甘油1 片未见好转,伴憋气、乏力、出汗,二便正常。既往高血压病史6年,最高血压160/100 ,未规律治疗,糖尿病病史 5 年,一直口服降糖药物治疗,无药物过敏史,吸烟10 年,每日20 支左右,不饮酒。 查体:T37C, P100次/分,R24次/分,150/90,半卧位,无皮疹及出血点,全身浅表淋巴结不大,巩膜无黄染,口唇稍发绀,未见颈静脉怒张,心叩不大,心律100次/分,律齐,心尖部H /6级收缩期吹风样杂音,两肺叩清,两肺底可闻及细小湿罗音,腹平软,肝脾未及,双下肢不肿。 化验:134, 9.6 X 109,分类:中性分叶粒72% 淋巴26% 单核2% 250X 109,尿蛋白微量,尿糖(2+),尿酮体(-),镜检(-) 一、诊断及诊断依据( 4 分) 、鉴别诊断(1.5 分) 三、进一步检查(1.5 分) 四、治疗原则(3 分)

心内科出科考试试题(有答案)

心内科出科考试试题 科室:姓名:分数: 一、名词解释(共9分) 1.射血分数:搏出量与心室舒张末期容积的百分比。 2.缺血-再灌注损伤:血液再灌注后缺血性损伤进一步加重的现象称为缺血-再灌注损伤。 3.心力储备:指心脏能适应机体代谢需要而提高心输出量的能力。或(心输出量能够随机体代谢的增强而增加的能力)。心力储备可用最大心输出量与安静时的心输出量之差值表示。 二、填空(共21分) 1.主动脉弓的凸侧发出三大分支,自右向左分别为头臂干(无名动脉)、左颈总动脉、左锁骨下动脉。 2.氯吡格雷是血小板ADP受体的拮抗剂 3.心脏骤停的最主要的特征是:意识丧失和大动脉搏动消失。 4.急性冠脉综合症包括:不稳定型心绞痛(UA)、急性非ST段抬高的心肌梗死(NSTEMI)、急性ST段抬高的心肌梗死(STEMI)。 5.收缩压的高低主要取决于心肌收缩力和心搏量;舒张压则主要取决于外周血管阻力。 6.急性冠脉综合征中非ST段抬高型病生理基础为血栓不完全堵塞动脉或微栓塞,而ST段抬高型则为血栓完全阻塞动脉血管。

7.三级高血压是指收缩压≥180mmHg和(或)舒张压≥110 mmHg。 8.心输出量等于每搏输出量乘以心率。 9.急性广泛前壁心肌梗死患者,于起病后3天在心尖区听到Ⅲ级收缩期杂音,应考虑合并乳头肌功能不全。 10.抗栓治疗可分为抗血小板治疗和抗凝血酶治疗。 三、是非题(共20分) 1.房颤的危险性远小于室颤. (√) 2.心房颤动患者,心室率为42次/分,QRS波形宽大畸形,但节律整齐,可以诊断为房颤合并三度房室传导阻滞。(√) 3.某预激综合症患者,发生阵发性室上性心动过速,可首选西地兰或心得安静脉注射治疗。(×) 4.平均动脉压=舒张压+1/2脉压。(×) 5.吗啡可以有效治疗心源性哮喘和支气管哮喘。(×) 6.胃肠道反应是强心甙中毒的早期指征。(√) 7.预激综合症伴心房颤动者禁用洋地黄治疗。(×)。 8.主动脉瓣狭窄患者常有晕厥发作并可能猝死。(√) 9.根治由旁路引发的折返性室上性心动过速,应首选射频消融治疗。(√) 10.强心甙治疗心房纤颤的机制主要是缩短心房有效不应期。(×)

内科学试题 病例分析

内科学试题病例分析 第一题、 病史摘要男性,64岁,咳嗽、咳痰、喘息30余年,活动后气促10余年,下肢水肿1周。30年来每年冬季咳嗽、咳痰、喘息,持续3-4个月,经抗感染及平喘治疗有所缓解。近10余年来于症状加重时出现活动后心悸、气促。1周前感冒症状加重,并出现少尿、下肢水肿,抗感染治疗效果不佳。发病以来食欲差,有时夜间发作呼吸困难,坐起后可有所减轻,体重无明显变化。否认高血压病、心脏病、结核病、肝病等病史,吸烟40余年,每日20支。 查体:T37.5°C,P110次/分,R26次/分,BP135/70mmHg,神志清,浅表淋巴结不大,巩膜无黄染,口唇略发绀,颈静脉怒张,桶状胸,双肺叩过清音,双肺呼吸音弱,呼气延长,双肺散在哮鸣音,肺底部可闻及少许湿性罗音,心界缩小,剑突下可见心尖搏动。肝肋下2cm,触痛阳性,肝颈静脉回流征阳性,睥肋下未及,移动性浊音可疑阳性。双下肢水肿(++)。辅助检查:WBC5×109,92%。 请回答诊断、诊断依据,鉴别诊断,需要作的进一步检查和治疗原则。 第二题、 病史摘要男性,60岁,胸骨后压榨性疼痛4小时。 4小时前无诱因突发胸骨后压榨性疼痛,伴胸闷、大汗、恶心,未吐。旁人给予硝酸甘油0.6mg舌下含服疼痛仍未缓解,遂来急诊,在连接心电监护仪时,病人突然抽搐,意识丧失,无二便失禁,经急诊医生胸外按压后,意识立即恢复。既往无冠心病、糖尿病、高血压病史,无药物过敏史,无烟酒嗜好。 查体:T36.5°C,P82次/分,R19次/分,BP90/60mmHg。神志清,巩膜无黄染,睑结膜无苍白,口唇无紫绀,双肺底细湿罗音,心界不大,心率82次/分,律不齐,可闻及早搏3-5次/分,心音稍低,未闻及杂音。腹平软,肝脾未及,双下肢

心血管内科试题及答案

心血管内科试题及答案 心血管内科学主治医师考试试题及答案 1.以下哪种情况是洋地黄适用证 A.预激综合征合并心房颤动 B.风心病,心衰,快速房颤 C.肥厚梗阻型心肌病 D.高度房室传导阻滞 E.急性心肌梗死 正确答案:B 2.最早能反映急性心肌梗死发生的酶学检查是 A.肌酸磷酸激酶() 亚型:31比值 D.α-羟丁酸脱氢酶(α) E.血清肌红蛋白 正确答案:B 3.下列哪种诊断技术诊断心包积液既安全又准确 A.心包穿刺术 B.心脏听诊 C.心电图 D.超声心动图 E.胸部X线摄片 正确答案:D 4.梗阻性肥厚型心肌病,胸骨左缘第3、4肋间收缩期喷射性杂音,其强度的改变,下列哪项正确 A.含硝酸甘油减弱 B.运动时减弱 C.屏气减弱 D.下蹲减弱 E.用心得安增强 正确答案:D 5.下列哪种情况常提示亚急性感染性心内膜炎 A.心房颤动 B.期前收缩 C.心脏扩大 D.新出现病理杂音 E.原有的病理杂音强度增加Ⅱ级以上,或杂音的性质、强度突然改变 正确答案:E 6.主动脉粥样硬化形成的主动脉瘤最常见于 A.冠状动脉开口处

B.肾动脉开口处 C.主动脉弓 D.降主动脉 E.腹主动脉 正确答案:E 7.亚急性细菌性心内膜炎,行人工瓣膜置换的指征是 A.脾肿大伴左上腹疼痛并闻及摩擦音 B.中度贫血有杵状指 C.抗生素治疗2周无效者 D.高热持续不退,白细胞持续升高 E.适当抗生素治疗但血培养持续阳性或反复复发 正确答案:E 8.男性,16岁,查体发现心尖部舒张期隆隆样杂音伴开瓣音,心率72次/分,律齐,肺无异常。肝脾未触及,下肢不肿。超声心动图示二尖瓣瓣口面积 1.72,平时活动无受限。应作哪项处理 A.抗生素预防感染性心内膜炎 B.二尖瓣分离术 C.洋地黄治疗 D.利尿剂治疗 E.避免重体力活动,定期随诊 正确答案:E 9.某患者服用地高辛0.25,共2周。出现下列何种情况应予停药 A.血钾降低 B.房颤心率由120次/分降为80次/分 C.心尖区收缩期杂音增强 D.心率40次/分,心尖区闻大炮音 E.血肌酐升高 正确答案:D 10.体循环淤血的最可靠体征为 A.静脉压升高 B.肝脏肿大及压痛 C.胸、腹腔积液 D.双下肢水肿 E.颈静脉曲张 正确答案:A 11.心绞痛发作的典型部位为 A.心前区 B.胸骨中、上段胸 骨后 C.胸骨下段

临床医学-内科学期末考试卷、试题及答案

以下每一考题以下有A、B、C、D、E 5个备选答案,请从中选一个最佳答案,并在答题卡将相应题号的相应字母所属方框涂黑。 1.下列哪一项是气肿型阻塞性肺气肿的临床表现 A.多见于青年 B.有紫绀 C.气促明显,多是持续性 D.桶状胸不明显 E.呼吸音正常 2.引起慢性肺原性心脏病的最常见病因是 A.支气管扩张 B.支气管哮喘 C.尘肺 D.慢支并发阻塞性肺气肿 E.慢性弥漫性肺间质纤维化 3.下列哪一项为外源性哮喘的临床表现 A.过敏原皮试明性 B.IgE正常 C.多发于老年 D.间歇性发作 E.嗜酸粒细胞正常 4.成人呼吸窘迫综合征的常见死因是 A.多脏器功能衰竭 B.DIC C.肺水肿 D.自发性气胸 E.纵隔气肿 5.最常见的肺炎类型是 A.细菌性肺炎 B.病毒性肺炎 C.支原体肺炎 D.真菌性肺炎 E.过敏性肺炎 6.肺脓肿发病的最主要原因是 A.血源性痈疖所致败血症播散 B.继发性支气管扩张 C.继发性支气管肺癌 D.食管穿孔感染穿破至肺形成肺脓肿 E.吸入性肺脓肿 7.洋地黄类的毒性反应最早出现的是 A.恶心、呕吐 B.食欲不振 C.头晕、头痛 D.视力模糊

8.下列各类型心绞痛中,属于劳累性心绞痛的是A.卧位型心绞痛 B.变异型心绞痛 C.梗塞后心绞痛 D.恶化型心绞痛 E.急性冠状动脉功能不全 9.治疗变异型心绞痛,应首选 A.肾上腺素能β受体阻滞剂 B.钙通道阻滞剂 C.冠状动脉扩张剂 D.硝酸异山梨醇酯 E.长效硝酸甘油制剂 10.发生心肌梗塞的最先出现的症状是 A.心律失常 B.低血压和休克 C.疼痛 D.呼吸困难,疼痛 E.恶心,呕吐 11.为缓解二尖瓣梗阻,应首选 A.静脉注射利尿剂 B.经皮球囊二尖瓣成形术 C.闭式分离术 D.直视分离术 E.人工瓣膜置换术 12.急性心包炎心包渗液时最突出的症状是 A.心前区痛 B.干咳、嘶哑 C.呼吸困难 D.吞咽困难 E.烦躁不安 13.渗液性心包炎的体征是,Ewart征是指 A. 心浊音界向两例增大,呈绝对浊音 B. 心兴冲动减弱,位于心浊音界左缘的内侧或不能扪及C.背部左肩肿角下呈浊音,语颤增强和支气管呼吸音D.心音低而遥远 E.胸骨右缘第3—6肋间出现实音 14.治疗因胆汁返流引起的B、型胃炎,应首选 A.考来烯胺 B.多潘立酮 C.甲氧氯普胺 D.西沙必利 E.胶体次枸橼酸铋 15.血吸虫性肝硬化的病理类型是

相关主题
文本预览
相关文档 最新文档